Вы находитесь на странице: 1из 229

Module 1.

General and special questions of clinical laboratory


diagnostics
Text test question
1. ____ is a hormone produced by the ____ when tissue levels of oxygen are low.
A. Bilirubin stomach
B. Bilirubin; bone marrow
C. * Erythropoietin; kidneys
D. Erythropoietin; bone marrow
E. Hemoglobin; liver
2. A L L occurs in which of the following:
A. Is more common in adults.
B. * is more common in Children.
C. is extremely rare in children and unusual before the age of 40.
D. has a peak age of 30-50.
E. All of the above.
3. Absolute erythrocytosis may lead to
A. Liver insufficiency
B. Kidney insufficiency
C. Remove of stomach
D. Ionizing irradiation
E. * Cardiac insufficiency
4. Acute leukemia is characterized by:
A. Acantolytic cells
B. Mature cells
C. * Poorly differentiated cells with many blasts.
D. Low serum ferritin
E. None of the above.
5. Acute myelocytic leukemia includes the following except:
A. Myeloblastic.
B. Promyelocyte.
C. Monocyte.
D. Myelomonocytic.

E. * Prolymphocytic.
6. Albumin contents of normal plasma is:
A. 55-75 g/l
B. 45-85 g/l
C. * 35-50 g/l
D. 55-95 g/l
E. 15-25 g/l
7. ALL is more common in:
A. * Children
B. Adults
C. Neonates
D. Adults above 60
E. None of the above
8. All of the following are true of erythrocytes except
A. When mature, they have no nucleus or other organelles
B. Their plasma membrane contains many antigens (molecules projecting
from the surface)
C. They normally contain practically all of the hemoglobin that is present
in blood
D. They transport oxygen from the lungs to body tissues
E. * They have a finite life span averaging about 60 days
9. All of the following cause Microcytic Hypochromic anemia except:
A. * Lead poisoning
B. Thalassemia
C. Iron deficiency anemia
D. Fanconi's anemia
E. Nothing is correct
10. An excessive number of white blood cells is called:
A. Lymphoma
B. Leukopenia
C. * Leukocytosis
D. All of the above
E. None of the above

11. Anemia is hereditary enzymepathy on the base of glucose-6-phosphat


dehydrogenase insufficiency. Type of hereditance it is:
A. Autosome-recessive
B. Linked X-chromosome, recessive
C. Autosome-dominant
D. Intermediate
E. * Linked X-chromosome, dominant
12. Anemia resulting from B12 deficiency is called :
A. * Pernicious anemia
B. Hemorragic anemia
C. Aplastic anemia
D. Sickle cell anemia
E. Nutritional anemia
13. Anemia, thrombocytopenia is prominent feature of..
A. Chronic leukemia
B. * Acute leukemia
C. Both Acute and Chronic
D. None
E. Subacute leukemia
14. Anticoagulants are:
A. Heparin
B. * Everything is correct
C. Ethylene diamine tetra acetic acid
D. Sodium fluoride
E. Potassium or sodium oxalate
15. At the deficit of Villebrands factor is disordered
A. Retraction of clot
B. Aggregation of thrombocytes
C. Formation of fibrin
D. Activation of prothrombin
E. * Adgesion of thrombocytes
16. B cells function by promoting ____ .

A. Phagocytosis
B. * Antibody production
C. Release of histamine
D. Cell to cell killing of viral infected cells
E. The production of erythropoietin
17. B12-deficiency anemia developed in a patient after the total resection of
stomach. What type of cells are present in blood?
A. Anulocytes
B. Microcytes
C. Ovalocytes
D. Spherocytes
E. * Megalocytes
18. Because of local intravessels of blood coagulation arises
A. Slage-syndrome
B. Embolism
C. DIC-syndrome
D. Arterial hyperemia
E. * Thrombosis
19. Bence Jones protein is a Paraprotein and it is characteristics of the following:
A. * Multiple myeloma
B. Sidoroblastic anemia
C. Aplastic anemia
D. Iron deficiency
E. Leukemia
20. Biochemical investigations can be performed on types of blood specimens:
A. 2
B. * 4
C. 6
D. 8
E. 10
21. Blood perform such functions:
A. Gas transport

B. Transport of nutritional substances


C. Regulative
D. Osmotic
E. * All of the above
22. By strong inhibitors of aggregation of thrombocytes is
A. Encephalin
B. Prostaglandin
C. Estrogen
D. Globulin
E. * Prostacyclin
23. By the genetic marker of myeloleucosis is philadelphian chromosome. What
chromosomal aberration it formed as a result of?
A. Inversion of short shoulder of 21-th chromosome
B. Deletion of short shoulder of 22-th chromosome
C. Translocation of short shoulder of 22-th chromosome on 21-th
D. Duplication of long shoulder of 22-th chromosome
E. * Translocation of long shoulder of 22-th chromosome on 9-th
24. Categories of discretionary tests:
A. * Everything is correct
B. To confirm a diagnosis
C. To aid differential diagnosis
D. To refine a diagnosis
E. To asses the severity of disease
25. Categories of selective tests:
A. * Everything is correct
B. To monitor progress
C. To aid differential diagnosis
D. To detect complications or side effects
E. To monitor therapy
26. Choose the indexes of neutrophilic band granulocytes :
A. 19 37 %
B. 3 11 %

C. 0 1 %
D. * 1 6 %
E. 45 72 %
27. Choose the indexes of normal basophilic granulocytes:
A. 19 37 %
B. 3 11 %
C. * 0 1 %
D. 0,5 5 %
E. 45 72 %
28. Choose the indexes of normal concentrations of RBC in male:
A. 3-4,1? 1012/L
B. 2,1-5,1? 1012/L
C. * 4-5,1? 1012/L
D. 12-15 ? 1012/L
E. 5-10? 1012/L
29. Choose the indexes of normal concentrations of RBC in female:
A. * 3,7-4,7? 1012/L
B. 2,1-5,1? 1012/L
C. 4-5,1? 1012/L
D. 12-15 ? 1012/L
E. 5-10? 1012/L
30. Choose the indexes of normal eosinophilic granulocytes:
A. 19 37 %
B. 3 11 %
C. 0 1 %
D. * 0,5 5 %
E. 45 72 %
31. Choose the indexes of normal hematocrit in female:
A. 3,7-4,7 %
B. 2,1-5,1 %
C. * 36- 42 %
D. 12-15 %

E. 50-60 %
32. Choose the indexes of normal hematocrit in male:
A. 3,7-4,7 %
B. 2,1-5,1 %
C. * 40- 48 %
D. 12-15 %
E. 50-60 %
33. Choose the indexes of normal lymphocytes:
A. * 19 37 %
B. 3 11 %
C. 0 1 %
D. 0,5 5 %
E. 45 72 %
34. Choose the indexes of normal maintenance of hemoglobin in women:
A. 140-160 g/l
B. * 120-140 g/l
C. 125-160 g/l
D. 50-80 g/l
E. 70-100 g/l
35. Choose the indexes of normal maintenance of hemoglobin in men:
A. 80-100 g/l
B. * 130-160 g/l
C. 115-145 g/l
D. 70-90 g/l
E. 100-120 g/l
36. Choose the indexes of normal monocytes:
A. 19 37 %
B. * 3 11 %
C. 0 1 %
D. 0,5 5 %
E. 45 72 %
37. Choose the indexes of normal neutrophilic segmented granulocytes:

A. 19 37 %
B. 3 11 %
C. 0 1 %
D. 0,5 5 %
E. * 45 72 %
38. Choose the indexes of normal plateletes CBC:
A. 100-220 ? 109/L
B. 100-120 ? 109/L
C. 80-120 ? 109/L
D. * 180-320 ? 109/L
E. 10-32 ? 109/L
39. Choose the indexes of normal reticulocytes CBC:
A. 3,7-4,7 %
B. 2,1-5,1 %
C. 36- 42 %
D. * 0,5-1 %
E. 50-60 %
40. Choose the indexes of normal total leukocytes CBC:
A. 10-20 ? 109/L
B. 1-2 ? 109/L
C. 8-12 ? 109/L
D. * 4-9 ? 109/L
E. 10-32 ? 109/L
41. Circulating mature RBCs lack:
A. Ribosomes
B. * Mitochondria
C. Nuclei
D. All of the above
E. None of the above
42. Classes of immunoglobulin produced by multiple myeloma:
A. IgM
B. IgD

C. * IgG
D. IgE
E. IgA
43. Classification of leukemia:
A. * Acute and chronic
B. Mild and Grievous
C. Hemorrhagic and post Hemorrhagic
D. Microcytic and Macrocytic.
E. Hyperchromic and Hypochromic
44. Combination of haemoglobin with oxygen named:
A. Carboxyhemoglobin
B. Methemoglobin
C. Carbylaminhemoglobin
D. Carbhemoglobin
E. * Oxyhemoglobin
45. Combination, that transported CO2 from tissues to lung named:
A. Methemoglobin
B. * Carbhemoglobin
C. Oxyhemoglobin
D. Carbylaminhemoglobin
E. Carboxyhemoglobin
46. Complete blood count in patients with vitamin B12 deficiency anaemia shows
all changes except of the following:
A. Decreased amount of erythrocytes and haemoglobin
B. Macrocytosis
C. Increased colour index more than 1,1
D. * Decreased colour index below 0.8
E. Zhollis bodies and Kebots rings in erythrocites
47. Coomb's Positive Hemolytic Anaemia is seen in except:
A. * Alcoholic cirrhosis
B. Chronic active hepatitis
C. Primary biliary cirrhosis

D. Primary sclerosing cholangitis


E. Nothing is correct
48. Core biochemical tests, except:
A. Sodium, potassium, chloride and bicarbonate
B. * Hormones
C. Glucose
D. Everything is correct
E. Total protein
49. Core biochemical tests, except:
A. Sodium, potassium, chloride and bicarbonate
B. * Vitamins
C. Glucose
D. Everything is correct
E. Total protein
50. Core biochemical tests, except:
A. Sodium, potassium, chloride and bicarbonate
B. * DNA analyses
C. Glucose
D. Everything is correct
E. Total protein
51. Core biochemical tests:
A. Urea and creatinine
B. Bilirubin
C. Glucose
D. * Everything is correct
E. Amylase
52. Core biochemical tests:
A. Sodium, potassium, chloride and bicarbonate
B. Calcium
C. Glucose
D. * Everything is correct
E. Total protein

53. Cyanosis of skin develops as a result increase in the blood capillaries:


A. Carboxyhemoglobin
B. Erythrocytes
C. Methemoglobin
D. Carbhemoglobin
E. * Deoxyhemoglobin
54. Deficit of what vitamin in the liver leads to decrease in coagulation factors II,
V, VIII?
A. B6
B. C
C. PP
D. B1
E. * K
55. Definative diagnosis of Hodgkins Lymphoma is:
A. * Lymph node biopsy
B. General blood test
C. General urine analysis
D. X-ray of the chest
E. None of the Above
56. Definitive diagnosis of lymphogranulomatosis is made by:
A. General blood test
B. Biochemical blood analysis
C. Plasmaphoresis
D. * Lymph node biopsy
E. General urine analysis.
57. Destruction of erythrocytes in ecquired hemolytic anemia occurs in
A. Macrophages of liver
B. Macrophages of spleen
C. Lymphatic nodes
D. Intracellular liquid
E. * Blood vessels
58. Detoxification function of blood is conditioned:

A. Gas transport (CO2 and O2)


B. Transport of nutritional substances
C. Exchange of heat between tissues and blood
D. * Detoxification toxic substanses by the enzymes of blood
E. Presents in blood of antibodies and by the phagocit function of
leucocytes
59. Electrophoresis of blood proteins carry out at pH:
A. 5,5
B. 7,0
C. * 8,6
D. 4,7
E. 3,0
60. Eosinophils are involved in which of the following?
A. Phagocytosis
B. * Parasitic infection
C. Viral infection
D. All of the above
E. None of the above
61. Erythrocytes sedimentation rate (ESR) for males is:
A. 1 12 mm/hr
B. 1 10 mm/hr
C. * 2 10 mm/hr
D. 3 15 mm/hr
E. 2 15 mm/hr
62. Erythropoietin:
A. * Stimulates red blood cell synthesis
B. Stimulates white blood cell synthesis
C. Is released in response to a decrease in blood flow to the bone marrow
D. A and C
E. B and C
63. Erytropoietin synthesis is disordered in chronic kidney insufficiency.
Development of what blood elements will be decreased?
A. Neutrophils

B. Monocytes
C. Thrombocytes
D. Lymphocytes
E. * Erythrocytes
64. Examples of tests used in case-finding programmes in neonates:
A. Plasma cholinesterase activity
B. * Serum [TSH] and/or [thyroxine]
C. Drug screen
D. Maternal serum [a-fetoprotein]
E. Nothing is correct
65. Examples of tests used in case-finding programmes in neonates:
A. Plasma [albumin] and/or [pre-albumin]
B. * Serum [phenylalanine]
C. Plasma and urine [glucose]
D. Maternal serum [a-fetoprotein]
E. Everything is correct
66. Examples of tests used in case-finding programmes in pregnancy:
A. Plasma [albumin] and/or [pre-albumin]
B. Serum [phenylalanine]
C. Drug screen
D. * Maternal serum [a-fetoprotein]
E. Everything is correct
67. Fibrinogen contents of normal plasma is:
A. 5-7 g/l
B. 5-8 g/l
C. * 2-4 g/l
D. 5-9 g/l
E. 1-2 g/l
68. Fluoride is also anticoagulant. It should not be used for:
A. Blood urea
B. Blood albumin
C. * Enzyme assays

D. Blood bilirubin
E. Blood creatinine
69. Globulins contents of normal plasma is:
A. 5-75 g/l
B. 45-85 g/l
C. * 25-35 g/l
D. 55-95 g/l
E. 15-25 g/l
70. Haemoglobin A of erythrocytes in the adult include:
A. 22 and 1 polypeptide chains
B. 11 and 2 polypeptide chains
C. 4 4 - polypeptide chains
D. 4 - polypeptide chains
E. * 2 and 2 - polypeptide chains
71. Haemoglobin A of erythrocytes in the adult include:
A. 22 and 1 polypeptide chains
B. 11 and 2 polypeptide chains
C. 4 4 - polypeptide chains
D. 4 - polypeptide chains
E. * 2 and 2 - polypeptide chains
72. Hemoglobine of erythrocytes include:
A. * Hem and globin
B. Histones and hem
C. Protamines and hem
D. Globin and NAD
E. Iron, copper and protein
73. Hemoglobine of erythrocytes include:
A. * Hem and globin
B. Histones and hem
C. Protamines and hem
D. Globin and NAD
E. Iron, copper and protein

74. High level of reticulocytes in peripheral blood smear is indicative of which of


the following?
A. * Post hemmorhagic anemia
B. Iron deficiency anemia
C. CLL
D. AML
E. None of the above
75. Hodgkins lymphoma is also known as:
A. Lymphoma
B. * Lymphogranulomatosis
C. Lipoma
D. Mesothelioma
E. Ependymoma
76. How is anaemia named, in base which lie decrease of enzymes activition
which take part in the hem synthesis?
A. Metaplastic
B. Sickle cell
C. Toxic-hemolytic
D. Irondeficiency
E. * Sideroblastic
77. Hypoxic hypoxia causes change in the system of blood, namely:
A. Decrease of erythrocytes without the changes of Hb
B. Increase of erythrocytes without the changes of Hb
C. Increase of Hb without the changes of erythrocytes
D. Decrease of erythrocytes and Hb
E. * Increase of erythrocytes and Hb
78. If the prominent cell line is of myeloid series, it is
A. Lymphocytic leukemia
B. * Myelocytic leukemia
C. Myelolymphocytic leukemia
D. Basophilic leukemia
E. All of the above
79. Immune status of organism is provided by:

A. * Leukocytes
B. Trombocytes
C. Erythrocytes
D. Hemoglobin
E. Bilirubin
80. In eight years old child in general blood test is revealed: erythrocytes
1.2*1012/1, hemoglobin 34 g/l, color index 0.9 , thrombocytes 50*109/1,
leukocytes 2.3*109/1, blasts 30%, neutrophyls 22%, lymphocytes 43%,
monocytes 5%, ESR 62 mm/hour. Characterize these changes.
A. Hemophilia
B. Thrombocytopenic purpura
C. Henoch's disease
D. Anemia
E. * Leukosis
81. In general, biochemical tests can be broadly divided into groups:
A. * 2
B. 4
C. 6
D. 8
E. 10
82. In microscopy of a peripheral blood smear, auer rods where seen. Which of
the following diseases is it seen in?
A. CML
B. * AML
C. CLL
D. ALL
E. None of the above.
83. In Polycythemia vera, all the following are seen except:
A. * Thrombocytopenia
B. Increased GI bleed
C. Thrombosis
D. Transient visual loss
E. Nothing is correct

84. In ten years old boy in general blood count is revealed: erythrocytes
1.2*1012/1, hemoglobin 34 g/l, color index 0.9 , thrombocytes 50*109/1,
leukocytes 12.3*109/1, blasts 45 %, neutrophyls 22%, lymphocytes 28%,
monocytes 5%, ESR 52 mm/hour. Characterize these changes.
A. Hemophilia
B. Thrombocytopenic purpura
C. Henoch's disease
D. Anemia
E. * Leukosis
85. In the sickle cell anemia is synthesized anomalous HbS as a result of gene
mutation, in which in place of glutamic acid present
A. Alanin
B. Cystine
C. Leucin
D. Tyrosine
E. * Valin
86. In which of the following types of white blood cells do the cytoplasmic
granules stain preferentially with red-staining dyes?
A. Neutrophils
B. Basophils
C. * Eosinophils
D. Lymphocytes
E. Monocytes
87. Increase related to O2 in the perinatal period of child has:
A. HbS
B. * HbF
C. HbA
D. HbE
E. HbC
88. It is known that in pathological condition erythroblastic type of blood
formation in bone marrow may change on megaloblastic. It is characterized
for:
A. Cancer of duodenum
B. Tuberculous intoxication
C. Sickle

D. Cronical blood loss


E. * B12-deficiency anemia
89. Leukocytes are divided into two classes based on the presence or absence of
microscopically visible structures called
A. Nuclei
B. * Granules
C. Ribosomes
D. Mitochondria
E. Golgi complexes
90. Leukocytes are divided into two classes based on the presence or absence of
microscopically visible structures called
A. Nuclei
B. * Granules
C. C. Ribosomes
D. Mitochondria
E. Golgi complexes
91. Lymphogranulomatosis is characterized by growth of giant cells called:
A. Sickel cells
B. Romanovich cells
C. Hodgkins cells
D. Burr cells.
E. * Reed-Sternberg cells.
92. Maximum ESR is seen in:
A. * Multiple myeloma
B. CHF
C. Polycythemia vera
D. Sickle cell anemia
E. Nothing is correct
93. Megaloblastic anemia includes which of the following?
A. B12 deficiency anemia
B. Folic acid deficiency
C. Hemolytic anemia
D. * A and B

E. B and C
94. Most of the circulating leukocytes are:
A. Basophils
B. Eosinophils
C. Leukocytes
D. Monocytes
E. * Neutrophils
95. Multiple myeloma can be diagnosed with which of the following?
A. Serum protein electrophoresis
B. Bone marrow examination
C. Urine protein electrophoresis
D. X-ray of the the involved bones
E. * All of the above
96. Multiple myeloma is also known as
A. * Kahlers disease
B. Hodgkins disease
C. Reed- Stenberg disease
D. Arthurs syndrome.
E. All of the above.
97. Multiple myeloma is also known as:
A. Lymphogranulomatosis.
B. * Plasma cell myeloma
C. Myelomatosis.
D. Hodgkins disease.
E. Granulomatosis
98. Myeloma is diagnosed with:
A. Blood tests
B. Bone marrow examination
C. X-rays of commonly involved bones
D. Urine protein electrophoresis
E. * All of the following
99. Normal anisocytosis is :

A. * 11-14 %
B. 24-33 %
C. 15-35 %
D. 50-100 %
E. 1-10 %
100.

Normal concentration of ferritin in blood serum is:


A. 5-10 ng/dL
B. 40-60 ng/dL
C. * 45-340 ng/dL
D. 450-640 ng/dL
E. 450-550 ng/dL

101.

Normal Iron binding capacity, total is:


A. * 30,6 - 84,6 micromole/l
B. 306 - 846 micromole/l
C. 3 - 8 micromole/l
D. 10,6 -14,6 micromole/l
E. 6 - 8 micromole/l

102.

Normal level of MCHC?


A. 17-25 %
B. 30-50 %
C. 40-57 %
D. 60-75 %
E. * 33-37 %

103.

Normal MCH is:


A. 80-90 pg
B. * 24-33 pg
C. 15-35 pg
D. 50-100 pg
E. 1-10 pg

104.

Normal MCV is:


A. Blood loss, decreased production of blood.
B. * Blood loss, excessive production, and excessive destruction of blood.

C. Impaired or decreased production of blood, blood loss.


D. Blood loss, excessive destruction of RBC, impaired or decreased
production of RBC.
E. Blood loss, increased production, and excessive destruction of blood.
F. 80-90 fl
G. 80-100 fl
H. 15-35 fl

105.

I.

50-100 fl

J.

1-10 fl
Normal RDW is:

A. * 11-14 %
B. 24-33 %
C. 15-35 %
D. 50-100 %
E. 1-10 %
106.

Normal Soluble transferrin receptor is:


A. * 1.8 4.6 mg/L
B. 306 - 846 micromole/l
C. 3 - 8 micromole/l
D. 10,6 -14,6 micromole/l
E. 6 - 8 g/l

107.

Normal Transferrin Saturation is:


A. * 20 45 %
B. 10 35 %
C. 20 30 %
D. 2 4 %
E. 10 15 %

108.
Normal value of MCHC (mean corpuscular hemoglobin concentration)
in blood:
A. 44 45 %
B. 45 47 %
C. * 33 37 %
D. 36 47 %

E. 57 60 %
109.

Normal value of platelets in blood:


A. 120-220 x 109/l
B. 150-180 x109/l
C. * 180-320 x109/l
D. 18-32 x109/l
E. None of the above

110.
?Phenylketonuria of newborn was diagnosed after a reaction of urine
with:
A. CuSO4
B. NaCl
C. Fe+
D. Na3PO4
E. * FeCI3
111.

Requirements for well-population screening:


A. The disease is common or life-threatening
B. The tests are sensitive and specific
C. The tests are readily applied and acceptable to the population to be
screened
D. Clinical, laboratory and other facilities are available for follow-up
E. * Everything is correct

112.

Screening may take . forms:


A. * 2
B. 4
C. 6
D. 8
E. 10

113.
Sideroblastic anemia often arises at treatment by some
antituberculousis drugs (isoniasid), because in the process of treatment
appears a deficit of vitamin
A. C
B. 12
C. A
D. K

E. * 6
114.

Smudge cells are characteristic for ?


A. CML
B. ALL
C. AML
D. * CLL
E. None of the above

115.

Sodium fluoride is usually used as a preservative for:


A. Blood urea
B. Blood albumin
C. * Blood glucose
D. Blood bilirubin
E. Blood creatinine

116.

Specialized tests, except:


A. Hormones
B. DNA analyses
C. Trace elements
D. * Total protein
E. Drugs

117.

Specialized tests:
A. Hormones
B. DNA analyses
C. Trace elements
D. * Everything is correct
E. Drugs

118.

Splenomegaly is often prominent in..


A. * Chronic leukemia
B. Acute leukemia
C. Both Acute and Chronic
D. None
E. Subacute leukemia

119.

The basic function of erythrocytes in blood:

A. Promotes agregation of thrombocytes


B. * Transport of C2 and O2
C. Syntheses of proteins of the contraction system (actine, myosine)
D. Provide immune status of organism
E. Take part in formation of active forms of oxygen
120.

The basic function of erythrocytes in blood:


A. Promotes agregation of thrombocytes
B. * Transport of C2 and O2
C. Syntheses of proteins of the contraction system (actine, myosine)
D. Provide immune status of organism
E. Take part in formation of active forms of oxygen

121.
The biological fluids employed in the clinical biochemistry laboratory
include:
A. Blood
B. Urine
C. Saliva
D. * Everything is correct
E. Tissue and cells
122.
The biological fluids employed in the clinical biochemistry laboratory
include:
A. Cerebrospinal fluid
B. Peritoneal fluide
C. Saliva
D. * Everything is correct
E. Stones
123.

The coagulation test includes:


A. ESR, partial thromboplastine time, prothrombine time.
B. Platelet count, bleeding time, prothrombine time, concentration of
fibrinogen in plasma.
C. * Partial thromboplastine time, prothrombine time, concentration of
fibrinogen in plasma.
D. Hematocrit, bleeding time, clotting time, concentration of fibrinogen in
plasma.

E. Blood type, clotting time, partial thromboplastine time, prothrombine


time.
124.

The etiology of Leukemia is:


A. * Unknown
B. Viruses
C. Bacteria
D. Radiation
E. Drugs and Chemicals

125.

The following are features of acute leukemias except:


A. Onset is usually rapid.
B. Disease is very aggressive.
C. The cells involved are usually poorly differentiated.
D. * The cells involved are usually more mature cells.
E. Presence of many blast cells.

126.

The following are seen in bone marrow aspiration in CML except:


A. Myeloid hyperplasia
B. Relatively few blast cells
C. Mostly mature neutrophils
D. Increased megakaryotes
E. * Mostly immature neutrophils

127.

The lymphocyte that is responsible for cell-mediated immunity is the:


A. * T lymphocyte
B. NK cell
C. B lymphocyte
D. None of the above
E. All of the above

128.

The most numerous white blood cell in normal blood is the:


A. * Neutrophil
B. Lymphoctye
C. Monocyte
D. Eosinophil
E. B lymphocyte

129.

The normal life span of the red cell is:


A. * 120 days
B. 120 hours
C. 30 days
D. 3-5 days
E. 24 hours

130.
The oxyphylic normocytes were appeared in the blood of a patient after
acute blood loss. 25 % of reticulocytes were found with a supravital dye.
Name the type of this anemia according to the bone marrow capacity to
regeneration?
A. Aregenerative
B. Hyperegenerative
C. Hyporegenerative
D. Disregenerative
E. * Regenerative
131.

The primary function of a mature red blood cell is:


A. Defense against toxins and pathogens
B. Delivery of enzymes to target tissues
C. Transport of respiratory gases
D. * All of the above
E. None of the above

132.

The results of laboratory tests may be of use in:


A. Diagnosis and in the monitoring of treatment.
B. Screening for disease or in assesing the prognosis.
C. Reseach into the biochemical basis of disease
D. Clinical trials of new drugs
E. * Everything is correct

133.

The Schilling test is a medical investigation used for patients with:


A. * Vitamin B12 deficiency
B. Hepatitis
C. Leukosis
D. Thrombocytopenia
E. Non of above

134.

The ultimate source of all types of blood cells are the...


A. Thrombocytes
B. * Hemocytoblasts
C. Myeloid stem cells
D. Lymphoid stem cells
E. Granular leukocytes

135.

The white blood cell that is most like the mast cell is the:
A. * Basophil
B. Lymphocyte
C. Neutrophil
D. Eosinophil
E. Monocyte

136.
To measure or differentiate anemia of mixed causes and forms such as
anisocytosis, which of the following is most appropriate to use?
A. Red cell distribution width
B. MCHC
C. * MCV
D. Non of above
E. MCH
137.
To the boy of 8 months that is cured because of pneumonia and rickets
moderate severity anemia was diagnosed. What indicators of hemoglobin in
the blood are characteristic for this degree of anemia?
A. * 70-89 g / l
B. 90 - 110 g / l
C. 80 - 100 g / l
D. 69 g / l and less
E. 100 - 120 g / l
138.
To the girl of 12 months that is cured because of pneumonia and rickets
mild severity anemia was diagnosed. What indicators of hemoglobin in the
blood are characteristic for this degree of anemia?
A. 70-89 g / l
B. * 90 - 110 g / l
C. 80 - 100 g / l
D. 69 g / l and less

E. 100 - 120 g / l
139.

Total protein contents of normal plasma is:


A. 55-75 g/l
B. 45-85 g/l
C. * 65-85 g/l
D. 55-95 g/l
E. 15-25 g/l

140.

Urine preservatives:
A. Formalin
B. Thymol
C. * Everything is correct
D. Chloroform
E. Concentrated HCl

141.

Watermelon stomach is characteristic of the following:


A. Vitamin B12
B. Iron deficiency
C. Malaria
D. Jaundice
E. * Hodgkins lymphoma

142.

What anemia characteristic with megaloblastic type of blood forming?


A. Hypoplastic
B. Toxicohemolytic
C. Metaplastic
D. Posthemorrhagic
E. * Pernicious

143.
What are the main laboratory findings in patient with chronic lymphatic
leukemia?
A. Thrombocytosis
B. * Anemia and thrombocytopenia
C. Thrombocytosis and lymphocytosis
D. No findings,
E. Philadelphia chromosome in abnormal cells

144.
What does mean of erythrocytosis in a sick with heart congenital
defect?
A. Independent disease
B. Complication
C. Terminal state
D. Index of convalescence
E. * Compencatory reaction
145.

What function of blood belong a concept oncotic pressure of blood?


A. Gas transport
B. Protective
C. Detoxification
D. Termoregulation
E. * Osmotic

146.

What is found in multiple myeloma:


A. * Hypercalcemia
B. Increased Alkaline phosphatase
C. Decreased IgA
D. Hypouricemia
E. Nothing is correct

147.

What is not seen in multiple myeloma?


A. * Increased alkaline phosphatase
B. Anemia
C. Hypercalcemia
D. Ted ESR
E. Nothing is correct

148.

What is not seen in polycythemia vera?


A. * Increase erythropoietin level
B. Increase RBC count
C. Increased Vit B12 binding capacity
D. Ocular congestion
E. Nothing is correct

149.

What is the normal value for total serum transferrin in blood?

A. * 1.7 4.7 mg/l


B. 1.7 8.5 mg/l
C. C.1.7 10.0 mg/l
D. D.1.7 12.0 mg/l
E. E.1.7 15.0 mg/l
What organs and in what order take part in embryonic hematopoiesis?

150.

A. Blood loss, decreased production of blood.


B. Blood loss, excessive production, and excessive destruction of blood.
C. * Impaired or decreased production of blood, blood loss.
D. Blood loss, excessive destruction of blood, impaired or decreased
production of blood.
E. Blood loss, increased production, and excessive destruction of blood.
F. 110-90 g/L, 3.5-3*1012/L
G. 90-70 g/L, 3,0-2.5*1012/L
H. 70-50 g/L, 2.5-2*1012/L
I.

120-100 g/L, 3,5-4*1012/L

J.

Less than 50 g/L, less than 2.0*1012/L

K. Hyperleucocytosis or leucopenia, anemia, blasts in peripheral blood


more than 5 %.
L. Lymphocytosis, hyperleucocytosis, anemia, thrombocytopenia, blasts
in peripheral blood more than 20 %.
M. Leucocytosis, anemia, monocytosis, blasts in peripheral blood more
than 10 %.
N. Anemia, thrombocytosis, eosynophilia, leucopenia, blasts in peripheral
blood more than 5 %.
O. Hyperleucocytosis or leucopenia, anemia, thrombocytopenia, blasts in
peripheral blood, blasts in marrow aspirate more than 20 %.
P. Spleen, liver, bone marrow
Q. Yolk sac, liver, kidneys, bone marrow
R. Yolk sac, liver, spleen, bone marrow
S. Spleen, liver, blood vessels, bone marrow
T. Yolk sac, spleen, muscles, liver
151.
What quantitative measurements are necessary to establish a
diagnosis and to monitor the disease Multiple myeloma?
A. Potassium

B. Vit. B12
C. * Paraprotein
D. Iron
E. All of the Above.
152.

When oxygen is carried by the blood, it is bonded to


A. Platelets.
B. Antibodies.
C. Plasma.
D. * Hemoglobin.
E. Water

153.
Which formed elements are most directly associated with the immune
responses that defend the body against pathogens?
A. Erythrocytes
B. * Leucocytes
C. Platelets
D. None of the above (the immune response is strictly a function of
plasma)
E. All of the above
154.
Which formed elements are most directly associated with the immune
responses that defend the body against pathogens?
A. Erythrocytes
B. * Leucocytes
C. Platelets
D. None of the above (the immune response is strictly a function of
plasma)
E. All of the above
155.

Which indexes are responsible for the normal volume of blood in adult?
A. 15 L
B. 25 L
C. * 5 L
D. 50 L
E. 2 L

156.

Which is the most common type of blood cell in a healthy human?


A. * Erythrocytes

B. Monocytes
C. Lymphocytes
D. Eosinophils
E. Basophils
157.

Which is the most common type of blood cell in a healthy human?


A. * Erythrocytes
B. Monocytes
C. Lymphocytes
D. Eosinophils
E. Basophils

158.

Which of the following blood cells has phagocytic function?


A. * Monocyte
B. RBC
C. Basophils
D. Lymphocytes
E. Platelets

159.
Which of the following cells play a crucial role in the pathogenesis of
alveolar-capillary damage in adult respiratory distress syndrome (ARDS)?
A. CD4-positive lymphocytes
B. CD8-positive lymphocytes
C. Eosinophils
D. Mast cells
E. * Neutrophils
160.

Which of the following is a type of leukocyte?


A. Macrophage
B. Eosinophil
C. Monocyte
D. * All of the above
E. None of the above

161.

Which of the following is a type of leukocyte?


A. Macrophage
B. Eosinophil

C. Monocyte
D. * All of the above
E. None of the above
162.

Which of the following is NOT a type of granular white blood cell? ____
A. Monocytes
B. Neutrophils
C. * Eosinophils
D. Basophils
E. None of the above

163.

Which of the following is the optimum age for development of ALL:


A. * 3-4 years old
B. 15-20 years old
C. 50 years old
D. 20-50 years old
E. 90-100 years old

164.
Which of the following medical investigation is used for patients with
vitamin B12 deficiency:
A. Tzanck test
B. Wood lamp test
C. * Schillings test
D. Elisa test
E. Wassermans test
165.

Which of the these are not myeloid cells?


A. Erythrocyte.
B. Promegakaryocyte.
C. * lymphocyte.
D. myelocyte.
E. erythroblast.

166.

Which of these can cause hemolytic anemia except:


A. PNH
B. Lead poisoning
C. Thalassemia

D. G6PD deficiency
E. * Iron deficiency
167.
____ is a hormone produced by the ____ when tissue levels of oxygen
are low.
A. Bilirubin stomach
B. Bilirubin; bone marrow
C. * Erythropoietin; kidneys
D. Erythropoietin; bone marrow
E. Hemoglobin; liver
168.

A L L occurs in which of the following:


A. Is more common in adults.
B. * is more common in Children.
C. is extremely rare in children and unusual before the age of 40.
D. has a peak age of 30-50.
E. All of the above.

169.

Absolute erythrocytosis may lead to


A. Liver insufficiency
B. Kidney insufficiency
C. Remove of stomach
D. Ionizing irradiation
E. * Cardiac insufficiency

170.

Acute leukemia is characterized by:


A. Acantolytic cells
B. Mature cells
C. * Poorly differentiated cells with many blasts.
D. Low serum ferritin
E. None of the above.

171.

Acute myelocytic leukemia includes the following except:


A. Myeloblastic.
B. Promyelocyte.
C. Monocyte.
D. Myelomonocytic.

E. * Prolymphocytic.
172.

ALL is more common in:


A. * Children
B. Adults
C. Neonates
D. Adults above 60
E. None of the above

173.

All of the following are true of erythrocytes except


A. When mature, they have no nucleus or other organelles
B. Their plasma membrane contains many antigens (molecules projecting
from the surface)
C. They normally contain practically all of the hemoglobin that is present
in blood
D. They transport oxygen from the lungs to body tissues
E. * They have a finite life span averaging about 60 days

174.

All of the following cause Microcytic Hypochromic anemia except:


A. * Lead poisoning
B. Thalassemia
C. Iron deficiency anemia
D. Fanconi's anemia
E. Nothing is correct

175.

An excessive number of white blood cells is called:


A. Lymphoma
B. Leukopenia
C. * Leukocytosis
D. All of the above
E. None of the above

176.
Anemia is hereditary enzymepathy on the base of glucose-6-phosphat
dehydrogenase insufficiency. Type of hereditance it is:
A. Autosome-recessive
B. Linked X-chromosome, recessive
C. Autosome-dominant
D. Intermediate

E. * Linked X-chromosome, dominant


177.

Anemia resulting from B12 deficiency is called :


A. * Pernicious anemia
B. Hemorragic anemia
C. Aplastic anemia
D. Sickle cell anemia
E. Nutritional anemia

178.

Anemia, thrombocytopenia is prominent feature of..


A. Chronic leukemia
B. * Acute leukemia
C. Both Acute and Chronic
D. None
E. Subacute leukemia

179.

At the deficit of Villebrands factor is disordered


A. Retraction of clot
B. Aggregation of thrombocytes
C. Formation of fibrin
D. Activation of prothrombin
E. * Adgesion of thrombocytes

180.

B cells function by promoting ____ .


A. Phagocytosis
B. * Antibody production
C. Release of histamine
D. Cell to cell killing of viral infected cells
E. The production of erythropoietin

181.
B12-deficiency anemia developed in a patient after the total resection
of stomach. What type of cells are present in blood?
A. Anulocytes
B. Microcytes
C. Ovalocytes
D. Spherocytes
E. * Megalocytes

182.

Because of local intravessels of blood coagulation arises


A. Slage-syndrome
B. Embolism
C. DIC-syndrome
D. Arterial hyperemia
E. * Thrombosis

183.
Bence Jones protein is a Paraprotein and it is characteristics of the
following:
A. * Multiple myeloma
B. Sidoroblastic anemia
C. Aplastic anemia
D. Iron deficiency
E. Leukemia
184.

Blood perform such functions:


A. Gas transport
B. Transport of nutritional substances
C. Regulative
D. Osmotic
E. * All of the above

185.

By strong inhibitors of aggregation of thrombocytes is


A. Encephalin
B. Prostaglandin
C. Estrogen
D. Globulin
E. * Prostacyclin

186.
By the genetic marker of myeloleucosis is philadelphian chromosome.
What chromosomal aberration it formed as a result of?
A. Inversion of short shoulder of 21-th chromosome
B. Deletion of short shoulder of 22-th chromosome
C. Translocation of short shoulder of 22-th chromosome on 21-th
D. Duplication of long shoulder of 22-th chromosome
E. * Translocation of long shoulder of 22-th chromosome on 9-th
187.

Choose the indexes of neutrophilic band granulocytes :

A. 19 37 %
B. 3 11 %
C. 0 1 %
D. * 1 6 %
E. 45 72 %
188.

Choose the indexes of normal basophilic granulocytes:


A. 19 37 %
B. 3 11 %
C. * 0 1 %
D. 0,5 5 %
E. 45 72 %

189.

?Choose the indexes of normal concentrations of RBC in male:


A. 3-4,1? 1012/L
B. 2,1-5,1? 1012/L
C. * 4-5,1? 1012/L
D. 12-15 ? 1012/L
E. 5-10? 1012/L

190.

Choose the indexes of normal concentrations of RBC in female:


A. * 3,7-4,7? 1012/L
B. 2,1-5,1? 1012/L
C. 4-5,1? 1012/L
D. 12-15 ? 1012/L
E. 5-10? 1012/L

191.

Choose the indexes of normal eosinophilic granulocytes:


A. 19 37 %
B. 3 11 %
C. 0 1 %
D. * 0,5 5 %
E. 45 72 %

192.

Choose the indexes of normal hematocrit in female:


A. 3,7-4,7 %
B. 2,1-5,1 %

C. * 36- 42 %
D. 12-15 %
E. 50-60 %
193.

Choose the indexes of normal hematocrit in male:


A. 3,7-4,7 %
B. 2,1-5,1 %
C. * 40- 48 %
D. 12-15 %
E. 50-60 %

194.

Choose the indexes of normal lymphocytes:


A. * 19 37 %
B. 3 11 %
C. 0 1 %
D. 0,5 5 %
E. 45 72 %

195.

Choose the indexes of normal maintenance of hemoglobin in women:


A. 140-160 g/l
B. * 120-140 g/l
C. 125-160 g/l
D. 50-80 g/l
E. 70-100 g/l

196.

Choose the indexes of normal maintenance of hemoglobin in men:


A. 80-100 g/l
B. * 130-160 g/l
C. 115-145 g/l
D. 70-90 g/l
E. 100-120 g/l

197.

Choose the indexes of normal monocytes:


A. 19 37 %
B. * 3 11 %
C. 0 1 %
D. 0,5 5 %

E. 45 72 %
198.

Choose the indexes of normal neutrophilic segmented granulocytes:


A. 19 37 %
B. 3 11 %
C. 0 1 %
D. 0,5 5 %
E. * 45 72 %

199.

Choose the indexes of normal plateletes CBC:


A. 100-220 ? 109/L
B. 100-120 ? 109/L
C. 80-120 ? 109/L
D. * 180-320 ? 109/L
E. 10-32 ? 109/L

200.

Choose the indexes of normal reticulocytes CBC:


A. 3,7-4,7 %
B. 2,1-5,1 %
C. 36- 42 %
D. * 0,5-1 %
E. 50-60 %

201.

Choose the indexes of normal total leukocytes CBC:


A. 10-20 ? 109/L
B. 1-2 ? 109/L
C. 8-12 ? 109/L
D. * 4-9 ? 109/L
E. 10-32 ? 109/L

202.

Circulating mature RBCs lack:


A. Ribosomes
B. * Mitochondria
C. Nuclei
D. All of the above
E. None of the above

203.

Classes of immunoglobulin produced by multiple myeloma:

A. IgM
B. IgD
C. * IgG
D. IgE
E. IgA
204.

Classification of leukemia:
A. * Acute and chronic
B. Mild and Grievous
C. Hemorrhagic and post Hemorrhagic
D. Microcytic and Macrocytic.
E. Hyperchromic and Hypochromic

205.

Combination of haemoglobin with oxygen named:


A. Carboxyhemoglobin
B. Methemoglobin
C. Carbylaminhemoglobin
D. Carbhemoglobin
E. * Oxyhemoglobin

206.

Combination, that transported CO2 from tissues to lung named:


A. Methemoglobin
B. * Carbhemoglobin
C. Oxyhemoglobin
D. Carbylaminhemoglobin
E. Carboxyhemoglobin

207.
Complete blood count in patients with vitamin B12 deficiency anaemia
shows all changes except of the following:
A. Decreased amount of erythrocytes and haemoglobin
B. Macrocytosis
C. Increased colour index more than 1,1
D. * Decreased colour index below 0.8
E. Zhollis bodies and Kebots rings in erythrocites
208.

Coomb's Positive Hemolytic Anaemia is seen in except:


A. * Alcoholic cirrhosis

B. Chronic active hepatitis


C. Primary biliary cirrhosis
D. Primary sclerosing cholangitis
E. Nothing is correct
209.

Cyanosis of skin develops as a result increase in the blood capillaries:


A. Carboxyhemoglobin
B. Erythrocytes
C. Methemoglobin
D. Carbhemoglobin
E. * Deoxyhemoglobin

210.
Deficit of what vitamin in the liver leads to decrease in coagulation
factors II, V, VIII?
A. B6
B. C
C. PP
D. B1
E. * K
211.

Definative diagnosis of Hodgkins Lymphoma is:


A. * Lymph node biopsy
B. General blood test
C. General urine analysis
D. X-ray of the chest
E. None of the Above

212.

Definitive diagnosis of lymphogranulomatosis is made by:


A. General blood test
B. Biochemical blood analysis
C. Plasmaphoresis
D. * Lymph node biopsy
E. General urine analysis.

213.

Destruction of erythrocytes in ecquired hemolytic anemia occurs in


A. Macrophages of liver
B. Macrophages of spleen

C. Lymphatic nodes
D. Intracellular liquid
E. * Blood vessels
214.

Detoxification function of blood is conditioned:


A. Gas transport (CO2 and O2)
B. Transport of nutritional substances
C. Exchange of heat between tissues and blood
D. * Detoxification toxic substanses by the enzymes of blood
E. Presents in blood of antibodies and by the phagocit function of
leucocytes

215.

Electrophoresis of blood proteins carry out at pH:


A. 5,5
B. 7,0
C. * 8,6
D. 4,7
E. 3,0

216.

Eosinophils are involved in which of the following?


A. Phagocytosis
B. * Parasitic infection
C. Viral infection
D. All of the above
E. None of the above

217.

Erythrocytes sedimentation rate (ESR) for males is:


A. 1 12 mm/hr
B. 1 10 mm/hr
C. * 2 10 mm/hr
D. 3 15 mm/hr
E. 2 15 mm/hr

218.

Erythropoietin:
A. * Stimulates red blood cell synthesis
B. Stimulates white blood cell synthesis
C. Is released in response to a decrease in blood flow to the bone marrow

D. A and C
E. B and C
219.
Erytropoietin synthesis is disordered in chronic kidney insufficiency.
Development of what blood elements will be decreased?
A. Neutrophils
B. Monocytes
C. Thrombocytes
D. Lymphocytes
E. * Erythrocytes
220.

Haemoglobin A of erythrocytes in the adult include:


A. 22 and 1 polypeptide chains
B. 11 and 2 polypeptide chains
C. 4 4 - polypeptide chains
D. 4 - polypeptide chains
E. * 2 and 2 - polypeptide chains

221.

Haemoglobin A of erythrocytes in the adult include:


A. 22 and 1 polypeptide chains
B. 11 and 2 polypeptide chains
C. 4 4 - polypeptide chains
D. 4 - polypeptide chains
E. * 2 and 2 - polypeptide chains

222.

Hemoglobine of erythrocytes include:


A. * Hem and globin
B. Histones and hem
C. Protamines and hem
D. Globin and NAD
E. Iron, copper and protein

223.

Hemoglobine of erythrocytes include:


A. * Hem and globin
B. Histones and hem
C. Protamines and hem
D. Globin and NAD

E. Iron, copper and protein


224.
High level of reticulocytes in peripheral blood smear is indicative of
which of the following?
A. * Post hemmorhagic anemia
B. Iron deficiency anemia
C. CLL
D. AML
E. None of the above
225.

Hodgkins lymphoma is also known as:


A. Lymphoma
B. * Lymphogranulomatosis
C. Lipoma
D. Mesothelioma
E. Ependymoma

226.
How is anaemia named, in base which lie decrease of enzymes
activition which take part in the hem synthesis?
A. Metaplastic
B. Sickle cell
C. Toxic-hemolytic
D. Irondeficiency
E. * Sideroblastic
227.

Hypoxic hypoxia causes change in the system of blood, namely:


A. Decrease of erythrocytes without the changes of Hb
B. Increase of erythrocytes without the changes of Hb
C. Increase of Hb without the changes of erythrocytes
D. Decrease of erythrocytes and Hb
E. * Increase of erythrocytes and Hb

228.

If the prominent cell line is of myeloid series, it is


A. Lymphocytic leukemia
B. * Myelocytic leukemia
C. Myelolymphocytic leukemia
D. Basophilic leukemia
E. All of the above

229.

Immune status of organism is provided by:


A. * Leukocytes
B. Trombocytes
C. Erythrocytes
D. Hemoglobin
E. Bilirubin

230.
In eight years old child in general blood test is revealed: erythrocytes
1.2*1012/1, hemoglobin 34 g/l, color index 0.9 , thrombocytes 50*109/1,
leukocytes 2.3*109/1, blasts 30%, neutrophyls 22%, lymphocytes 43%,
monocytes 5%, ESR 62 mm/hour. Characterize these changes.
A. Hemophilia
B. Thrombocytopenic purpura
C. Henoch's disease
D. Anemia
E. * Leukosis
231.
In microscopy of a peripheral blood smear, auer rods where seen.
Which of the following diseases is it seen in?
A. CML
B. * AML
C. CLL
D. ALL
E. None of the above.
232.

In Polycythemia vera, all the following are seen except:


A. * Thrombocytopenia
B. Increased GI bleed
C. Thrombosis
D. Transient visual loss
E. Nothing is correct

233.
In ten years old boy in general blood count is revealed: erythrocytes
1.2*1012/1, hemoglobin 34 g/l, color index 0.9 , thrombocytes 50*109/1,
leukocytes 12.3*109/1, blasts 45 %, neutrophyls 22%, lymphocytes 28%,
monocytes 5%, ESR 52 mm/hour. Characterize these changes.
A. Hemophilia
B. Thrombocytopenic purpura
C. Henoch's disease

D. Anemia
E. * Leukosis
234.
In the sickle cell anemia is synthesized anomalous HbS as a result of
gene mutation, in which in place of glutamic acid present
A. Alanin
B. Cystine
C. Leucin
D. Tyrosine
E. * Valin
235.
In which of the following types of white blood cells do the cytoplasmic
granules stain preferentially with red-staining dyes?
A. Neutrophils
B. Basophils
C. * Eosinophils
D. Lymphocytes
E. Monocytes
236.

Increase related to O2 in the perinatal period of child has:


A. HbS
B. * HbF
C. HbA
D. HbE
E. HbC

237.
It is known that in pathological condition erythroblastic type of blood
formation in bone marrow may change on megaloblastic. It is characterized
for:
A. Cancer of duodenum
B. Tuberculous intoxication
C. Sickle
D. Cronical blood loss
E. * B12-deficiency anemia
238.
Leukocytes are divided into two classes based on the presence or
absence of microscopically visible structures called
A. Nuclei
B. * Granules

C. C. Ribosomes
D. Mitochondria
E. Golgi complexes
239.
Leukocytes are divided into two classes based on the presence or
absence of microscopically visible structures called
A. Nuclei
B. * Granules
C. C. Ribosomes
D. Mitochondria
E. Golgi complexes
240.

Lymphogranulomatosis is characterized by growth of giant cells called:


A. Sickel cells
B. Romanovich cells
C. Hodgkins cells
D. Burr cells.
E. * Reed-Sternberg cells.

241.

Maximum ESR is seen in:


A. * Multiple myeloma
B. CHF
C. Polycythemia vera
D. Sickle cell anemia
E. Nothing is correct

242.

Megaloblastic anemia includes which of the following?


A. B12 deficiency anemia
B. Folic acid deficiency
C. Hemolytic anemia
D. * A and B
E. B and C

243.

Most of the circulating leukocytes are:


A. Basophils
B. Eosinophils
C. Leukocytes

D. Monocytes
E. * Neutrophils
244.

Multiple myeloma can be diagnosed with which of the following?


A. Serum protein electrophoresis
B. Bone marrow examination
C. Urine protein electrophoresis
D. X-ray of the the involved bones
E. * All of the above

245.

Multiple myeloma is also known as


A. * Kahlers disease
B. Hodgkins disease
C. Reed- Stenberg disease
D. Arthurs syndrome.
E. All of the above.

246.

Multiple myeloma is also known as:


A. Lymphogranulomatosis.
B. * Plasma cell myeloma
C. Myelomatosis.
D. Hodgkins disease.
E. Granulomatosis

247.

Myeloma is diagnosed with:


A. Blood tests
B. Bone marrow examination
C. X-rays of commonly involved bones
D. Urine protein electrophoresis
E. * All of the following

248.

Normal anisocytosis is is:


A. * 11-14 %
B. 24-33 %
C. 15-35 %
D. 50-100 %
E. 1-10 %

249.

Normal concentration of ferritin in blood serum is:


A. 5-10 ng/dL
B. 40-60 ng/dL
C. * 45-340 ng/dL
D. 450-640 ng/dL
E. 450-550 ng/dL

250.

Normal Iron binding capacity, total is:


A. * 30,6 - 84,6 micromole/l
B. 306 - 846 micromole/l
C. 3 - 8 micromole/l
D. 10,6 -14,6 micromole/l
E. 6 - 8 micromole/l

251.

Normal level of MCHC?


A. 17-25 %
B. 30-50 %
C. 40-57 %
D. 60-75 %
E. * 33-37 %

252.

Normal MCH is:


A. 80-90 pg
B. * 24-33 pg
C. 15-35 pg
D. 50-100 pg
E. 1-10 pg

253.

Normal MCV is:


A. Blood loss, decreased production of blood.
B. * Blood loss, excessive production, and excessive destruction of blood.
C. Impaired or decreased production of blood, blood loss.
D. Blood loss, excessive destruction of RBC, impaired or decreased
production of RBC.
E. Blood loss, increased production, and excessive destruction of blood.
F. 80-90 fl

G. 80-100 fl
H. 15-35 fl

254.

I.

50-100 fl

J.

1-10 fl
Normal RDW is:

A. * 11-14 %
B. 24-33 %
C. 15-35 %
D. 50-100 %
E. 1-10 %
255.

Normal Soluble transferrin receptor is:


A. * 1.8 4.6 mg/L
B. 306 - 846 micromole/l
C. 3 - 8 micromole/l
D. 10,6 -14,6 micromole/l
E. 6 - 8 g/l

256.
Normal value of MCHC (mean corpuscular hemoglobin concentration)
in blood:
A. 44 45 %
B. 45 47 %
C. * 33 37 %
D. 36 47 %
E. 57 60 %
257.

Normal value of platelets in blood:


A. 120-220 x 109/l
B. 150-180 x109/l
C. * 180-320 x109/l
D. 18-32 x109/l
E. None of the above

258.
Sideroblastic anemia often arises at treatment by some
antituberculousis drugs (isoniasid), because in the process of treatment
appears a deficit of vitamin
A. C

B. 12
C. A
D. K
E. * 6
259.

Smudge cells are characteristic for ?


A. CML
B. ALL
C. AML
D. * CLL
E. None of the above

260.

Splenomegaly is often prominent in..


A. * Chronic leukemia
B. Acute leukemia
C. Both Acute and Chronic
D. None
E. Subacute leukemia

261.

The basic function of erythrocytes in blood:


A. Promotes agregation of thrombocytes
B. * Transport of C2 and O2
C. Syntheses of proteins of the contraction system (actine, myosine)
D. Provide immune status of organism
E. Take part in formation of active forms of oxygen

262.

The basic function of erythrocytes in blood:


A. Promotes agregation of thrombocytes
B. * Transport of C2 and O2
C. Syntheses of proteins of the contraction system (actine, myosine)
D. Provide immune status of organism
E. Take part in formation of active forms of oxygen

263.

The coagulation test includes:


A. ESR, partial thromboplastine time, prothrombine time.
B. Platelet count, bleeding time, prothrombine time, concentration of
fibrinogen in plasma.

C. * Partial thromboplastine time, prothrombine time, concentration of


fibrinogen in plasma.
D. Hematocrit, bleeding time, clotting time, concentration of fibrinogen in
plasma.
E. Blood type, clotting time, partial thromboplastine time, prothrombine
time.
264.

The etiology of Leukemia is:


A. * Unknown
B. Viruses
C. Bacteria
D. Radiation
E. Drugs and Chemicals

265.

The following are features of acute leukemias except:


A. Onset is usually rapid.
B. Disease is very aggressive.
C. The cells involved are usually poorly differentiated.
D. * The cells involved are usually more mature cells.
E. Presence of many blast cells.

266.

The following are seen in bone marrow aspiration in CML except:


A. Myeloid hyperplasia
B. Relatively few blast cells
C. Mostly mature neutrophils
D. Increased megakaryotes
E. * Mostly immature neutrophils

267.

The lymphocyte that is responsible for cell-mediated immunity is the:


A. * T lymphocyte
B. NK cell
C. B lymphocyte
D. None of the above
E. All of the above

268.

The most numerous white blood cell in normal blood is the:


A. * Neutrophil
B. Lymphoctye

C. Monocyte
D. Eosinophil
E. B lymphocyte
269.

The normal life span of the red cell is:


A. * 120 days
B. 120 hours
C. 30 days
D. 3-5 days
E. 24 hours

270.
The oxyphylic normocytes were appeared in the blood of a patient after
acute blood loss. 25 % of reticulocytes were found with a supravital dye.
Name the type of this anemia according to the bone marrow capacity to
regeneration?
A. Aregenerative
B. Hyperegenerative
C. Hyporegenerative
D. Disregenerative
E. * Regenerative
271.

The primary function of a mature red blood cell is:


A. Defense against toxins and pathogens
B. Delivery of enzymes to target tissues
C. Transport of respiratory gases
D. * All of the above
E. None of the above

272.

The Schilling test is a medical investigation used for patients with:


A. * Vitamin B12 deficiency
B. Hepatitis
C. Leukosis
D. Thrombocytopenia
E. Non of above

273.

The ultimate source of all types of blood cells are the...


A. Thrombocytes
B. * Hemocytoblasts

C. Myeloid stem cells


D. Lymphoid stem cells
E. Granular leukocytes
274.

The white blood cell that is most like the mast cell is the:
A. * Basophil
B. Lymphocyte
C. Neutrophil
D. Eosinophil
E. Monocyte

275.
To measure or differentiate anemia of mixed causes and forms such as
anisocytosis, which of the following is most appropriate to use?
A. Red cell distribution width
B. MCHC
C. * MCV
D. Non of above
E. MCH
276.
?To the boy of 8 months that is cured because of pneumonia and
rickets moderate severity anemia was diagnosed. What indicators of
hemoglobin in the blood are characteristic for this degree of anemia?
A. * 70-89 g / l
B. 90 - 110 g / l
C. 80 - 100 g / l
D. 69 g / l and less
E. 100 - 120 g / l
277.
To the girl of 12 months that is cured because of pneumonia and rickets
mild severity anemia was diagnosed. What indicators of hemoglobin in the
blood are characteristic for this degree of anemia?
A. 70-89 g / l
B. * 90 - 110 g / l
C. 80 - 100 g / l
D. 69 g / l and less
E. 100 - 120 g / l
278.

Watermelon stomach is characteristic of the following:


A. Vitamin B12

B. Iron deficiency
C. Malaria
D. Jaundice
E. * Hodgkins lymphoma
279.

What anemia characteristic with megaloblastic type of blood forming?


A. Hypoplastic
B. Toxicohemolytic
C. Metaplastic
D. Posthemorrhagic
E. * Pernicious

280.
What are the main laboratory findings in patient with chronic lymphatic
leukemia?
A. Thrombocytosis
B. * Anemia and thrombocytopenia
C. Thrombocytosis and lymphocytosis
D. No findings,
E. Philadelphia chromosome in abnormal cells
281.
What does mean of erythrocytosis in a sick with heart congenital
defect?
A. Independent disease
B. Complication
C. Terminal state
D. Index of convalescence
E. * Compencatory reaction
282.

What function of blood belong a concept oncotic pressure of blood?


A. Gas transport
B. Protective
C. Detoxification
D. Termoregulation
E. * Osmotic

283.

What is found in multiple myeloma:


A. * Hypercalcemia
B. Increased Alkaline phosphatase

C. Decreased IgA
D. Hypouricemia
E. Nothing is correct
What is not seen in multiple myeloma?

284.

A. * Increased alkaline phosphatase


B. Anemia
C. Hypercalcemia
D. Ted ESR
E. Nothing is correct
What is not seen in polycythemia vera?

285.

A. * Increase erythropoietin level


B. Increase RBC count
C. Increased Vit B12 binding capacity
D. Ocular congestion
E. Nothing is correct
What is the normal value for total serum transferrin in blood?

286.

A. * 1.7 4.7 mg/l


B. 1.7 8.5 mg/l
C. C.1.7 10.0 mg/l
D. D.1.7 12.0 mg/l
E. E.1.7 15.0 mg/l
What organs and in what order take part in embryonic hematopoiesis?

287.

A. Blood loss, decreased production of blood.


B. Blood loss, excessive production, and excessive destruction of blood.
C. * Impaired or decreased production of blood, blood loss.
D. Blood loss, excessive destruction of blood, impaired or decreased
production of blood.
E. Blood loss, increased production, and excessive destruction of blood.
F. 110-90 g/L, 3.5-3*1012/L
G. 90-70 g/L, 3,0-2.5*1012/L
H. 70-50 g/L, 2.5-2*1012/L
I.

120-100 g/L, 3,5-4*1012/L

J.

Less than 50 g/L, less than 2.0*1012/L

K. Hyperleucocytosis or leucopenia, anemia, blasts in peripheral blood


more than 5 %.
L. Lymphocytosis, hyperleucocytosis, anemia, thrombocytopenia, blasts
in peripheral blood more than 20 %.
M. Leucocytosis, anemia, monocytosis, blasts in peripheral blood more
than 10 %.
N. Anemia, thrombocytosis, eosynophilia, leucopenia, blasts in peripheral
blood more than 5 %.
O. Hyperleucocytosis or leucopenia, anemia, thrombocytopenia, blasts in
peripheral blood, blasts in marrow aspirate more than 20 %.
P. Spleen, liver, bone marrow
Q. Yolk sac, liver, kidneys, bone marrow
R. Yolk sac, liver, spleen, bone marrow
S. Spleen, liver, blood vessels, bone marrow
T. Yolk sac, spleen, muscles, liver
288.
What quantitative measurements are necessary to establish a
diagnosis and to monitor the disease Multiple myeloma?
A. Potassium
B. Vit. B12
C. * Paraprotein
D. Iron
E. All of the Above.
289.

When oxygen is carried by the blood, it is bonded to


A. Platelets.
B. Antibodies.
C. Plasma.
D. * Hemoglobin.
E. Water

290.
Which formed elements are most directly associated with the immune
responses that defend the body against pathogens?
A. Erythrocytes
B. * Leucocytes
C. Platelets

D. None of the above (the immune response is strictly a function of


plasma)
E. All of the above
291.
Which formed elements are most directly associated with the immune
responses that defend the body against pathogens?
A. Erythrocytes
B. * Leucocytes
C. Platelets
D. None of the above (the immune response is strictly a function of
plasma)
E. All of the above
292.

Which indexes are responsible for the normal volume of blood in adult?
A. 15 L
B. 25 L
C. * 5 L
D. 50 L
E. 2 L

293.

Which is the most common type of blood cell in a healthy human?


A. * Erythrocytes
B. Monocytes
C. Lymphocytes
D. Eosinophils
E. Basophils

294.

Which of the following blood cells has phagocytic function?


A. * Monocyte
B. RBC
C. Basophils
D. Lymphocytes
E. Platelets

295.
Which of the following cells play a crucial role in the pathogenesis of
alveolar-capillary damage in adult respiratory distress syndrome (ARDS)?
A. CD4-positive lymphocytes
B. CD8-positive lymphocytes
C. Eosinophils

D. Mast cells
E. * Neutrophils
296.

Which of the following is a type of leukocyte?


A. Macrophage
B. Eosinophil
C. Monocyte
D. * All of the above
E. None of the above

297.

Which of the following is NOT a type of granular white blood cell? ____
A. Monocytes
B. Neutrophils
C. * Eosinophils
D. Basophils
E. None of the above

298.

Which of the following is the optimum age for development of ALL:


A. * 3-4 years old
B. 15-20 years old
C. 50 years old
D. 20-50 years old
E. 90-100 years old

299.
Which of the following medical investigation is used for patients with
vitamin B12 deficiency:
A. Tzanck test
B. Wood lamp test
C. * Schillings test
D. Elisa test
E. Wassermans test
300.

Which of the these are not myeloid cells?


A. Erythrocyte.
B. Promegakaryocyte.
C. * lymphocyte.
D. myelocyte.

E. erythroblast.
301.

Which of these can cause hemolytic anemia except:


A. PNH
B. Lead poisoning
C. Thalassemia
D. G6PD deficiency
E. * Iron deficiency

302.
What part of tooth (in 100g of tissues) containes: 30 40 g of water, 40
g of organic compounds 20 -30 g of inorganic compounds, Ca 30 g, 17
g?
A. Dentine
B. * Pulp
C. Enamel
D. Cementum
E. None of the above
303.

1,25-dihydroxycholecalciferol promotes absorption of:


A. P
B. Cu
C. Zn
D. Na
E. * Ca

304.
A carbonate apatite is soluble in an acidic environment. What diet is
most promotes formation of organic acids?
A. Protein
B. * Carbohydrate
C. Lipid
D. Vitamin
E. Diet isnt important
305.
A collagen is synthesized from procollagen ripening of which includes
a few stages, except:
A. * Partial hydrolysis of polypeptide chains
B. Posttranslative modification with participation of hydroxylases and
glycosyltransferases
C. Transmembrane transfer into intracellular space from fibroblasts

D. Extacellular modification with formation of transversal connections


E. Formation of fibrils, fibres
306.
A fibronectin is an important representative of teeth and bones
glycoproteins which located on-the-cells membranes. It executes a such
function as:
A. Fermentativ
B. Transport
C. Reseptor
D. * Adheziv
E. Energetic
307.
A highly complex mixture of water and organic and non-organic
components created by salivary glands is :
A. Mucous
B. Sweat
C. Sputum
D. Urine
E. * Saliva
308.
A transport environment of enamel for different matters is an enamel
liquid of microspaces of hydroxyapatite, where easily such components may
be transported from saliva:
A. Ions of Ca, , F
B. Glucose, citrate, vitamins
C. Amylase
D. Cholesterol
E. * Both A and B
309.

Acid-Base balance is important for:


A. Normal enzyme functions.
B. Normal metabolite solubility.
C. Normal membrane potentials.
D. A and C
E. * All of the above

310.

Action of parathormone is tightly connected with vitamin:


A.
B. A

C. * D
D.
E. K
311.

Acyclovir is main pharmacological treatment for:


A. * Herpes Simplex Virus
B. Chlamydia
C. Yersinia
D. Streptococcus
E. AIDS

312.
After strenuous muscle activity, lactate produced from anaerobic
glycolysis is recycled in the liver by being ____________________ in a reaction
catalyzed by lactate dehydrogenase.
A. oxidized by NADH to form oxaloacetate
B. reduced by NADH to form oxaloacetate
C. * oxidized by NAD+ to form pyruvate
D. reduced by NAD+ to form pyruvate
E. none of above
313.

Alkaline phosphatase of tooth executes the followings functions:


A. Transfers a phosphate of anions from phosphoric ethers of glucose to
the organic matrix of tooth
B. Takes part in formation of crystallization nuclei and teeth
mineralization
C. Takes part in breaking up of glycogen
D. Demineralizing
E. * Both A and B correct

314.

Amino acid remnants wich the most often are met in collagen are:
A. tryptophane, oxilysine, cysteine, valine
B. tryptophane, cysteine, glycine, methionine
C. lysine, arginine, cysteine, tryptophane
D. * oxiproline, oxilysine, glycine, proline
E. aminosuccinamic, glutamine, lysine

315.
Anemia, hemorrhage, and chronic obstructive pulmonary disease can
all cause metabolic acidosis. The best explanation is that the lack of oxygen
causes

A. a decrease in insulin that, in turn, increases anaerobic glycolysis in the


brain
B. * a decrease in oxidative phosphorylation so the cells have to rely upon
anaerobic glycolysis
C. a decrease in the oxidation of tyrosine to epinephrine which decreases
gluconeogenesis in muscle
D. a decrease in the removal of CO2 from the blood. The resulting
decrease in pH causes an increase in glycolysis in most cells
E. an increase in glycolysis in red blood cells
316.

Antibacterial compounds of saliva are all, except :


A. Mucine
B. Thiocyanate
C. Hydrogen peroxide
D. * Estrogen
E. Immunoglobulin A

317.

Antibacterial compounds of saliva are all, except :


A. Mucine
B. Thiocyanate
C. Hydrogen peroxide
D. Immunoglobulin A
E. * Immunoglobulin C

318.

Antibacterial compounds of saliva are all, except :


A. Mucine
B. Thiocyanate
C. * Thyroxine
D. Hydrogen peroxide
E. Immunoglobulin A

319.
At a hyperparathyroidism (Recklinghausen disease) mineralization of
the bony system and tooth tissues is violated as a result of:
A. * Hypercalciemia, phosphateuria, osteoporosis
B. Violation of Ca absorption in an intestine
C. Violation of hydroxylation of calciferol
D. Violation of posttranslational modification of collagen

E. At this disease there are no any changes of the bony system and teeth
tissues
320.
At wound's healing scars appear. What matter is the main component
of scar's connective tissue?
A. keratan sulfate
B. elastin
C. hyaluronic acid
D. chondroitin sulfate
E. * collagen
321.
Beginning of caries is characterized by enamel demineralization as
result of such changes as:
A. Maintenance of Ca, , F decrease in the area of caries,
B. A form, size and configuration of crystals of hydroxyapatite are
changed
C. A soluble proteins are washed in the area of caries from an enamel and
dentine
D. Activity of alkaline Phosphatase decreases
E. * All of the above
322.

Below named are the main clinical symptoms of scurvy, except:


A. Gums bleeding
B. Pain in heart
C. Tachycardia
D. * Xeroftalmia
E. Petechias

323.

Chemical nature of parathormone:


A. * Protein
B. Steroid
C. Derivate of amino acid
D. Lipid
E. Derivate of arachidonic acid

324.

Choose correct answer according to concept of vitamins:


A. Are inorganic substances that function in living cells in trace amounts
and are vital for many forms of life
B. * Are organic substances that function in living cells in trace amounts
and are vital for many forms of life

C. Are organic substances that function only in liver in trace amounts


D. Are inorganic substances that must acquire from endogenous sources
E. All answers correct
325.
Choose the active form of vitamin D, which has characteristics of
hormone?
A. 7-dehydrocholesterol
B. * 1,25-dihydrocholecalciferol
C. . 25- hydroxycholecalciferol
D. Ergocalciferol
E. . Cholecalciferol
326.

Chose the effects that are caused by parathormone?


A. Hypocalciemia, hypophosphatemia, hyperphosphaturia
B. * Hypercalciemia, hypophosphatemia, hyperphosphaturia
C. Hypocalciemia, hyperphosphatemia, hyperphosphaturia
D. Hypercalciemia, hypophosphatemia, hypophosphaturia
E. Hypercalciemia, hyperphosphatemia, hypophosphaturia

327.

Clinical sign of the candidiasis:


A. * White cottage cheese-like plaques
B. Grey plaque
C. Grey cheese-like plaque
D. Yellow plague
E. Pink plague

328.

Common causes of Xerostomia are all listed, except:


A. Medications
B. Dehydration
C. Infections of the mouth
D. * Eating too much sweets
E. Surgery

329.

Common causes of Xerostomia are all listed, except:


A. Medications
B. Dehydration
C. Infections of the mouth

D. Surgery
E. * Problems with heart
330.

Daily production of saliva is :


A. 1000 ml
B. * 1500 ml
C. 2000 ml
D. 500 ml
E. 1700 ml

331.
Day's requirement of adults in F (with a meal and water) is 2,7-5 mg. It
excess causes a risk of fluorosis. The toxity of F increases such factor as:
A. Lack of vitamin of
B. Excess of carbohydrates in a meal
C. * Lack of Ca
D. Increase of amylase activity
E. All of the above
332.

Decreased production of saliva is named:


A. Xerostomia
B. Diasthema
C. Glossitis
D. * Hyposalivation
E. Hypersalivation

333.
Demineralization (resorption) of bony tissues takes place at all below
mentioned states, except:
A. Hypervitaminosis D
B. Hyperparathyroidism
C. Icenko-Kushing disease
D. Hypervitaminosis A
E. * Acromegalia
334.
Demineralization is a process of dissolving of tooth tissues. The
mechanisms of this process are all of the following, except:
A. * An increase of activity of alkaline phosphatase in saliva.
B. Increase of activity ? glycosidases and acidic phosphatase in saliva.
C. Splitting of connections of inorganic ions with the molecules of organic
substance.

D. Disorders of collagen synthesis.


E. A decrease of Ca in saliva, increase of acidic products in it.
335.

Density of saliva is:


A. 15-20
B. * 18-35
C. 20-43
D. 17-22
E. 13-65

336.

Description of caries consists of all the following, except:


A. Demineralization and destruction of hard tissues of teeth
B. This pathological process is irreversible
C. * A process is reversible at the optimum terms of remineralization
D. The damage of enamel is irreversible
E. The damage of dentine is irreversible

337.

Diabetic ketoacidosis is an example of which imbalance?


A. Respiratory acidosis
B. Respiratory alkalosis
C. Metabolic alkalosis
D. * Metabolic acidosis
E. Respiratory chain

338.
Disorders of tooth tissues calcification is a consequence of calcium
homoeostasis violation, caused:
A. By the deficiency of vitamin D
B. By a hyperparathyroidism
C. By a hypoparathyroidism
D. By the deficiency of calcitonin
E. * By the all of the above factors
339.
Dissolving of teeth surface because of damaging action of some factors
is named:
A. Diastheme
B. Threme
C. * Erosion
D. Absorption

E. Extraction
340.
During muscle contraction, calcium is released from the endoplasmic
reticulum. An increase in glycogenolysis is initiated when calcium binds to
A. * Calmodulin
B. Troponin I
C. Protein kinase A
D. Zymogen
E. Protein phosphatase
341.

Dysgeusia is:
A. Changes in tooth structure
B. Changes in the structure of mucous
C. Changes in saliva compounds
D. * Changes in taste
E. Changes in salivary glands

342.

Dysphagia is:
A. Difficulty in speaking
B. Difficulty in moving of the jaws
C. * Difficulty in swallowing
D. Difficulty in touching
E. Difficulty in biting

343.
Excess of F in an organism, particular in children, causes a fluorosis
and such pathology of teeth as:
A. Hypoplasia
B. * Hyperplasia
C. Loose of teeth
D. Caries
E. Excess of F is not dangerous for teeth
344.

For Drooling are liable people with all the listed diseases, except:
A. Parkinsons
B. Cerebral vascular accidents
C. Dementia
D. Amyotropic lateral sclerosis
E. * Xerostomia

345.

For Drooling are liable people with all the listed diseases, except:
A. Parkinsons
B. Cerebral vascular accidents
C. * Hyposalivation
D. Amyotropic lateral sclerosis
E. Dementia

346.

For Drooling are liable people with all the listed diseases, except:
A. Parkinsons
B. Cerebral vascular accidents
C. Dementia
D. Amyotropic lateral sclerosis
E. * Dry mouth syndrome

347.
Formation of collagen in the organism needs hydroxylating of proline
which takes place with participation of proline hydroxalase. Which vitamin
activates this process?
A. NAD
B. Biotin
C. * Ascorbic acid
D. FAD
E. Pyridoxine phosphate
348.

Glycogen of organic matrix of teeth executes the followings functions:


A. It is an energy source for the processes of mineralization
B. It is a source of phosphoric ethers of glucose substrates of alkaline
phosphatase
C. It is the source of glucose phosphates substrates of anaerobic
glycolysis
D. It is the source of monosaccharides and their derivatives for the
synthesis of GAG
E. * All of the above

349.
How many ascorbic acid in normal condition is excreted per day with
urine:
A. * 20-30 mg
B. 113,5-170,5 mg
C. 10-28 g

D. 80-100 mg
E. 18-33 g
350.
Hyaluronidase splits hyaluronic acid, intercellular permeability rises as
a result. Which vitamin does inhibit activity of hyaluronidase, assist in
strengthening of walls of vessels?
A. * Vitamin P
B. Vitamin A
C. Vitamin B1
D. Vitamin B2
E. Vitamin D
351.
Hypercalciemia, hyperphosphatemia and decreased excretion of
phosphorus with urine were observed. It can be caused by:
A. Increased production of the parathormone
B. Decreased production of the parathormone
C. * Decreased production of calcitonin
D. Increased production of calcitonin
E. Increased production of glucocorticoids
352.
Hypercalciemia, hypophosphatemia and the intensive excretion of
phosphorus with urine were found. It can be caused by:
A. * Increased production of parathormone
B. Decreased production of parathormone
C. Decreased production of calcitonin
D. Increased production of calcitonin
E. Increased production of glucocorticoids
353.
Hypocalciemia, hyperphosphatemia and decreased excretion of
phosphorus with urine were found. It can be caused by:
A. Increased production of parathormone
B. * Decreased production of parathormone
C. Decreased production of calcitonin
D. Increased production of calcitonin
E. Increased production of glucocorticoids
354.
Hypocalciemia, hypophosphatemia and the intensive excretion of
phosphorus with urine were observed in a patient. It can be caused by:
A. Increased production of the parathormone

B. Decreased production of the parathormone


C. Decreased production of calcitonin
D. * Increased production of calcitonin
E. Increased production of glucocorticoids
355.
Hypoplasia is an underdevelopment of dental tissues. It appears in
children as a result of:
A. Insufficient secretion of osteotropic factor parotin
B. Diminishing of ratio Ca/P in saliva
C. Rickets
D. * All of the above
E. B and C only
356.
In a cells of pulp - dontoblasts and fibroblasts compounds which form
a crystalline grate for mineralization formed. Choose these compounds:
A. Fibrils of collagen
B. GAG
C. Glycoproteins, GAG
D. * A, B, are correct
E. All of the above
357.
In the organism of 7 years old girl was discovered lack of copper. The
possibility of break of flexible fibres, in which desmosine and isodesmosine
are absent, grows. This is caused by the decrease of activity of
coppercontaining enzyme:
A. * lysyl oxidase
B. proline hydrooxilase
C. cytochrome oxidase
D. elastase
E. peptidase
358.
In the patient's blood plasma is discovered the increase of
hyaluronidase activity. What biochemical figure of blood plasma
determination will allow to prove the connective tissue pathology?
A. * sialic acid
B. bilirubin
C. uric acid
D. glucose
E. galactose

359.
In the process of organism's ageing is decreased bonding of water by
connective tissue. This is tied with the decrease of concentration of:
A. * glycosaminoglycans
B. collagen
C. phospholipids
D. hyaluronic acid
E. chondroitin sulphuric acid
360.
In the synthesis of 1,25-dihydroxycholecalciferol from 7dehydroxycholesterol
A. The steroid ring structure remains intact
B. Cholesterol is an intermediate
C. * Ultraviolet light is required
D. Three hydroxylation occur
E. Calcitonine is required
361.
Increase of strontium maintenance in food products or effect of radioactive strontium is dangerous for an organism. A multiple increase in a diet of
such component as _____ can prevent including of strontium on 50% :
A. Fluorine
B. * Calcium
C. Sodium
D. Phosphorus
E. Iodine
362.

Increased production of saliva is named:


A. Xerostomia
B. Diasthema
C. Glossitis
D. Hyposalivation
E. * Hypersalivation

363.
Insufficiency of ascorbic acid results in development of scurvy.
Synthesis of which protein is diminished:
A. * Collagen
B. Protrombin
C. Fibrinogen
D. Albumin

E. Ceruloplasmin
364.

Intensive metabolic processes take plase in the following tissue:


A. * Dentine
B. Enamel
C. Cement
D. A and C
E. All of the above

365.
It is known that the excessive entering of fluorine in an organism
causes development of fluorosis spots of enamel. It develops in a 100% of
population at a hit in an organism of F in an amount which exceeds:
A. * 5 mg
B. 0,5 mg
C. 100 mg
D. 1 mg
E. 1g
366.
It was established disorders of hydroxylation of proline and lysine in
composition of a collagen of a patient with scurvy. Inhibition of which
biochemical process does result in this disorders?
A. * Microsomal oxidation
B. Peroxidation of lipids
C. Tissue respiration
D. Peroxidase oxidation
E. Phosphorylation
367.

Lack of saliva can lead to all of these, except:


A. Dental decay
B. Oral yeast infections
C. Taste problems
D. * Social problems
E. Bad breath

368.

Lack of saliva can lead to all of these, except:


A. Dental decay
B. Oral yeast infections
C. Taste problems
D. Bad breath

E. * Masticatory disorders
369.

Lack of saliva can lead to all of these, except:


A. * Skin disorders
B. Oral yeast infections
C. Taste problems
D. Dental decay
E. Bad breath

370.

Lack of vitamin K causes:


A. Diarrhea
B. Kakheksiya
C. Growth retardation
D. * Hemorrhages
E. Steatoreya

371.

Lipid soluble vitamins are:


A. Coenzymes
B. * Modulators of cell membranes
C. Regulators of secretion of digestive juices
D. Transport of substances
E. . Mediators of nervous system

372.
Lipid soluble vitamins execute numerous functions in an organism
except:
A. * They are components of enzymes
B. Modulators of cell membranes
C. Antioxidants
D. They have provitamins
E. . Cause hypervitaminosis
373.

Main function of coenzyme A is:


A. A carrier of methyl group
B. Removal of hydrogen atom from specific substrate
C. Catalyze reaction of carbohydrate metabolism
D. * A carrier of acyl groups
E. Participation in respiratory chain

374.

Major proteins of connective tissue are:


A. fibroin and keratin
B. albumins and globulins
C. * collagen and elastin
D. myosin and actin
E. protamines and histons

375.
Mineralization of tooth is promoted by glycoproteins and
proteoglycans, which are synthesized at participation of vitamin A by:
A. * Formation of covalent bonds of carbohydrate fragments with residues
of serine and threonine of proteins
B. Formation of hydrogen bonds between protein and carbohydrate
components
C. Formation of ionic bonds between protein and carbohydrate
components
D. Apoprotein and carbohydrate components form micelles
E. All of the above
376.
Mineralization of tooth tissues - is a formation of organic matrix and it
saturation by mineral elements. This proces includes the row of
transformations, except:
A. Synthesis of pulps cells and releasing into intercellular space of
collagen, GAG
B. Forming of organic matrix of mineralization
C. Ionization of GAG
D. * Splitting of intermolecular connections in the molecules of organic
substance
E. Insertion of apatites into a matrix
377.

Name biologicaly active form of vitamin D:


A. * Ergocalciferol
B. Cholecalciferol
C. 7-dehydrocholesterol
D. 25-hydroxycholecalciferol
E. 1,25-dihydroxycholecalciferol

378.

Name provitamins of vitamin A:


A. Flavonoids
B. Cholecalciferol

C. * Carotenoids
D. Hesperedin
E. Dehydroascorbic acid
379.
One of main factors of caries development is decrease of in an oral
cavity due to the presence of organic acids at the excessive use of
carbohydrates. Such situation is caused by activation of:
A. Gluconeogenesis
B. Hydrolysis of starch
C. Hydrolysis of glycogen
D. * Fermentation
E. Breaking up of GAG
380.

Other name of Dry Mouth disorder is :


A. * Xerostomia
B. Hyperglossia
C. Glossitis
D. Hypersalivation
E. Bruxisms

381.
D:

Parathyroid hormone stimulates the formation of active form of vitamin


A. In liver
B. In blood
C. In brain
D. In intestine
E. * In kidneys

382.
Permeability of enamel is increased by all of the following factors
except:
A. Calcitonin
B. Calcitriol
C. * Parathormone
D. Hyaluronidase, acidic environment
E. Ions of Fe, carotin
383.

Plastic, trophic, sensory - are the functions of the following tooth tissue:
A. Enamel
B. * Pulp

C. Cementum
D. Dentine
E. Periodontal ligaments
384.

Presence of such component in food promotes risk of dental caries:


A. Calcium
B. Magnesium
C. Chlorine
D. * Strontium
E. Iodine

385.

Product of parotid salivary gland is:


A. * Serous
B. Mucous
C. Serous and mucous
D. None of above
E. All of above

386.

Product of sublingual salivary gland is:


A. Serous
B. * Mucous
C. Serous and mucous
D. None of above
E. All of above

387.

Product of submandibular salivary gland is:


A. Serous
B. Mucous
C. * Serous and mucous
D. None of above
E. All of above

388.
Proline and lysine was founded in composition of collagen fibres of a
patient with the frequent bleeding of internal organs and mucous
membranes. Lack of which vitamin caused disorders of hydroxylation?
A. Vitamin E
B. Vitamin K
C. Vitamin A

D. Thiamin
E. * Vitamin C
389.
Proper tooth tissues mineralization, prevention of defects, caries are
provided by vitamins:
A. D and B1
B. K and B5
C. and F
D. * D and
E. A and
390.
Pulp is a tooth tissue with high content of such soluble proteins,
enzymes, except of:
A. Enzymes of glycolisis, TCA
B. Enzymes of PPP, respiratory chain
C. Enzymes of proteins and nucleic acids biosynthesis
D. * Enzymes of fatty acids biosynthesis
E. Alkaline and acidic phosphatase
391.

Saliva consist of all of these, except:


A. Electrolytes
B. Mucous
C. Antibacterial compounds
D. Various enzymes
E. * Hemoglobine

392.

Saliva consist of all of these, except:


A. Electrolytes
B. Mucous
C. Antibacterial compounds
D. Various enzymes
E. * Bile

393.

Saliva consist of all of these, except:


A. Electrolytes
B. * Synovial fluid
C. Antibacterial compounds
D. Various enzymes

E. Mucous
394.

Saliva consist of all of these, except:


A. * Blood
B. Mucous
C. Antibacterial compounds
D. Various enzymes
E. Electrolytes

395.

Saliva consist of all of these, except:


A. Electrolytes
B. Mucous
C. * Sweat
D. Various enzymes
E. Antibacterial compounds

396.
Salivary glands which are lying in tile submucosa of hard and soft
palates are:
A. * Palatine
B. Buccal
C. Labial
D. Submandibular
E. Sublingual
397.

Salivary glands which are lying on the inner surface of the lips are:
A. Sublingual
B. Submandibular
C. * Labial
D. Buccal
E. Parotid

398.

Stensen duct is a duct from :


A. Labial salivary gland
B. Buccal salivary gland
C. Submandibular salivary gland
D. * Parotid salivary gland
E. Sublingual salivary gland

399.
Synthesis of protein of glycoproteins and proteoglycans, which take
part at tooth mineralization stimulates:
A. Vitamin of D
B. Somatotropin
C. * Vitamin A
D. Vitamin
E. Glucocortikoids
400.
The active form of vitamin D (1,25-dihydrocholecalciferol) maintain in
an organism a constant level of:
A. Potassium and phosphorus
B. * Calcium and phosphorus
C. S. Iron and calcium
D. Iron and magnesium
E. . Magnesium and manganese
401.
The affection of collagen fibers structure by the lack of vitamin C in the
organism is caused by enzymes activity malfunction:
A. glycosyltransferase
B. lysine hydroxylase and lysine oxidase
C. * lysine hydroxylase and proline hydroxylase
D. procolagenpeptidase
E. collagenase
402.

The amount of bicarbonates in saliva is:


A. 2-30 mmol/l
B. 2-4.5 mmol/l
C. 13-15 mmol/l
D. 1-5 mmol/l
E. * 2-13 mmol/l

403.

The amount of calcium in saliva is:


A. 1-2 mmol/l
B. 0.5-3.0 mmol/l
C. 2 mmol/l
D. * 1.2-2.8 mmol/l
E. 3.5-3.8 mmol/l

404.

The amount of chloride in saliva is:


A. 2-30 mmol/l
B. 2-21 mmol/l
C. * 5-40 mmol/l
D. 15-20 mmol/l
E. 24-60 mmol/l

405.

The amount of fluid translocated each day through salivary glands is:
A. 500 ml
B. 950 ml
C. * 750 ml
D. 380 ml
E. 280 ml

406.

The amount of magnesium in saliva is:


A. * 0.08-0.5 mmol/l
B. 0.5- 1 mmol/l
C. 1.5-1.8 mmol/l
D. 1-2 mmol/l
E. 1.2-3.4 mmol/l

407.

The amount of phosphates in saliva is:


A. * 1.4-39 mmol/l
B. 2-21 mmol/l
C. 0.4-1.4 mmol/l
D. 0.08-1 mmol/l
E. 4-6 mmol/l

408.

The amount of potassium in saliva is:


A. 15-39 mmol/l
B. 2-21 mmol/l
C. * 10-36 mmol/l
D. 15-16 mmol/l
E. 5- 15 mmol/l

409.

The amount of saliva produced by healthy person per day is:


A. 2 L

B. 3 L
C. * 1,5 L
D. 1,2 L
E. 5 L
410.

The amount of saliva secreted by minor mucous glands is :


A. * 8 %
B. 10 %
C. 15 %
D. 6 %
E. 14 %

411.

The amount of saliva secreted by parotid gland is :


A. 43 %
B. 48 %
C. 66 %
D. * 23 %
E. 25 %

412.

The amount of saliva secreted by sublingual gland is :


A. 5 %
B. 10 %
C. 7 %
D. * 4 %
E. 15 %

413.

The amount of saliva secreted by submandibular gland is :


A. * 65 %
B. 60 %
C. 75 %
D. 55 %
E. 70 %

414.
The amount of sialic acids in patient's blood is 1060 mg/l. From the
probable diagnosis should be removed the following:
A. * atherosclerosis
B. tuberculosis

C. malignant growth
D. rheumatitis
E. cardiac infarction
415.

The amount of sodium in saliva is:


A. 2-30 mmol/l
B. * 2-21 mmol/l
C. 23-25 mmol/l
D. 15-20 mmol/l
E. 24-60 mmol/l

416.

The antivitamin of vitamin K is:


A. Vikasol
B. Cholecalciferol
C. * Dikumarol
D. Thiamin
E. . Cholin

417.

The bolus is :
A. * The ball of food
B. Parotid gland
C. Ulcer of oral cavity
D. Inflammation of the tongue
E. Part of digestive system

418.
The carbohydrate components of organic matrix of tooth are the
following compounds, except:
A. Glucose, mannose
B. Galactose, maltose
C. * Cellulose, dextrane
D. Glucuronic acid, sialic acids
E. Chondroitin-4 and 6-sulphates, glycogen
419.

The colour of saliva is:


A. White
B. Grey
C. * Colourless

D. Yellow
E. Pink
420.
The damage of the immune system reduces resisting of human
organism to the infections. Immune status of organism depends on the
followings structures, except:
A. Bony marrow
B. Thymus
C. Tissue macrophages
D. Lymphatic nodes
E. * Hypophysis
421.

The day requirement of vitamin C is:


A. 50-70 g.
B. * 50-70 mg.
C. 50-70 g.
D. 500-700 mg.
E. 5-7 g

422.
The duct of this salivary gland opens into the floor of the mouth on
either side of tongue:
A. Sublingual
B. Parotid
C. Lingual
D. Buccal
E. * Submandibular
423.
The duct of this salivary gland opens into the mouth opposite side of
second molar on both sides:
A. Sublingual
B. * Parotid
C. Lingual
D. Buccal
E. Submandibular
424.

The duct, which opens opposite to second upper molar is :


A. * Stensen duct
B. Selling duct
C. Parotid duct

D. Steinberg duct
E. Romanovskyy duct
425.

The features of acidic phosphatase of teeth are the following:


A. It takes part in formation of crystallization nuclei and teeth
mineralization
B. It splits glucose phosphate with energy releasion
C. * Demineralizing
D. There are no any differences of alkaline and acidic phosphatase
functions
E. It stimulates the phosphorolysis of glycogen

426.
The following acute phase protein increase dramatically in
concentration during inflammation :
A. ceruloplasmin
B. haptoglobulin
C. C-reactive protein
D. fibrinogen
E. C3
427.
The greatest concentration of antibodies is found in the fraction of the
serum.
A. * Gamma globulin
B. Albumin
C. Beta globulin
D. Alpha globulin
E. Prealbumin
428.
The labial minor salivary gland is the most frequently sampled site in
evaluation of :
A. * Sjogrens syndrome
B. Downs syndrome
C. Arthurs syndrome
D. Xerostomia
E. Xerophthalmia
429.
The lack of vitamin D in the organism of children causes a rickets. All
below mentioned are reasons of this disease except:
A. Disorders of Ca and metabolism

B. Disorders of mineralisation
C. * Lower activity of alkaline phosphatase
D. Lack of vitamin of D in diet
E. . Lack of ultraviolet
430.

The main function of salivary amylase (ptyalin) is :


A. Starts the process of breaking down the carbohydrates
B. Starts the process of breaking down the cellulose
C. * Starts the process of breaking down the complex of starchy sugars
D. Starts the process of breaking down the fats
E. Starts the process of breaking down the proteins

431.

The main insoluble protein of tooth tissues is:


A. Fibrinogen
B. Fibronectin
C. Collagen
D. Elastin
E. Convertin

432.
The most important marker of transition of demineralization as a
physiology process into pathological one is a decline of the index Ca/P in
enamel below than:
A. 1,6
B. * 1,3
C. 1,5
D. 0,9
E. 1,9
433.

The organic components of tooth are the following compounds, except:


A. Albumins, globulins
B. Glycoproteins, proteoglycans
C. * Triacylglycerols
D. Phosphoproteins
E. Alkaline and acidic phosphatase

434.
The patient has vessel penetration malfunction. Name the connective
tissue protein, which synthesis is affected:
A. tropomyosin

B. myoglobine
C. albumin
D. * collagen
E. ceruloplasmin
435.

The pH of the saliva is:


A. 6
B. * 6,4
C. 6,64
D. 7,2
E. 7,64

436.
The remnants of desmosine and isodesmosine in elastin are formed
from aminoacid radicals of the following amino acids:
A. * lysine
B. glycine
C. proline
D. ornithine
E. hydrooxilysine
437.

The salivary gland that is situated just under the back of the tongue is:
A. Parotid
B. Buccal
C. * Sublingual
D. Submandibular
E. Lingual

438.
The second stage of mineralization is inseretion of apatites on an
organic matrix, in crystallization points. The ions of Ca and contact with the
followings components, except:
A. OH groups of serine, threonine, tyrosine
B. OH groups of hydroxy-proline, oxylysine
C. * CH3 groups of alanine and valine
D. COOH groups of glutamate, aspartate
E. With osteocalcin
439.

The secretion of saliva is controlled by :


A. * Sympathetic and parasympathetic nervous system

B. Sympathetic
C. Parasympathetic
D. Sensory division of CNS
E. Peripheral nervous system
440.
The such organic components of tooth, as soluble proteins, execute the
following functions, except:
A. Catalytic
B. Protective
C. Transport
D. Mineralizing
E. * Immune
441.
The toxity of F at it excessive entering in an organism may be
decreased by:
A. Sufficient maintenance of vitamin of
B. Normal salivation
C. Normal activity of amylase
D. * Excess of Ca
E. Sufficient maintenance of vitamin D
442.
The toxity of fluorine develops at it hit in an organism in an amount
which exceeds 5 mg due to it:
A. Changes of oral cavity
B. * Forms CaF2 which leaves tooth tissues
C. Reduces salivation
D. Promotes fermentation in an oral cavity
E. Prevents the synthesis of -bounding proteins of tooth
443.
The typical symptoms of hyperfunction of adrenal cortex are
osteoporosis and negative balance of calcium and phosphates. Disorders of
synthesis and disintegration of which substance is a result of these
symptoms:
A. * Collagen
B. Parathormone
C. Calcitonine
D. Corticotropine
E. Somatotropine

444.
This hormone promotes the transition of calcium from bones to blood,
inhibits reabsorption of phosphorus in kidneys and stimulates absorption of
calcium in intestine. Which hormone is it?
A. Calcitonin
B. Thyroxine
C. Triiodthyronine
D. * Parathormone
E. Thyrotropin
445.
This hormone stimulates the formation of 1,25dihydroxycholecalciferol in kidneys. What hormone is it?
A. Calcitonin
B. Thyroxine
C. Triiodthyronine
D. * Parathormone
E. Thyrotropin
446.
This salivary gland that is situated on either side of the head in front of
ears:
A. Sublingual
B. Parotid
C. Buccal
D. Lingual
E. Submandibular
447.

?Three major salivary glands are :


A. Sublingual, parotid, submental
B. * Sublingual, parotid, submandibular
C. Submandibular, buccal, labial
D. Buccal, submandibular, parotid
E. Parotid, labial, palatal

448.

Three major salivary glands are :


A. Single
B. Unpaired
C. Bifurcated
D. * Paired
E. Simple

449.
Tooth mineralization includes 2 stages- formation of organic matrix and
calcification. In the first stage the most important role belongs to:
A. Enamel
B. Dentine
C. Cement
D. * Pulp
E. Periodontal tissues
450.

Tropocolagene fibres in collagen fiber are interconnected by:


A. * aldol bonds
B. hydrophobic bonds
C. disulfide bonds
D. hydrogen bonds
E. peptide bonds

451.
Viscoelastic features of major matter of connective tissue are
conditioned mainly by:
A. * glycosaminoglycans
B. elastin
C. ATP
D. collagen
E. keratin
452.
Vitamin A play a general role in the folowing biochemical processes,
except:
A. * The transport of Ca2+ across certain membranes
B. Processes of growth and cell differentiation
C. Inhibits the oxidation of hemoglobin
D. Processes of glycoproteins formation
E. Reduces the disulfide bonds to sulfhydril bonds
453.
Vitamin A provides the synthesis of proteoglycans, formation of PAPS
as source for the synthesis of chondroitinsulfates, that why hypovitaminosis A
has all below mentioned consequences, except:
A. Decrease of activity of odontoblasts and fibroblasts
B. Disorders of calcification of enamel and dentine
C. * Decrease of salivation
D. Problems with teeth eruption

E. Dryness of mucous membranes of oral cavity, erosions


454.
Vitamin of is a powerful and main antioxidant. What from the below
mentioned function does not confirm it?
A. Participating in the transport of electrons and protons in a respiratory
chain
B. Stimulation of ubikhinon synthesis
C. Serves as a trap for free radicals
D. * Stimulates mobilization of Ca from bones
E. Prevents peroxide oxidation of unsaturated fatty acids in composition
of cells membranes
455.
Vitamins at their simultaneous usage can strengthen action of each
other. Which of following vitamins assist in anti-hyaluronidase activity of
vitamin P?
A. Vitamin B2
B. Vitamin D
C. Vitamin B1
D. * Vitamin C
E. Vitamin A
456.

What disease is developed in the vitamin C deficiency?


A. * Scurvy.
B. Pellagra.
C. Beri-beri.
D. Dermatitis.
E. Neuritis

457.

What is the function of calcitonin?


A. Promotes the transition of calcium from bones to blood, inhibits
reabsorption of phosphorus in kidney
B. Promotes the transition of calcium from blood to bones, stimulates
reabsorption of phosphorus in kidney
C. * Promotes the transition of calcium from blood to bones, inhibits
reabsorbtion of phosphorus in kidney
D. Promotes the transition of calcium from bones to blood, stimulates
reabsorption of phosphorus in kidney
E. Inhibits the exit of calcium from bones, activates reabsorption of
phosphorus in kidney

458.

What is the function of parathormone?

A. * Promotes transition of calcium from bones to blood, inhibits


reabsorbtion of phosphorus in kidney
B. Promotes transition of calcium from blood to bones, stimulates
reabsorption of phosphorus in kidney
C. Promotes transition of calcium from blood to bones, inhibits
reabsorbtion of phosphorus in kidney
D. Promotes transition of calcium from bones to blood, stimulates
reabsorption of phosphorus in kidney
E. Inhibits the exit of calcium from bones, inhibits reabsorption of
phosphorus in kidney
459.
What polysacharide (glycosaminoglycan) is the base of conjunctive
tissue intercellular matter, vitreous humor, synovial fluid, umbilical cord?
A. chondroitin sulphuric acid
B. * hyaluronic acid
C. keratan sulfate
D. heparin
E. heparan sulfate
460.
What tooth tissue containes: (in 100g of tissues): water 13g, organic
matters 20g, inorganic matters 69g, Ca 35g, 17g?
A. * Dentine
B. Pulp
C. Enamel
D. Cementum
E. All of the above
461.
What tooth tissue containes: (in 100g of tissues): water 2,5g,, organic
matters 4, inorganic matters 96g, Ca 36g, 17g?
A. Dentine
B. Pulp
C. * Enamel
D. Cementum
E. Root
462.
What tooth tissue containes: (in 100g of tissues): water 3,2g, organic
matters 2,5g, inorganic matters 70g, Ca 35,5g, 17g?
A. Root
B. Pulp
C. Dentine

D. * Cementum
E. Enamel
463.

Which class of antibody is found in saliva?


A. * IgA
B. IgG
C. IgM
D. IgD
E. IgE

464.
Which of the following do you expect to see in a patient with metabolic
acidosis with no respiratory compensation?
A. Decrease of [H+] in blood
B. * Decrease of [HCO3-] in blood
C. Decrease of [H2CO3] in blood
D. Decrease of pCO2 in blood
E. Decrease of dissolved CO2 in blood
465.

Which of the following effects are caused by calcitonin?


A. * Hypocalciemia, hypophosphatemia, hyperphosphaturia
B. Hypercalciemia, hypophosphatemia, hyperphosphaturia
C. Hypocalciemia, hyperphosphatemia, hyperphosphaturia
D. Hypercalciemia, hypophosphatemia, hypophosphaturia
E. Hypercalciemia, hyperphosphatemia, hypophosphaturia

466.

Which of the following occurs during prolonged fasting (starvation)?


A. Tissues use the same amount of glucose that they use during a brief
fast
B. Blood glucose levels decrease drastically during a four-week fast
C. * As a fast progresses, tissues rely predominantly on fuels derived from
adipose triacylglycerols
D. Blood ketone levels decrease dramatically after 3-5 days of fasting
E. After 3-5 days of fasting, the brain increases its utilization of glucose
and the rate of gluconeogenesis and the production of urea increases

467.

Xerostomia is:
A. * Dry mouth syndrome
B. Lack of one tooth
C. Lack of two teeth

D. Lack of enzymes in saliva


E. Hypersalivation

468.
A boy with congenital stenosis of pulmonary artery the dyspnea
increase during a walk, the cyanosis appears and he lost consciousness.
What is the basic mechanism development of this state?
A. * Acute brain hypoxia
B. Dilatation of peripheral vessels
C. Decrease of arterial pressure
D. Disorders of pulmonary ventilation
E. Disorders of gases diffusion in the lungs
469.

A human has an active expiration at the rest. What is the cause of this?
A. * Narrowing of respiratory tracts
B. Irritation of respiratory ways
C. Decrease of lung stretch
D. Negative intrapleural pressure
E. Irritation of intercostal muscles

470.
A miner has diagnosis pneumoconiosis. What non-respiratory
function of lungs is changed?
A. * Protective
B. Filtration
C. Excrection
D. Absorption
E. tabolic
471.
A patient has the atelectasis of lung with collapse of alveoli. What
factor is the most impotant in the pathogenesis of this disease?
A. * Defficiency of surfactant
B. Hyperventilation
C. Spasm of pulmonary vessels
D. Arterial hypertension
E. Respiratory acidosis
472.
A patient was poisoned by carbon monooxide. In how many times the
affinity for hemoglobin is higher than for oxygen?
A. * 300

B. 20
C. 15
D. 10
E. 5
473.
A patient with bronchial asthma has an acute respiratory insufficiency.
What type of breathing disorders is typical in this case?
A. * Obstructive disorders of alveolar ventilation
B. Restrictive disorders of alveolar ventilation
C. Perfusion
D. Diffusive
E. Disregulative disorders of alveolar ventilation
474.
A patient with the diagnosis of pneumoconiosis was hospitalized to
the clinic of professional diseases. The disorders of what component of the
external breathing is present in this patient?
A. * Depression diffusions of gases
B. Disorders ventilation of lungs
C. Disorders perfusion of lungs
D. Disorders the neural reguation of the external breathing
E. Disorders humoral regualtion of the external breathing
475.
A patiet with the penetrating wound of pectoral wall was hospitalized
to a clinic. What form disorders of the external breathing may develop in this
patient?
A. * Ventilative-restrictive
B. Obstructive
C. Primary-diskinetic
D. Difusion-restrictive
E. Difusion-pneumonosis
476.
A student executed intensive arbitrary hyperventilation. What changes
will be observed in his organism?
A. * Respiratory alcalosis
B. Respiratory acidosis
C. Hypoxemia
D. Hypercapnia
E. Hypoxemia and hypercapnia

477.
A woman has a bronchial asthma. What type of respiratory
insufficiency does she have?
A. * Obstructive
B. Restrictive
C. Pectoral
D. Abdominal
E. Mixed
478.
A woman has the attack of bronchial asthma with the bronchial spasm.
Activity what nerves nucleus is higter in this case?
A. * Vagus
B. Sympatethic
C. Gloso-faringeus
D. Trigeminus
E. Facialis
479.
A woman who inherits a pair of BRCA1 gene alleles has what chance for
developing breast cancer during her lifetime?
A. 0% chance
B. Approximately 50% chance
C. Approximately 100% chance
D. * Chance is the same as for the general population
480.

Alveolar ventilation is not violated at:


A. * Ischemic heart disease
B. Pneumonia
C. Bronchial asthma
D. Pleurisy
E. Child crowing

481.
An acute atelectasis of lungs developed in the newborn baby. What was
the most possible primary cause of this diseases?
A. * Absence of alveolar surfactant
B. Under-development of central nervous system
C. Increased aerodynamic resistance
D. Increased elastic resistance
E. Under-development respiratory muscles
482.

Arteriovenous difference on oxygen at tissue hypoxia:

A. * Diminishes
B. Does not change
C. Rises
D. Grows
E. Become even
483.

Arteriovenous difference on oxygen decrease at hypoxia:


A. * Tissue
B. Hemic
C. Circulatory
D. Respiratory
E. Hypoxic

484.

Arteriovenous difference on oxygen increase at hypoxia:


A. * Circulatory hypoxia
B. Hemic hypoxia
C. Respiratory hypoxia
D. Tissue hypoxia
E. Mixed hypoxia

485.

As a result decrease of oxygen capacity blood is developed:


A. * Hemic hypoxia
B. Mixed hypoxia
C. Hypoxic hypoxia
D. Circulatory hypoxia
E. Histotoxic hypoxia

486.

As a result of inactivation of hemoglobin is developed:


A. * Hemic hypoxia
B. Mixed hypoxia
C. Hypoxic hypoxia
D. Circulatory hypoxia
E. Histotoxic hypoxia

487.

As a result of methemoglobinemia is developed:


A. * Hemic hypoxia
B. Mixed hypoxia

C. Hypoxic hypoxia
D. Circulatory hypoxia
E. Histotoxic hypoxia
488.

At anaemia is developed:
A. * Hemic hypoxia
B. Mixed hypoxia
C. Hypoxic hypoxia
D. Circulatory hypoxia
E. Histotoxic hypoxia

489.

At bronchial asthma is developed:


A. * Hypoxic hypoxia
B. Mixed hypoxia
C. Hemic hypoxia
D. Circulatory hypoxia
E. Histotoxic hypoxia

490.

At cardiovascular insufficiency is developed:


A. * Circulatory hypoxia
B. Respiratory hypoxia
C. Hemic hypoxia
D. Hypoxic hypoxia
E. Histotoxic hypoxia

491.

At climb to the height in blood is developed:


A. * Hypocapnia
B. Hypercapnia
C. Acidosis
D. Hypoglycemia
E. Lymphocytosis

492.
At injection of sodium nitrite to the experimental frog colour of blood
will be:
A. * Dark red
B. Bright red
C. Green

D. Dark blue
E. Ordinary
493.
At injection to the experiment frog cyanic potassium the colour of
venous blood will be:
A. * Bright red
B. Dark red
C. Green
D. Dark blue
E. Ordinary
494.

At pneumonia is developed:
A. * Hypoxic hypoxia
B. Mixed hypoxia
C. Hemic hypoxia
D. Circulatory hypoxia
E. Histotoxic hypoxia

495.

At poisoning by carbon oxide is developed:


A. * Hemic hypoxia
B. Mixed hypoxia
C. Hypoxic hypoxia
D. Circulatory hypoxia
E. Histotoxic hypoxia

496.

At poisoning by cyanic potassium is developed:


A. * Histotoxic hypoxia
B. Mixed hypoxia
C. Hypoxic hypoxia
D. Circulatory hypoxia
E. Hemic hypoxia

497.

At poisoning by nitrates hypoxic hypoxia develop as a result:


A. * Formation methemoglobin
B. Formation carboxyhemoglobin
C. Formation deoxyhemoglobin
D. Formation reduced hemoglobin

E. Formation carbhemoglobin
498.

At poisoning by nitrates is developed:


A. * Hemic hypoxia
B. Mixed hypoxia
C. Hypoxic hypoxia
D. Circulatory hypoxia
E. Histotoxic hypoxia

499.

At poisoning carbon oxide develops hypoxic hypoxia as a result:


A. * Formation carboxyhemoglobin
B. Formation methemoglobin
C. Formation deoxyhemoglobin
D. Formation reduced hemoglobin
E. Formation carbhemoglobin

500.

At the disorders of blood circulation in the organ is developed in it:


A. * Circulatory hypoxia
B. Mixed hypoxia
C. Hemic hypoxia
D. Hypoxic hypoxia
E. Histotoxic hypoxia

501.

At the disorders of blood circulation will be development:


A. * Circulatory hypoxia
B. Mixed hypoxia
C. Hemic hypoxia
D. Hypoxic hypoxia
E. Histotoxic hypoxia

502.
Based on research findings, what preoperative factor is most likely to
result in increased postoperative pain?
A. Older age
B. * High anxiety
C. Smoking history
D. Presence of chronic pain
503.

Bradypnea it is:

A. * Slow and deep breathing


B. Slow and shallow breathing
C. Frequent and shallow breathing
D. Frequent and deep breathing
E. Increasing breathing
504.

Bradypnea typical for:


A. * Stenosis of larynges
B. Hyperketonemic comas
C. Edema lungs
D. Asphyxias
E. Pneumonia

505.

Circulatory hypoxia is arose at:


A. * Cardiovascular insufficiency
B. Pneumonia
C. Poisoning by carbon oxide
D. Poisoning by cyanides
E. Bronchitis

506.

Collaps airway is named:


A. * Atelectasis
B. Hypercapnia
C. Hypoxia
D. Apnea
E. Asphyxia

507.
A patient with congenital stenosis of pulmonary artery the dyspnea
increase during a walk, the cyanosis appears and he lost consciousness.
What is the basic mechanism development of this state?
A. * Acute brain hypoxia
B. Dilatation of peripheral vessels
C. Decrease of arterial pressure
D. Disorders of pulmonary ventilation
E. Disorders of gases diffusion in the lungs
508.

Cyanosis appears at the increase of maintenance in a blood:


A. * Reduced hemoglobin

B. Methemoglobin
C. Oxyhemoglobin
D. Carboxyhemoglobin
E. Carbhemoglobin
509.

Cyanosis of skin develops as a result increase in the blood capillaries:


A. * Deoxyhemoglobin
B. Carboxyhemoglobin
C. Erythrocytes
D. Methemoglobin
E. Carbhemoglobin

510.

Decrease of saturation of arterial blood by oxygen characterized for:


A. * Hypoxic hypoxia
B. Mixed hypoxia
C. Hemic hypoxia
D. Circulatory hypoxia
E. Histotoxic hypoxia

511.

Decrease of surfactant is results development


A. * Restrictive respiratory insufficiency
B. Obstructive respiratory insufficiency
C. Mixed respiratory insufficiency
D. Central respiratory insufficiency
E. Peripheral respiratory insufficiency

512.
Deep and infrequent breathing, that is arose at stenosis of upper
respiratory ways is named:
A. * Stenotic
B. Bradypnea
C. Hypoxic
D. Apnea
E. Hyperpnea
513.

Dense regular collagenous connective tissue would be found in


A. A nerve
B. The brain

C. A ligament
D. * Skull bone
E. Lung
514.

Development of tachypnea at the pneumonia is related to:


A. * Speed up reflex Hering-Breyer`s and surplus accumulation of 2 in
blood
B. Deceleration reflex Hering-Breyer`s and surplus accumulation of 2
in blood
C. Speed up reflex Hering-Breyer`s and decrease of accumulation of 2
in blood
D. Deceleration of the Hering-Breyer`s reflex and decrease of
accumulation of 2 in blood
E. Absence of reflex Hering-Breyer`s and surplus accumulation of 2

515.
During auscultation of lungs wheeze are listened. What type of external
breathing disorders may be suspected in that patient?
A. * Obstructive
B. Primary-diskinetic
C. Ventilation-restrictivE)
D. Difusion-restrictive
E. Difusion-pneumonosis
516.
During climbing up to the mountains without oxygen bulbs in the
alpinists can develop:
A. * Hypoxic hypoxia
B. Mixed hypoxia
C. Hemic hypoxia
D. Circulatory hypoxia
E. Histotoxic hypoxia
517.
During dehermetization of airplane at height 9000m in the passengers
will develop:
A. * Hypoxic hypoxia
B. Mixed hypoxia
C. Hemic hypoxia
D. Circulatory hypoxia
E. Histotoxic hypoxia

518.
During the preoperative assessment, the client tells the nurse that he
has been taking NSAIDs for years. What question should the nurse ask the
client?
A. * When was the last time you took your NSAIDs?
B. Have you ever vomited blood?
C. Have you told the anesthesiologist that you are taking NSAIDs?
D. What is the dosage or how many tablets do you take every day?
519.

Erythrocytosis at hypoxia develops under influence of:


A. * Erythropoietin
B. Adrenaline
C. Insulin
D. Glucagon
E. Lymphokines

520.

For bronchial asthma typically:


A. * Expiratory dyspnea
B. Inspiratory dyspnea
C. Mixed dyspnea
D. Pectoral dyspnea
E. Abdominal dyspnea

521.

For hemic hypoxia there are the typical changes in blood:


A. * Anemia
B. Erythrocytosis
C. Leukocytosis
D. Leukopenia
E. Lymphocytosis

522.

For hypoxic hypoxia there are the typical changes in blood:


A. * Erythrocytosis
B. Anemia
C. Leukocytosis
D. Leukopenia
E. Lymphocytosis

523.

For pneumonia is typical:

A. * Speed up reflex Hering-Breyer`s and surplus accumulation 2 in


blood
B. Deceleration reflex Hering-Breyer`s and surplus accumulation 2 in
blood
C. Speed up reflex Hering-Breyer`s and decrease of the accumulation
2 in blood
D. eceleration of the Hering-Breyer`s reflex and decrease of the
accumulation 2 in blood
E. Absence reflex Hering-Breyer`s and surplus accumulation 2
524.

For pneumonia typical:


A. * Tachypnea
B. Hyperpnea
C. Bradypnea
D. Periodic breathing
E. Apneuistic breathing

525.
For the improvement of the common status a patient with bronchial
asthma must breathe out:
A. * Slowly
B. Fast
C. Usually
D. Speed-up
E. Throug nose
526.
For what type of cancer should the nurse be prepared to administer
chemotherapy by the intrathecal route?
A. Lung tumor
B. * Brain tumor
C. Ovarian tumor
D. Prostate tumor
527.
How does residual volume change at the obstructive type of respiratory
insufficiency?
A. * Increase
B. Does not change
C. Decrease
D. Does not change or increase
E. Does not change

528.

How does residual volume change at the restrictive lung disease?


A. * Does not change or decrease
B. Increase
C. Decrease
D. Does not change or increase
E. Does not change

529.
How does the vital capacity of lungs at the obstructive type of
pulmonary insufficiency change?
A. * Does not change or decrease
B. Increase
C. Decrease
D. Does not change or increase
E. Does not change
530.
How does the vital capacity of lungs at the restrictive type of
pulmonary insufficiency change?
A. * Decrease
B. Does not change or decrease
C. Increase
D. Does not change or increase
E. Does not change
531.

Hypercapnia it is:
A. * Increase of 2
B. Decrease of 2
C. Decrease of 2
D. Increase of 2
E. Increase of 2 and 2

532.

Hyperpnea develops as a result:


A. * Irritation of respiratory center by the surplus 2
B. Acceleration reflex Hering-Breyer`s
C. Increase of hemoglobin in blood
D. Deceleration of the Hering-Breyer`s reflex
E. Decrease of 2 amount in blood

533.

Hyperpnea it is:

A. * Frequent and deep breathing


B. Slow and shallow breathing
C. Frequent and shallow breathing
D. Slow and deep breathing
E. Increasing breathing
534.

Hyperpnea typical for:


A. * Hyperketonemic comas
B. Stenosis larynx
C. Bronchitis
D. Asphyxias
E. Pneumonia

535.

Hypocapnia it is:
A. * Decrease of 2
B. Increase of 2
C. Decrease of 2
D. Increase of 2
E. Increase of 2 and 2

536.

Hypoxemia it is decrease :
A. * Contents of oxygen in a blood
B. Contents of oxygen in tissue
C. Alveolar ventilation
D. Contents of 2 in tissue
E. Contents of 2 in a blood

537.

Hypoxia is the state when tissues:


A. * Got insufficient of oxygen
B. Glucose is not got
C. The increased amount of oxygen is got
D. The insufficient amount of hemoglobin is got
E. The insufficient amount of red corpuscles is got

538.
In a child is deficit of surfactant. What pathological changes in
pulmonary tissue as a result of absence this substance?
A. * Atelectasis

B. Emphysema
C. Bronchoconstriction
D. Edema
E. Limfostasis
539.
In a child with bronchial asthma an acute respiratory failure developed
due to asthmatic attack. The cause of this complication is disorders of:
A. * Alveolar ventilation
B. Alveolar blood circulation
C. Oxygen capacity of blood
D. Dissociation of oxyhemoglobin
E. Utilization of oxygen
540.
In a patient decrease activity of lung surfactant. What changes can be
expected in this patient?
A. * Tendency alveolus to collapse
B. Change elasticity of lung
C. Decrease tracheobronchial secret
D. Change diffusion of gases
E. Violation perfusion of lungs
541.
In a patient was diagnosed emphysema of lungs. What is the main
cause of this condition?
A. * The decrease elastic properties of lungs
B. Decrease alveolar ventilation
C. Decease extensibility of lungs
D. Decrease blood stream in lungs
E. Disorders ventilation-perfusion ratio
542.
In a patient with emphysema the functional status of the respiratory
system was examined The most typical changes characterized for this
disease?
A. * Increase remaining volume of lungs
B. Increase vital capacity of lungs
C. Increase reserve volume of inhalation
D. Decrease general capacity of lungs
E. Decrease reserve volume of inhalation
543.

In a patients with the nonclosed of the Botal`s duct may develop:

A. * Hypoxic hypoxia
B. Mixed hypoxia
C. Hemic hypoxia
D. Circulatory hypoxia
E. Histotoxic hypoxia
544.

In basis of primary tissue hypoxia is decrease:


A. * Activity of respiratory enzymes
B. Oxygen capacity of blood
C. Partial pressure of oxygen in air, that inhales
D. Hemoglobin
E. Red corpuscles

545.
In how many times the ability of 2 for diffusion through an alveolarcapillary membrane is higher, than that ability of oxygen?
A. * 25
B. 20
C. 15
D. 10
E. 5
546.
In premature new-born the syndrome of respiratory failure is often
developed What is the most possible reason of this disease?
A. * Insufficient quantity of surfactant
B. Prenatal asphyxia
C. Prenatal hypoxia
D. Swallowing of parafetus waters
E. Prenatal hypercapnia
547.
In which of the enumerated pathological processes you can determine
the obstructive form of external breathing disorders?
A. * Bronchial asthma
B. Opened pneumatothorax
C. Pneumonia
D. Pleurisy
E. Lung edema
548.

It is differed the following types of hypoxia with except of:

A. * Hypokinetic
B. Hypoxic
C. Hemic
D. Circulatory
E. Histotoxic
549.

Mixing of arterial and vein blood will result to development:


A. * Hypoxic hypoxia
B. Mixed hypoxia
C. Hemic hypoxia
D. Circulatory hypoxia
E. Histotoxic hypoxia

550.

Oxygen capacity of blood it is:


A. * Maximal amount of oxygen, which can bind 100 ml blood at complete
saturation of hemoglobin by oxygen
B. Amount of oxygen, that is contained in a blood
C. Amount of oxygen, dissolved in plasma
D. Amount of oxygen, that is contained in red corpuscles
E. Amount of oxygen, that is contained in 1 g hemoglobin

551.
Pathological process which develops as a result of low supply to tissues
of oxygen or violation of the use it by tissues is named:
A. * Hypoxia
B. Hypercapnia
C. Bradypnea
D. Tachypnea
E. Polypnea
552.

Periods of apnea typical for:


A. * Periodic respiration
B. Hyperpnea
C. Bradypnea
D. Tachypnea
E. Apneuistic breathing

553.
A boys has poisoning of nitrites. What is a main mechanism in
pathogenesis of this disorders?

A. * Insufficiency of met-b-reductase
B. Insufficiency of superoxyddesmutase
C. Blocade of cytochromoxydase
D. Insufficiency of glutationperoxydase
E. Insufficiency of katalase
554.

Restrictive type of respiratory insufficiency typical for:


A. * Pneumonia
B. Poisoning of cyanides
C. Bronchial asthma
D. Bronchitis
E. Child crowing

555.
Retropharyngeal abscess was diagnosed in a patient H. What form of
respiratory insufficiency is present in this patient?
A. * Obstructive
B. Central
C. Periferal
D. Restrictive
E. Thoracic-diaphragmatic
556.

Single respiratory motions before the final stop of breathing it is:


A. * Gasping
B. Bradypnea
C. Hypoxia
D. Periodic breathing
E. Hyperpnea

557.
State of organism, that is characterized by the decrease of
maintenance oxygen and increase of carbonic acid in blood and tissues is
named:
A. * Asphyxia
B. Hypercapnia
C. Hypoxia
D. Apnea
E. Hyperpnea
558.

Tachypnea it is:

A. * Frequent and shallow breathing


B. Slow and shallow breathing
C. Slow and deep breathing
D. Frequent and deep breathing
E. Increasing breathing
559.

Tachypnea typical for:


A. * Pneumonia
B. Stenosis of larynges
C. Bronchitis
D. Asphyxia
E. Violation of respiratory center

560.
Temporal decrease of the maintenance 2 in blood and reflex
disorder of breathing will result in development:
A. * Apnea
B. Bradypnea
C. Hypoxia
D. Stenosis
E. Hyperpnea
561.

Temporal stop of breathing this is:


A. * Apnea
B. Bradypnea
C. Hypoxia
D. Stenosis
E. Hyperpnea

562.
is:

A differential observation of pleural fluid associated with tuberculosis


A. Increased neutrophils
B. Decreased lymphocytes
C. * Decreased mesothelial cells
D. Increased mesothelial cells
E. No correct answer

563.

A mesothelioma cell seen in pleural fluid indicates:

A. Bacterial endocarditis
B. * Primary malignancy
C. Metastatic lung malignancy
D. Tuberculosis infection
E. No correct answer
564.

A milky-appearing pleural fluid is indicative of:


A. Thoracic duct leakage
B. Chronic inflammation
C. Microbial infection
D. * Both A and B
E. No correct answer

565.

A pleural fluid pH of 6.0 is indicative of:


A. * Esophageal rupture
B. Mesothelioma
C. Malignancy
D. Rheumatoid effusion
E. No correct answer

566.

Alkaline phosphates level in children is increase due to:


A. Decrease immunity
B. Increase immunity
C. * Decrease osteoblastic activity
D. Increase osteoblastic
E. All of the above

567.

All are physical properties of sputum except:


A. Smell
B. Colour
C. Consistency
D. Volume
E. * All of the above

568.

All of the following are characteristics of malignant cells except:


A. Cytoplasmic molding
B. * Absence of nucleoli

C. Mucin-containing vacuoles
D. Increased N:C ratio
E. No correct answer
569.

All of the following are normal cells seen in pleural fluid except:
A. Mesothelial cells
B. Neutrophils
C. Lymphocytes
D. * Mesothelioma cells
E. No correct answer

570.
An additional test performed on pleural fluid to classify the fluid as a
transudate or exudate is the:
A. WBC count
B. RBC count
C. * Fluid-to-cholesterol ratio
D. Fluid-toserum protein gradient
E. No correct answer
571.

An increase in the amount of serous fluid is called a/an:


A. Exudate
B. Transudate
C. * Effusion
D. Malignancy
E. No correct answer

572.

Analysis of paracentesis fluid is performed to:


A. Determing cause of fluid presence
B. Asses infection risk
C. Determine lung involvement
D. All of the above
E. * A and B only

573.

Another name for a peritoneal effusion is:


A. Peritonitis
B. Lavage
C. * Ascites

D. Cirrhosis
E. No correct answer
574.
Chemical tests primarily performed on peritoneal fluid include all of the
following except:
A. * Lactose dehydrogenase
B. Glucose
C. Alkaline phosphatase
D. Amylase
E. No correct answer
575.

Clinical analysis of sputum includes:


A. Bacteriological and Physiological investigation
B. Morphological and Physiological investigation
C. * Physical, Microscopic and Bacterioscopic investigation
D. Psychological and Bacteriological investigated
E. Physiological and Microscopical investigation

576.

Consistency of sputum depends on all except:


A. Viscous
B. Sticky
C. Liquid
D. Gelatinous
E. * Amount

577.

Cultures of peritoneal fluid are incubated:


A. Aerobically
B. Anaerobically
C. At 37C and 42C
D. * Both A and B
E. No correct answer

578.
Define, what pathologic state is characterized by the increasing of
creatine kinase activity in 2-3 hours from disease beginning , in 13-20 hours
exceeds a norm in 5 -10 times, normalizes on 2 - 3 days?
A. Lung infarction
B. * Myocardial infarction
C. Chronic heart failure

D. Heart attack, expressed by violation of blood coronal circulation


E. There is not a right answer
579.
Differentiation between bacterial peritonitis and cir-rhosis is done by
performing a/an:
A. WBC count
B. Differential
C. * Absolute neutrophil count
D. Absolute lymphocyte count
E. No correct answer
580.

During normal production of serous fluid, the slight excess of fluid is:
A. * Absorbed by the lymphatic system
B. Absorbed through the visceral capillaries
C. Stored in the mesothelial cells
D. Metabolized by the mesothelial cells
E. No correct answer

581.

Feculent odor in sputum has diagnostic value in case of:


A. Viral infection
B. Bronchial asthma
C. * Anerobic bacterial infection
D. Tuberculosis
E. None of the above

582.
Fluid-toserum protein and lactic dehydrogenase ratios are performed
on serous fluids:
A. When malignancy is suspected
B. * To classify transudates and exudates
C. To determine the type of serous fluid
D. When a traumatic tap has occurred
E. No correct answer
583.

Frothy sputum has diagnostic value in case of:


A. * Lungs edema
B. Tuberculosis
C. Lungs abscess
D. No illness

E. Bronchial asthma
584.
In a patient with chronic bronchitis what consistency of sputum will be
found?
A. Viscous
B. Sticky
C. * Liquid
D. Gelatinous
E. Hard
585.
In case of patient with pulmonary hemorrhage, edema of the lungs.
What will be the consistency of the sputum?
A. * Liquid
B. Gelatinous
C. Sticky
D. Viscous
E. Hard
586.

Indicate the correct color of sputum in bronchial asthma.


A. * Viscous-glassy
B. Purulent
C. Rusty
D. Mucous-purulent
E. Glassy-purulent

587.

Kurshmans spirals in sputum have diagnostic value in case of:


A. * Bronchial asthma
B. Bronchial pneumonia
C. Pulmonary tuberculosis
D. Lung cancer
E. Lungs abscess

588.
Patient was admitted to the hospital with lobar pneumonia, what kind
of mucous will be found?
A. * Viscous
B. Liquid
C. Gelatinous
D. Sticky

E. Hard
589.
Patient was diagnosed with bronchial asthma, what kind of sputum will
be found?
A. Purulent
B. * Mucous
C. Bloody
D. Serous
E. Mucous-purulent
590.

Pleural fluid is collected by:


A. Pleurocentesis
B. Paracentesis
C. Pericentesis
D. * Thoracentesis
E. No correct answer

591.

Pleural fluid transudate:


A. Reflects primary involvement of the pleura
B. Is characterized by an increases LD F/P ratio
C. Is characterized by an increases glucose F/P ratio
D. Is characterized by a total protein F/P ratio of 0,5
E. * All of the above

592.

Production of serous fluid is controlled by:


A. Capillary oncotic pressure
B. Capillary hydrostatic pressure
C. Capillary permeability
D. * All of the above
E. No correct answer

593.

Serous fluids:
A. Are derived from serum
B. Provide lubrication and protection
C. Fill the potential space
D. * All of the above
E. A and B only

594.
Some of the tests performed on the pleural fluid to classify the fluid as
transudate or exhudate is:
A. WBC count
B. RBC count
C. Platelet count
D. * Fluid-to-cholesterol ratio
E. No correct answer
595.

Sputum is taken for the investigation of the following diseases:


A. Malaria
B. * Tuberculosis
C. Glaucoma
D. Typhoid
E. Rheumatism
F. * Chylomicron

596.

The membrane that lines the wall of a cavity is the:


A. Visceral
B. Peritoneal
C. Pleural
D. * Parietal
E. No correct answer

597.

The most common cause of ascites is:


A. * Portal hypertension
B. Venous return
C. Parietal cell differentiation
D. Eccrine infection
E. Type A cell leakage

598.

The primary purpose of serous fluid is:


A. Removal of waste products
B. Lowering of capillary pressure
C. * Lubrication of serous membranes
D. Nourishing serous membranes
E. No correct answer

599.
The recommended test for determining if peritoneal fluid is a
transudate or an exudate is the:
A. Fluid-toserum albumin ratio
B. * Serum ascites albumin gradient
C. Fluid-toserum lactic dehydrogenase ratio
D. Absolute neutrophil count
E. No correct answer
600.

The test performed on peritoneal lavage fluid is:


A. WBC count
B. * RBC count
C. Absolute neutrophil count
D. Amylase
E. No correct answer

601.

Viscous mucous is found in case of all of the following except:


A. Bronchial pneumonia
B. Lungs abscess
C. * Pulmonary tuberculosis
D. Bronchiectasis
E. Lung cancer

602.

What is sputum?
A. A pathological substance secreted from the GIT
B. * A pathological secretion formed in case of respiratory diseases
C. Mucous secretion from the colon
D. Chemical substance which becomes mucous
E. An enzyme

603.
What is the amount of sputum secreted in case of acute bronchitis and
bronchial asthma?
A. 2 to 3 ml
B. * 200 to 300 ml
C. 300 to 500 ml
D. 50 to 100 ml
E. 10 to 25 ml
604.

What is the best time for collection of sputum sample?

A. At night
B. * Early morning before eating
C. After meal
D. Before meal
E. In the afternoon
605.
What is the character of the sputum in a patient with pulmonary
tuberculosis?
A. Mucous sputum
B. * Purulent sputum
C. Bloody sputum
D. Serous sputum
E. Purulent-mucous sputum
606.

What is the color of sputum in lung cancer?


A. Grey
B. Greyish-yellow
C. * Mucous purulent bloody
D. Rusty
E. Mucous

607.

What is the smell of fresh sputum?


A. Sharp
B. Acidic
C. Fruity
D. * Odorless
E. Sweet

608.

Which is the smell of fresh sputum?


A. Rotten
B. * Odourless
C. Rotten grapes
D. Evil smell
E. Sweet smell

609.

What kind of mucous is found in case of chronic bronchitis?


A. Viscous

B. * Liquid
C. Gelatinous
D. Sticky
E. Hard
610.

?What kind of color will be observed in bloody sputum?


A. * Red
B. Yellow
C. Gray
D. Brown
E. Yellow-gray

611.

What kind of mucous is found in case of lobar pneumonia?


A. * Viscous
B. Liquid
C. Gelatinous
D. Sticky
E. Hard

612.

What time should sputum be collected?


A. * In the morning before eating
B. In the evening after supper
C. Before brushing teeth
D. In the morning
E. After fasting

613.

What type of sputum is secreted in case of chronic bronchitis?


A. Viscous
B. * Mucous-purulent
C. Purulent
D. Mucous
E. Glassy

614.

What type of sputum will be seen in case of pulmonary tuberculosis?


A. * Bloody sputum
B. Purulent sputum
C. Serous sputum

D. Mucous sputum
E. Purulent-mucous sputum
615.

What type of sputum will be seen in case of bronchiectasis?


A. * Purulent
B. Mucous
C. Bloody sputum
D. Serous sputum
E. Mucous-purulent

616.

Which is the most appropriate way for collection of sputum?


A. * Needle aspiration
B. Vomiting
C. Spitting
D. Swab
E. Smear

617.
Which of the following appears in macroscopic examination of sputum
in case of bronchial asthma?
A. * Kurshmans spirals
B. Rice-like bodies
C. Koch lenses
D. Crystals of cholesterol
E. Ehrlich tetrads
618.
Which of the following appears in macroscopic examination of sputum
in case of pulmonary tuberculosis?
A. Kurshmans spirals
B. * Koch lenses
C. Elrich tetrasd
D. Mucous purulent bloody
E. Visceral membranes
619.
Which of the following appears in microscopic examination of sputum
in the case chronic bronchitis?
A. * Alveolar macrophages and leukocytes
B. Eosinophils
C. Charcot-leyden crystals

D. Crystals of fatty acids


E. Atypical cells
620.

Which of the following best represents a hemothorax?


A. Blood HCT: 42 Fluid HCT: 15
B. Blood HCT: 42 Fluid HCT: 10
C. Blood HCT: 30 Fluid HCT: 10
D. * Blood HCT: 30 Fluid HCT: 20
E. No correct answer

621.

Which of these elements are not present in sputum in healthy person?


A. Leukocytes
B. Flat epithelium
C. Elastic fibers
D. Eosinophils
E. * Red blood cells

622.

Which of these is not a pathological component of sputum?


A. Blood
B. Rice-like bodies
C. Kurshmans spirals
D. Ditryh tubes
E. * None of the above

623.

Which of this is not a characteristic of sputum?


A. Mucous sputum
B. Bloody sputum
C. Serous sputum
D. Purulent sputum
E. * Gelatinous

624.
____________ is hydrolytically cleaved to directly yield urea in the urea
cycle.
A. ornithine
B. glutamate

C. * arginine
D. carbamoyl phosphate
E. None of the above
625.

A diuresis of a healthy adult is about:


A. * 1000 2000 ml daily
B. 500 1000 ml daily
C. 1000 3000 ml daily
D. 100 500 ml daily
E. 3000 5000 ml daily

626.

A diuresis of a healthy adult is about:


A. * 1000 2000 ml daily
B. 500 1000 ml daily
C. 1000 3000 ml daily
D. 100 500 ml daily
E. 3000 5000 ml daily

627.
A hyperketonemic coma developed in a patient with diabetes mellitus.
What type violation of acid-basic balance he has?
A. * Metabolic acidosis
B. Exogenous acidosis
C. Respiratory acidosis
D. Respiratory alkalosis
E. Not respiratory alkalosis
628.

A main compensatory mechanism at the metabolic acidosis is


A. * Binding ions of hydrogen by a bicarbonate buffer
B. Infrequent and shallow breathing
C. Elimination of bicarbonates by kidneys
D. Decline produce of aldosterone
E. Movement ions of hydrogen from red corpuscles in plasma

629.
A man has chronic glomerulonephritis. In the examination in him
absence of appetite, vomiting, diarrhea, itch of skin, anemia. Contents of
residual nitrogen of blood - 43 mmol/l. The indicated signs are caused:
A. * Disorder of secretory function of nephron
B. Increase of permeability of glomerular filter

C. Ischemia of kidneys
D. Disorder of resorption of bicarbonates
E. Disorder of concentrating mechanism
630.
A man with a chronic pyelonephritis has arterial hypertension. What did
cause this phenomenon?
A. * Activation of renin-angiotensin system
B. Activation of central cholinergic mechanism
C. Decrease of partial pressure of oxygen in kidney
D. Activation of angiotensinase synthesis in kidney
E. Activation of acidogenesis and ammoniagenesis in kidneys
631.
A patient complaints of permanent thirst, polyuria (to 10 l per day).He
carried a cranio-cerebral trauma earlier. Relative density of urine -1008,
pathological components are not present. The secretion of what hormone is
disordered in this case?
A. * Deficient of antidiuretic hormone synthesis
B. Increase of antidiuretic hormone production
C. Deficient of insulin synthesis
D. Hyperproduction of aldosterone
E. Deficient of aldosterone synthesis
632.
A patient entered to the department of resuscitation. Arterial pressure
90/60 mm Hg, in a blood high maintenance of creatinine and urea, day's
diuresis 80 ml. In him:
A. * Anuria
B. Olyguria
C. Polyuria
D. Pollakiuria
E. Nycturia
633.
A patient had anuria. The arterial pressure - 55/20 mm Hg. Disorder of
what process of uropoiesis became the cause of acute decrease of urine
passage?
A. * Glomerular filtrations
B. Obligate resorption
C. Facultative resorption
D. Tubular secretion
E. All enumerated processes

634.
A patient has a bleeding gastric ulcer. Arterial pressure 80/60 mm Hg
He excretes 60-80 ml urine on days. The amount of residual nitrogen and
urea is increased in plasma of blood. What main mechanism of falling of day's
diuresis?
A. * Decrease hydrostatic pressure in the nephron capillaries
B. Increase of urine osmotic pressure
C. High level of nitrogen in blood
D. Increase of colloid-osmotic pressure in blood
E. Increase of renal pressure
635.
A patient has chronic glomerulonephritis, glomerular filtration rate is
reduced to 20% from normal. What is the principal cause of the decline of
glomerular filtration in this case?
A. * Decrease amounts of functioning nephrons
B. Tubulopathy
C. Obstruction of urinary tract
D. Ischemia of kidneys
E. Thrombosis of kidney arteries
636.
A patient has complaints of great pains in a lumbar area, increase of
temperature to 39 C. At examination was observed: positive symptom
Pastepnatsky. There is leukocytosis , ESR is raised. In urine: albumen 0,039
, leucocytes 250-300 in field of view, red corpuscles 8-10 in field of view,
mucus +++, bacterias +++. What does disease in the patient ?
A. * Acute pyelonephritis
B. Acute glomerulonephritis
C. Chronic glomerulonephritis
D. Nephrolithiasis
E. Chronic kidney insufficiency
637.
A patient has diabetic nephropathy with development of
uremia.Glomerular filtration rate - 9 ml/min. What main mechanism of
decreasing of glomerular filtration rate?
A. * Decrease of functioning nephrons amount
B. Decrease of systemic arterial pressure
C. Occlusion of tubules
D. Development of acidosis in tissues
E. Spasm of afferent glomerular arteriole
638.
A patient has lumbodynia, increase of temperature to 39C. Positive
Pasternatskys symptom on the other side, Hb - 115 g/l, leukocyturia. What is
disease in this patient?

A. * Acute pyelonephritis
B. Acute glomerulonephritis
C. Chronic glomerulonephritis
D. Nephrolithiasis
E. Chronic kidney insufficiency
639.
A patient who had frequent protracted quinsy in anamnesis complained
of periodic headache, rapid tiredness, periorbital edema. Clinicodiagnostic
researches was revealed arterial hypertension, proteinuria , hypoproteinemia,
hyperlipidemia. What disease is most likely in this case?
A. * Chronic glomerulonephritis
B. Acute pyelonephritis
C. Chronic pyelonephritis
D. Lipoid nephrosis
E. Acute kidney insufficiency
640.
A patient with arterial hypertension used furosemyd. In the next time in
him arose general weakness, loss of appetite, palpitation, lower of blood
pressure and intestine peristaltic . The cause of such change can be
A. * Hypokalemia
B. Hyponatremia
C. Hyperuricemia
D. Hypercalcemia
E. Hyperkalemia
641.
A patient with chronic glomerulonephritis has edemas. What is cause of
their development?
A. * Proteinuria
B. Disorder of liver functions
C. Hyperosmolarity of plasma
D. Hyperaldosteronism
E. Hyperproduction of vasopressin
642.
A patient with chronic kidney insufficiency has the dyspepsia,
irrepressible itch of skin, general weakness, smell of ammonia and greyearthy tint of skin. These symptoms are characterized for
A. * Decrease of excretion products of nitrogenous metabolism
B. Disorder of water-electrolyte metabolism
C. Disorder acid-basic balance

D. Disorder f carbohydrate metabolism


E. Disorder of lipid metabolism
643.
A patient with the cirrhosis of liver have the considerable edema.
Among the causes of their development the most likely is:
A. * Decrease of albumen synthesis
B. Surplus of aldosterone secretion
C. Surplus of antidiuretic hormone secretion
D. Surplus of natriuretic hormone secretion
E. Disorder of antidiuretic hormone inactivation
644.

A pH of urine in a norm is:


A. * 5,3 6,8
B. 3,5 5
C. 2,1 4,3
D. 1,45 1,85
E. 5,5 7,5

645.

About the kidney insufficiency testify decrease of filtration to:


A. * 40 ml/min
B. 50 ml/min
C. 70 ml/min
D. 60 ml/min
E. 80 ml/min

646.

About the kidney insufficiency testify decrease of filtration to:


A. * 40 ml/min
B. 50 ml/min
C. 70 ml/min
D. 60 ml/min
E. 80 ml/min

647.

Acid-Base balance is important for:


A. Normal enzyme functions.
B. Normal metabolite solubility.
C. Normal membrane potentials.
D. A and C

E. * All of the above


648.

Addis test is the measure of


A. * Impairment of the capacity of the tubule to perform osmotic work
B. Secretory function of liver
C. Excretory function of liver
D. Activity of parenchymal cells of liver
E. All of the above

649.

ADH test is based on the measurement of


A. * Specific gravity of urine
B. Concentration of urea in urine
C. Concentration of urea in blood
D. Volume of urine in ml/minute
E. A and D

650.

ADH test is based on the measurement of


A. * Specific gravity of urine
B. Concentration of urea in urine
C. Concentration of urea in blood
D. Volume of urine in ml/minute
E. Both a and B

651.

Albumin-globulin coefficient of blood plasma of healthy adult is:


A. 3,0-4,0
B. 1,0-1,5
C. 5,0-6,5
D. 8,0-10,0
E. * 1,5-2,0

652.

All the following are true about phenylketonuria except


A. Deficiency of phenylalanine hydroxylase
B. Mental retardation
C. * Increased urinary excretion of hydroxyphenyl pyruvic acid
D. Decrease serotonin formation
E. C and D

653.

All the following statements about phenylketonuria are correct except

A. Phenylalanine cannot be converted into tyrosine


B. Urinary excretion of phenylpyruvate and phenyllactate is increased
C. It can be controlled by giving a low phenylalanine diet
D. * It leads to decreased synthesis of thyroid hormones, catecholamines
and melanin
E. Both A and D
654.

Amount of uric acid which excreted daily is:


A. Not excreted
B. 10-12 g/day
C. 4-8 mg/day
D. * 270-600 mg/day
E. 2-5 g/day

655.

An early feature of renal disease is


A. * Impairment of the capacity of the tubule to perform osmotic work
B. Decrease in maximal tubular excretory capacity
C. Decrease in filtration factor
D. Decrease in renal plasma flow
E. All of the above

656.

An early feature of renal disease is


A. * Impairment of the capacity of the tubule to perform osmotic work
B. Decrease in maximal tubular excretory capacity
C. Decrease in filtration factor
D. Decrease in renal plasma flow
E. Both A and D

657.
As a result of starvation ascites developed in a child. What mechanism
of it development?
A. * Decrease of oncotic pressure
B. Membranogenic
C. Disregulatory
D. Lymphogenic
E. Hyperosmolar
658.
As a result of the frequent vomiting the pregnant lost the gastric juice.
What disorder of acid-basic balance can be in her?

A. * Metabolic alkalosis
B. Respiratory acidosis
C. Not respiratory acidosis
D. Respiratory alkalosis
E. Metabolic acidosis
659.
At a biochemical inspection of a patient were found a hyperglycemia,
glucosuria, high urines density, in blood found - enhanceable amount of
glucocorticoids. At the same time in blood and urine found excess of 17
ketosteroids. What type of diabetes developed:
A. * Steroid diabetes
B. Diabetes Mellitus, the I type
C. Diabetes Mellitus , the II type
D. Kidney diabetes
E. Hepatic diabetes
660.

At the diabetes mellitus is developed:


A. * Hyperosmolar dehydration
B. Isoosmolar dehydration
C. Hypoosmolar dehydration
D. Hypoosmolar hyperhydration
E. Isoosmolar hypohydration

661.
At the top of mountain respiratory alkalosis developed in the alpinists.
What level of the tension 2 is possible in his arterial blood?
A. * 30 mm Hg
B. 40 mm Hg
C. 50 mm Hg
D. 60 mm Hg
E. 70 mm Hg
662.

At what disease colour of urine is darkly brown?


A. Pyelonephritis
B. Diabetes mellitus
C. * Hemolytic anemia
D. Acute nephritis
E. Diabetes insipidus

663.

At what disease colour of urine is darkly brown?

A. Pyelonephritis
B. Diabetes mellitus
C. * Hemolytic anemia
D. Acute nephritis
E. Diabetes insipidus
664.

At what disease glomerular filtration is always decreased?


A. Acute nephritis
B. Chronic glomerulonephritis
C. Acute pyelonephritis
D. Amyloidosis of kidneys
E. * Acute kidneys insufficiency

665.

At what disease the excretions of calcium diminish with urine?


A. Icenko Cushing disease
B. Osteomalacia
C. Addison disease
D. * Rickets
E. Aldosteronism

666.

At what disease the excretions of calcium increase with urine?


A. Rickets
B. * Osteomalacia
C. Addison disease
D. Icenko Cushing disease
E. Aldosteronism

667.

Average creatinine clearance in an adult man is about


A. 54 ml/min
B. 75 ml/min
C. * 110 ml/min
D. 130 ml/min
E. None of above

668.
Benedicts test is less likely to give weakly positive results with
concentrated urine due to the action of
A. Urea

B. * Uric acid
C. Ammonium salts
D. Phosphates
E. All of these
669.
Benzoic acid in the organism is transformed to hypuric acid (Quick's
test). Name the compound it connects with:
A. * glycine
B. glycocol
C. valine
D. alanine
E. threonine
670.
By what method is conducted quantitative determination of glucose in
urine?
A. Felling
B. Roberts - Stolnikov
C. Gmelin and Rosenbakh
D. * Altgauzen
E. Salkovskiy
671.
By what test is conducted quantitative determination of proteins in
urine?
A. Altgauzen
B. * Roberts Stolnikov
C. Gmelin and Rosenbakh
D. Felling
E. Salkovskiy
672.
Cardiogenic shock in a patient was complicated by metabolic acidosis.
What compensatory mechanism it?
A. * Alveolar hyperventilation
B. Alveolar hypoventilation
C. Decrease ammoniagenesis in kidneys
D. Decrease of resorption of hydrogen carbonate in kidneys
E. Decrease acidogenesis in kidneys
673.

Choose the normal level of relative density of urine.


A. * 1006 - 1020

B. 1015 - 1012
C. 1011 - 1013
D. 1004 - 1010
E. 1010 - 1016
674.

Choose the substance of protein nature which is produced in kidneys:


A. Melatonin
B. * Eritropoetin
C. Epinephrine
D. Progesterone
E. Aldosterone

675.

Choose the substance of protein nature which is produced in kidneys:


A. Melatonin
B. * Erythropoietin
C. Epinephrine
D. Progesterone
E. Aldosterone

676.
Chronic insufficiency of kidneys developed in a patient with chronic
diffuse glomerulonephritis. In the terminal stage of chronic kidney
insufficiency olygo- and anuria developed. What is cause of disorders:
A. * Decrease of functioning nephron amount
B. Ischemia of kidney cortex
C. Decrease of filtration pressure
D. Increase of resorption of water in tubule
E. Spasm of afferent glomerular arteriole
677.

Chyluria can be detected by addition of the following to the urine:


A. Sulphosalicylic acid
B. Nitric acid
C. Acetic anhydride
D. * Chloroform
E. Ethanol

678.

Creatinine coefficient for male is:


A. 10-25
B. * 20-30

C. 50-100
D. 1-10
E. 2-6
679.

Cystinuria results from inability to


A. Metabolise cysteine
B. Convert cystine into cysteine
C. Incorporate cysteine into proteins
D. * Reabsorb cystine in renal tubules
E. Both C and D

680.

Cystinuria results from inability to


A. Metabolise cysteine
B. Convert cystine into cysteine
C. Incorporate cysteine into proteins
D. * Reabsorb cystine in renal tubules
E. Only A and D

681.

Daily excretion of creatinine with urine is:


A. 3-5 g
B. 4-6 g
C. 8-10 g
D. 7-9 g
E. * 1-2 g

682.

Daily excretion of creatinine with urine is:


A. 3-5 g
B. 4-6 g
C. 8-10 g
D. 7-9 g
E. * 1-2 g

683.

Daily excretion of urea with urine is:


A. 20 35 g
B. 20 35 g
C. 20 35 g
D. * 20 35 g

E. 20 35 g
684.

Daily excretion of urea with urine is:


A. 3-5 g
B. 4-6 g
C. 8-10 g
D. 7-9 g
E. * 1-2 g

685.

Daily excretion of uric acid with urine is:


A. * 0,6 1 g
B. 0,5 1,5 g
C. 2 3 g
D. 1,5 3,5 g
E. 0,1 0,5 g

686.
Determination of which products in urine is important for the
estimation of adrenal cortex function?
A. * 17-ketosteroids
B. Cholesterol
C. Cyclopentanoperhydrophenantren
D. Ketone bodies
E. Lanosterol
687.
Development of nephrotic syndrome is conditioned by immune
mechanisms. Antibodies that appear in reply to exogenous and endogenous
antigens belong to the class:
A. * Ig M or Ig G
B. Ig A or Ig M
C. Ig G or Ig A
D. Ig E or Ig G
E. Ig D or Ig E
688.

Diabetic ketoacidosis is an example of which imbalance?


A. Respiratory acidosis
B. Respiratory alkalosis
C. Metabolic alkalosis
D. * Metabolic acidosis

E. Respiratory chain
689.

Fixation of specific gravity of urine to 1.010 is found in


A. Diabetes insipidus
B. Polydypsia
C. Cystinosis
D. * Chronic glomerulonephritis
E. None of the above

690.
For a patient was found the increase of maintenance of urea and
creatinine in blood and diminishing in urine. What are possible reasons of
such state?
A. Main reason of such state it is disorder of detoxification, transport and
excretion of ammonia with urine
B. Main reason is a disease of liver
C. Such state appears at the disease of muscles
D. * Diseases which result in insufficiency of kidneys
E. Such state appears as a result of disorder of acid-base balance in an
organism
691.
For diagnostic of acute inflammatory process in kidneys was conducted
a specific test - determination of such substance in urine:
A. Content of lactose
B. Activity of creatinkinase
C. Concentrations of creatin
D. Activity of pepsin
E. * Activity of alanine amino peptidase
692.
For the patient in the preceding question, what is the most appropriate
treatment?
A. Magnesium oxide supplementation
B. * Potassium phosphate supplementation
C. Reduction of levothyroxine dose
D. Daily use of a tranquilizer
693.
For the patient in the preceding question, what is the most appropriate
treatment?
A. Magnesium oxide supplementation
B. * Potassium phosphate supplementation
C. Reduction of levothyroxine dose

D. Daily use of a tranquilizer


694.
For the patient with kidney insufficiency together with medicine
intravenous injected 500,0 ml 5 % solution of glucose. What disorder of water
balance can develop in a patient?
A. * Hypoosmotic hyperhydration
B. Hyperosmolar hyperhydration
C. Isoosmolar hyperhydration
D. Hypoosmotic dehydration
E. Change will not be
695.

For the preceding patient, what is the indicated treatment?


A. * Observation after immediate delivery
B. Plasma exchange
C. Peritoneal dialysis
D. Hemodialysis
E. Volume repletion

696.

For the preceding patient, what is the indicated treatment?


A. * Observation after immediate delivery
B. Plasma exchange
C. Peritoneal dialysis
D. Hemodialysis
E. Volume repletion

697.

For what disease is the most proper urine with low density?
A. Itchenko - Cushing
B. Addison disease
C. * Diabetes insipidus
D. Diabetes mellitus
E. Chronic pyelonephritis

698.

For what disease is the most proper urine with low density?
A. Itchenko - Cushing
B. Addison disease
C. * Diabetes insipidus
D. Diabetes mellitus
E. Chronic pyelonephritis

699.
Fresh erythrocytes are found in urine of a patient. What pathology it is
most characterized for?
A. * Nephrolithiasis
B. Acute diffuse glomerulonephritis
C. Chronic diffuse glomerulonephritis
D. Nephrotoxic syndrome
E. Acute kidney insufficiency
700.

Glomerular filtrate it is a:
A. Whole blood
B. Blood plasma with a proteins
C. * Blood plasma without any proteins
D. Blood plasma without glucose
E. Blood plasma without glucose and proteins

701.

Glomerular filtration mostly depends on


A. * Permeability of glomerular filter
B. Endothelia of capillaries
C. Level of filtration pressure
D. Hydrostatical pressure
E. Oncotic pressure

702.

Glomerular filtration mostly depends on


A. * Permeability of glomerular filter
B. Endothelia of capillaries
C. Level of filtration pressure
D. Hydrostatical pressure
E. Oncotic pressure

703.

Haematuria can occur in


A. Haemolytic anaemia
B. Mismatched blood transfusion
C. Yellow fever
D. * Stone in urinary tract
E. All of these

704.

Haematuria can occur in all of the following except

A. Acute glomerulonephritis
B. Cancer of urinary tract
C. Stone in urinary tract
D. * Mismatched blood transfusion
E. Both C and D
705.

Hormone associated with diuresis:


A. Oxytocine
B. * Vasopressin
C. Both oxytocine and vasopresin
D. Neither oxytocine nor vasopressin
E. ACTH

706.

hours and, then, measure


A. Serum urea
B. Serum creatinine
C. Urine output in one hour
D. * Specific gravity of urine
E. Both C and D

707.

How is diminishing of day's diuresis named?


A. Anuria
B. Polyuria
C. * Oliguria
D. Nicturia
E. Polakiuriya

708.

How is diminishing of day's diuresis named?


A. Anuria
B. Poliuria
C. * Oliguria
D. Nicturia
E. Polakiuriya

709.

How is frequent urination named?


A. * Polakiuria
B. Stranguria

C. Anuria
D. Nicturia
E. Polyuria
710.

How is frequent urination named?


A. * Polakiuria
B. Stranguria
C. Anuria
D. Nicturia
E. Poliuria

711.

How is the increase of acidity of urine named?


A. * Hyperaciduriya
B. Glucosuria
C. Ketonuria
D. Creatinuriya
E. Hyperacidity

712.

How is the increase of acidity of urine named?


A. * Hyperaciduriya
B. Glucosuria
C. Ketonuria
D. Creatinuriya
E. Hyperacidity

713.

How is the increase of day's diuresis named?


A. Anuria
B. Polakiuria
C. Oligouria
D. Ischuria
E. * Polyuria

714.

How is the increase of day's diuresis named?


A. Anuria
B. Polakiuria
C. Oligouria
D. Ischuria

E. * Poliuria
715.
How long do kidneys need for proceeding of imbalanced acid-base
state?
A. 30 sec. 1 minutes
B. 1 3 minutes
C. * 10 - 20 hours
D. 5 10 minutes
E. 1 - 2 hours
716.
How many ascorbic acid in normal condition is excreted per day with
urine:
A. * 20-30 mg
B. 113,5-170,5 mg
C. 10-28 g
D. 80-100 mg
E. 18-33 g
717.
How many ascorbic acid in normal condition is excreted per day with
urine:
A. * 20-30 mg
B. 113,5-170,5 mg
C. 10-28 g
D. 80-100 mg
E. 18-33 g
718.
How many grammes of potassium are contained in daily urine of
healthy adult?
A. * 2 5 g
B. 0,5 1,5 g
C. 2 3 g
D. 1,5 3,5 g
E. 0,1 0,5 g
719.
How many grammes of proteins can lose with urine patients with a
glomerulonephritis daily?
A. 100
B. 50
C. 60 -70

D. * 20 - 40
E. 120 - 150
720.

How many litters of blood do flow daily through kidneys?


A. 200 300
B. 50 150
C. * 700 900
D. 15 35
E. 100 500

721.
How much time do kidneys need for proceeding of imbalanced acidbase state?
A. 30 sec. 1 minutes
B. 1 3 minutes
C. * 10 - 20 hours
D. 5 10 minutes
E. 1 - 2 hours
722.
In a addict as a result of oppression respiratory center due to
overdosing of drugs appeared disorder acid-basic balance, namely:
A. * Respiratory acidosis
B. Metabolic acidosis
C. Respiratory alkalosis
D. Metabolic alkalosis
E. Mixed acidosis
723.
In a child a weakness, hypodynamia appeared as a result of protracted
diarrhea. What form of disorder of acid-basic balance will be in this case?
A. * Metabolic acidosis
B. Excretory acidosis
C. Metabolic alkalosis
D. Exogenous anrespiratory acidosis
E. Respiratory alkalosis
724.
In a child for 2 weeks after the carried tonsillitis developed acute
diffuse glomerulonephritis, which was characterized by oliguria, proteinuria,
hematuria, hyperazotemia, edema. What function of kidneys is desordered?
A. * Glomerular filtration
B. Tubular resorption

C. Tubular secretion
D. Depressive function
E. Incretory function
725.
In a child the foreign body of larynges. What disorders of acid-basic
balance can be observed in this case?
A. * Respiratory acidosis
B. Metabolic acidosis with the raised anionic difference
C. Metabolic alkalosis
D. Respiratory alkalosis
E. Metabolic acidosis with normal anionic difference
726.
In a man during starvation the edema on lower extremities, ascites
developed. What factor is main in pathogeny of edema in this case?
A. * Decrease of blood oncotic pressures
B. Increase of blood hydrostatic pressure
C. Increase of oncotic pressures of intercellular liquid
D. Increase of permeability of vascular wall
E. Disorder of lymphokinesis
727.
In a neonate with pylorostenosis there is the frequent vomiting. What
form of disorder of acid-basic balance will be in the child ?
A. * Not respiratory alkalosis
B. Respiratory alkalosis
C. Respiratory acidosis
D. Metabolic acidosis
E. Excretory acidosis
728.

In a norm the clearance of endogenous creatinine is:


A. * 110 150 ml/min
B. 100 250 ml/min
C. 110 200 ml/min
D. 10 150 ml/min
E. 10 100 ml/min

729.
In a patient acute glomerulonephritis the defeat basic membrane of
kidney nephrons on the allergy mechanism
A. * Immune complex type
B. Stimulating type

C. T-lymphocytic type
D. B-lymphocytic type
E. Delayed-type hypersensitivity
730.
In a patient acute kidney insufficiency with the anuria, uremia and
necrosis of renal cortical lobule kidneys developed as a result of the
pathogenic effect of unknown substance. What substance could be cause
such defeat of kidneys?
A. * Mercuric chloride
B. Ethanol
C. Vicasol
D. Penicillin
E. Casein
731.

In a patient after vomiting developed


A. * Metabolic alkalosis
B. Excretory acidosis
C. Exogenous acidosis
D. Respiratory acidosis
E. Respiratory alkalosis

732.
In a patient anuria developed with acute kidney insufficiency (day's
diuresis - 30 ml). What the basis mechanism of development in this case?
A. * Decrease glomerular filtration
B. Increase resorption of sodium
C. Violation of urine passage
D. Violation of kidney blood circulation
E. Increase resorption of water
733.
In a patient as a result of considerable blood loss (35 % volume) anuria
occur. What leading mechanism of its development in this case?
A. * Decrease of hydrostatical pressure on the wall of capillaries
B. Increase of oncotic pressures of blood
C. Increase of renal pressure
D. Decrease of functioning nephrons
E. Decrease of renal pressure
734.
In a patient diagnosed acute glomerulonephritis. What is basic
mechanism of anemia developmentat this disease

A. * Decrease of erythropoietin production


B. Decrease of glomerular filtration
C. Decrease of prostaglandins synthesis
D. Kidney azotemia
E. Kidney acidosis
735.
In a patient diagnosed chronic glomerulonephritis. What is damage
mechanisms of nephron basic membrane
A. * Autoimmune
B. Toxic
C. Degenerative
D. Hypoxic
E. Traumatic
736.
In a patient disorder of resorption of sodium ions, glucose, amino acid,
hydrogen carbonate, phosphates. For the damage of what part of nephron it
is characterized?
A. * Proximal renal tubule
B. Distal renal tubule
C. Henles loop
D. Collective tubule
E. Afferent glomerular arteriole
737.
In a patient for 2 weeks after festering tonsillitis there was acute
glomerulonephritis. Antibodies to the antigens of what microorganism is more
likely than all determined in this patient?
A. * Hemolytic streptococcus
B. Staphylococcus
C. Pneumococcus
D. Mycobacteria of tuberculosis
E. Meningococcus
738.
In a patient hemorrhagic shock was complicated by development of
acute kidney insufficiency. What main mechanism of development this
complication.
A. * Centralization of blood circulation with the development of kidneys
ischemia
B. Increase permeability of capillaries
C. Development of DIC-syndrome

D. Increase in the blood of vasopressin


E. Activation of sympathoadrenal system
739.
In a patient hemorrhagic shock was complicated by development of
acute kidney insufficiency. Name a main mechanism of this complication
development
A. * Centralization of blood circulation with development of kidney
ischemia
B. Increase of capillar permeability
C. Development of DIC-syndrome
D. Increase in the blood of vasopressin
E. Activation of sympathoadrenal systems
740.
In a patient hypoosmotic hypohydration developed due to vomiting and
diarrhea. The cause of its development is:
A. * Salts loss
B. Water loss
C. Inflammatory process
D. Polydipsia
E. Polyphagia
741.
In a patient in one and a half weeks the edema of face appeared after a
heavy streptococcus tonsillitis, arterial pressure increased. Hematuria,
proteinuria to 1,2 g/l. Antistreptococcus antibodies and decline of
complements component are exposed in the blood. In the capillary of what
structures is the most likely localized immune complexes?
A. * Glomerules
B. Henles loop
C. Proximal renal tubule
D. Pyramids
E. Distal renal tubule
742.
In a patient is chronic cardiac insufficiency with the edema of soft
tissues. What from the pathogenetic factors of edema is main in this case?
A. * Increase of hydrostatical pressure in capillaries
B. Decline of osmotic pressure in blood plasma
C. Increase of oncotic pressure in tissues
D. Increase of permeability of capillaries wall
E. Increase of osmotic pressure in tissues

743.
In a patient is chronic kidney insufficiency. What mass of functioning
nephron likely in these kidneys
A. * 10-30 %
B. 5-10 %
C. 30-50 %
D. 50-70 %
E. 70-90 %
744.
In a patient is determined in urine sugar. The table of glucose contents
in a blood corresponds to the norm. What is mechanism of glucosuria in this
case?
A. * Disorder of glucose resorption in nephron tubule
B. Insulin insufficiency
C. Hyperfunction of adrenals
D. Hyperfunction of thyroid gland
E. Hyperfunction of adrenal cortex
745.
In a patient is edema, at the examination proteinuria, arterial
hypertensionis, hypoproteinemia, hyperlipemia occur. How this syndrome is
named?
A. * Nephrotic
B. Anemic
C. Hypertensive
D. Thyrotoxic
E. Hypothyroid
746.
In a patient is general weakness, pain in the area of kidneys, edema of
face. In research of urine expressed proteinuria, hematuria, cylindruria was
revealed. What from enumerated is the leading pathogenetic mechanism of
edema development?
A. * Decrease of blood oncotic pressures
B. Increase of vascular permeability
C. Increase of hydrodynamic pressure of blood
D. Disorder of hormonal balance
E. Disorder of lymphokinesis
747.
In a patient is substantial decrease of kidney ability to osmotic
concentration of urine. How this disorder is named?
A. * Isohyposthenuria
B. Proteinuria

C. Hematuria
D. Cylinderuria
E. Leukocyturia
748.
In a patient occur disorders of sodium ions , glucose, amino acid,
hydrogen carbonate, phosphates resorption. For the damage of what part of
nephron it is characterized?
A. * Proximal tubule
B. Distal tubule
C. Henles loop
D. Collecting tubule
E. Connective segment
749.
In a patient pain in a lumbar part , sickly and frequent urination,
increased temperature of body to 39 C. In laboratory research of blood leukocytosis and ESR acceleration occur, at research of urine leukocyturia,
proteinuria, bakteriauria. What disease the most likely in this case?
A. * Pyelonephritis
B. Adnexitis
C. Urolithiasis
D. Glomerulonephritis
E. Radiculitis
750.
In a patient proteinuria (4,5 g/l), hematuria with the lixiviated
erythrocytes occur. What function of kidneys is disordered?
A. * Disorder of nephron permeability
B. Extrarenal disorders
C. Disorder of tubular resorption
D. Disorder of nephron secretion
E. Disorder of tubular secretion
751.
In a patient the edema of face appeared after a streptococcus
tonsillitis, arterial pressure increased. In the analysis of urine - hematuria and
proteinuria. Antistreptococcus antibodies and decrease of blood complement
components occur in blood. What part of kidney the most likely localization of
immune complexes in?
A. * Glomerules
B. Henles loop
C. Proximal part of tubule
D. Pyramids

E. Distal part of tubule


752.
In a patient the edema of lips, face, itch in the place of bite, pain
appeared after the bite of bee. Development of edema in this patient is
conditioned
A. * Increase of capillar permeability
B. Increase of osmotic pressure in tissue
C. Increase of oncotic pressures in tissue
D. Increase of hydrostatic pressure in a vessel
E. Disorder of lymphodynemic
753.
In a patient traumatic shock developed and the signs of acute kidney
insufficiency appeared. What is the main mechanism of its development in
this case?
A. * Decrease arterial pressure
B. Violation of urine passage
C. Increase of pressure in the nephron capsule
D. Increase of pressure in the kidney arteries
E. Decrease of blood oncotic pressure
754.
In a patient violation of respiratory ways. What changes of acid-basic
balance in the patient?
A. * Respiratory acidosis
B. Respiratory alkalosis
C. Metabolic acidosis
D. Metabolic alkalosis
E. The changes absent
755.
In a patient with acute cardiac insufficiency was revealed oliguria. What
is cause of this phenomenon?
A. * Decrease of glomerular filtration
B. Decrease of tubular resorption
C. Decrease of tubular secretion
D. Increase of hydrostatical pressure on the wall of capillaries
E. Decrease of hematocrit
756.
In a patient with acute kidney insufficiency polyuria occur. Increase of
diuresis at the 2 stage of acute kidney insufficiency due to
A. * Reinstitution of filtration in nephrons
B. Increase of blood circulation

C. Increase of natriuretic hormone in plasma


D. Decrease of aldosterone in plasma
E. Decrease of vasopressin in plasma
757.
In a patient with acute kidney insufficiency the amount of excreted
urine is 200 ml. How such change of diuresis is named?
A. * Olyguria
B. Anuria
C. Proteinuria
D. Polyuria
E. Leukocyturia
758.
In a patient with cardiac insufficiency there is the edema of lower
extremities. The leading chain of pathogeny is
A. * Increase of hydrostatic pressure in capillaries
B. Decrease of oncotic pressures in capillaries
C. Increase of catecholamines level
D. Disorder of lymphokinesis
E. Positive water balance
759.
In a patient with chronic cardiac insufficiency the increase of
hydrostatical pressure in inferior vena cava occur, that caused development:
A. * Cardiac edema
B. Hepatic edema
C. Renal edema
D. Lymphatic edema
E. Toxic edema
760.
In a patient with chronic glomerulonephritis at research of urine
proteinuria, hematuria, leukocyturia occur. What disorders of kidney function
stipulated proteinuria by?
A. * Disorder of glomerular filtration
B. Disorder of tubular secretions
C. Disorder of tubular resorption
D. Disorder of erythropoietin synthesis
E. Disorder of capillaries permeability
761.
In a patient with chronic glomerulonephritis normochromal anemia
developed. The mechanism of its development is related to

A. * Depression of erythropoietin secretion


B. Increase of erythropoietin secretion
C. Activation of juxtaglomerular apparatus
D. Streptococcus intoxication
E. Damaging action of immune complexes
762.
In a patient with chronic kidney insufficiency appeared anorexia,
dyspepsia, violation of hearts rhythm, itch of skin. What mechanism of
development of these violations is main?
A. * Accumulation of products of nitrogenous metabolism
B. Disorder of lipid metabolism
C. Change of carbohydrate metabolism
D. Kidney acidosis
E. Disorder of water-electrolyte metabolism
763.
In a patient with chronic kidney insufficiency the edema developed. In
laboratory research of blood anemia , hypoproteinemia, dysproteinemia,
increase of creatinin, nitrogen, decline level of glomerular filtration occur.
What mechanisms the most likely in development of edema in this case?
A. * Hypoproteinemia
B. Dysproteinemia
C. Anemia
D. Accumulation of nitrogenous compound
E. Decrease of glomerular filtration rate
764.
In a patient with chronic kidney insufficiency was revealed:
osteoporosis, pathological tissue calcification. With strengthening action of
what hormone are related these disorders?
A. * Parathormone
B. Thyroxin
C. Triiodothyronine
D. Calcitonin
E. Adrenaline
765.
In a patient with chronic pathology is observed isohyposthenuria. What
changes of urine passage will be observed ?
A. * Polyuria
B. Pyuria
C. Anuria

D. Pollakiuria
E. Hematuria
766.
In a patient with complaints of increased temperature, pain in area of
kidneys, frequent and sickly urination, acute pyelonephritis is diagnosed.
What factor is cause of illness?
A. * Colon bacillus
B. Staphylococcus
C. Streptococci
D. Brucelus
E. Anaerobic flora
767.
In a patient with diabetes mellitus as a result of accumulation ?oxybutteric and acetoacetic acids there was disordered of acid-basic balance
which is named
A. * Metabolic acidosis
B. Metabolic alkalosis
C. Respiratory acidosis
D. Mixed acidosis
E. Not respiratory alkalosis
768.
as

In humans the sulphur of methionine and cysteine is excreted mainly


A. Ethereal sulphate
B. * Inorganic sulphate
C. Sulphites
D. Thioorganic compound
E. All of these

769.
In the urine of a patient with liver disease is absent urobilinogen. It is
connected with the affection of:
A. stercobilin formation
B. * direct bilirubin formation
C. kidneys function
D. transformation of bilirubin in the intestine
E. bile passage to the intestine
770.

In what part of nephron does take place filtration of urine?


A. * In a glomerulus

B. In the loop of Henle


C. In a proksimal canaliculi
D. In a distal canaliculi
E. In the collective tubes
771.

Is there what normal concentration of urea in blood?


A. 3 - 8 gramme/l
B. 1 - 3 mmol/l
C. * 3 - 8 mmol/l
D. 3 - 10 mmol/l
E. 10 - 15 mmol/l

772.
Ketoacidosis refers to the metabolic acidosis caused by the
overproduction of ketoacids. The classic form of ketoacidosis is uncontrolled
diabetes mellitus. All of the following would occur in a patient suffering from
a diabetes-induced metabolic acidosis (diabetic ketoacidosis) EXCEPT:
A. Deep, rapid breathing causes a decrease in the partial pressure of
carbon dioxide (PaCO2) in the blood.
B. The removal of carbon dioxide through deep, rapid breathing results in
a rise in the blood pH
C. When endogenous acid production rises sharply, net acid excretion
cannot keep pace and the bicarbonate lost in buffering is not replaced
causing plasma HCO3- levels to fall
D. * Serum bicarbonate levels will decrease after the administration of an
insulin injection
E. The fall in insulin causes fat cells to liberate fatty acids, which flood the
hepatocytes
773.
Laboratory test of a patients urine with intestines dysbacteriosis
established the increase of indican. It testifies:
A. kidney disease
B. * normal neutralization liver function
C. increased fat hydrolysis
D. liver malfunction
E. vitamins F hypovitaminosis
774.

Maple syrup urine disease is the disorder of the:


A. tyrosine metabolism caused by the absence of homogentisate oxidase
B. phenylalanine metabolism caused by an absence or deficiency of
phenylalanine hydroxylase

C. * oxidative decarboxylation of o-ketoacids derived from valine,


isoleucine, and leucine caused by the defect of branched-chain
dehydrogenase
D. glutamate metabolism caused by the defect of glutamate
dehydrogenase
E. None of the above
775.

Maple syrup urine diseases is an inborn error of metabolism of


A. Sulphur-containing amino acids
B. Aromatic amino acids
C. * Branched chain amino acids
D. Dicarboxylic amino acids
E. B and C

776.

Maple syrup urine diseases is an inborn error of metabolism of


A. Sulphur-containing amino acids
B. Aromatic amino acids
C. * Branched chain amino acids
D. Dicarboxylic amino acids
E. None of the above

777.

Maximum urea clearance is


A. 54 ml/min
B. * 75 ml/min
C. 110 ml/min
D. 130 ml/min
E. Both C and D

778.

Mineralocorticoids increase the tubular reabsorption of


A. Sodium and calcium
B. Sodium and potassium
C. * Sodium and chloride
D. Potassium and chloride
E. Potassium and calcium

779.
Postoperatively, the preceding patient develops a fever, and pyuria is
noted. Laboratory studies show a leukocyte count of 12,000/?L, a blood urea
nitrogen level of 24 mg/dL, and a serum creatinine concentration of 1.4
mg/dL. The urine culture grows Escherichia coli, and treatment with
trimethoprim-sulfamethoxazole is started. Three days later, the pyuria and

fever have resolved. The leukocyte count is 10,000/?L, blood urea nitrogen
level is 24, serum creatinine is 1.8 mg/dL. Urinalysis shows no leukocytes in
high-power fields. What is the most likely explanation for the elevated serum
creatinine concentration?
A. Acute interstitial nephritis
B. Acute pyelonephritis
C. Obstructive uropathy
D. * Reduced creatinine excretion
E. Acute tubular necrosis
780.
Postoperatively, the preceding patient develops a fever, and pyuria is
noted. Laboratory studies show a leukocyte count of 12,000/?L, a blood urea
nitrogen level of 24 mg/dL, and a serum creatinine concentration of 1.4
mg/dL. The urine culture grows Escherichia coli, and treatment with
trimethoprim-sulfamethoxazole is started. Three days later, the pyuria and
fever have resolved. The leukocyte count is 10,000/?L, blood urea nitrogen
level is 24, serum creatinine is 1.8 mg/dL. Urinalysis shows no leukocytes in
high-power fields. What is the most likely explanation for the elevated serum
creatinine concentration?
A. Acute interstitial nephritis
B. Acute pyelonephritis
C. Obstructive uropathy
D. * Reduced creatinine excretion
E. Acute tubular necrosis
781.
Postoperatively, the preceding patient develops a fever, and pyuria is
noted. Laboratory studies show a leukocyte count of 12,000/?L, a blood urea
nitrogen level of 24 mg/dL, and a serum creatinine concentration of 1.4
mg/dL. The urine culture grows Escherichia coli, and treatment with
trimethoprim-sulfamethoxazole is started. Three days later, the pyuria and
fever have resolved. The leukocyte count is 10,000/?L, blood urea nitrogen
level is 24, serum creatinine is 1.8 mg/dL. Urinalysis shows no leukocytes in
high-power fields. What is the most likely explanation for the elevated serum
creatinine concentration?
A. Acute interstitial nephritis
B. Acute pyelonephritis
C. Obstructive uropathy
D. * Reduced creatinine excretion
E. Acute tubular necrosis
782.
Resorption of sodium ions in kidney tubule occur in the use of salt
water. What compensatory changes of hormones secretion will arise in that?
A. * Decrease of aldosterone excretion
B. Decrease of vasopressin excretion

C. Increase of aldosterone excretion


D. Decrease of natriuretic hormone excretion
E. Increase of vasopressin excretion
783.

Respiratory acidosis is developed at:


A. * Hypoventilation of lungs
B. Hyperventilation of lungs
C. Diabetes mellitus
D. Decrease of partial pressure of oxygen in air
E. Starvation

784.

Respiratory alkalosis is developed at:


A. * Hyperventilation of lungs
B. Hypoventilation of lungs
C. Diabetes mellitus
D. Gastric juices loss
E. Starvation

785.
Sinus tachycardia and extrasystole in a patient was caused by
diuretics. What from enumerateded is the most likely cause of this
complication?
A. * Hypokalemia
B. Hyperkalemia
C. Hypocalcemia
D. Hyponatremia
E. Hypovolemia
786.
Syndrome of Lesch-Nyhan hard hyperuricemia is the genetic
deficiency of such enzyme:
A. Xantine oxidase
B. Adenosine deaminase
C. Hypoxanthine oxidase
D. Phosphorylase
E. * Hypoxanthine guanine phosphoribosyltransferase
787.
The active form of vitamin D (1,25-dihydrocholecalciferol) maintain in
an organism a constant level of:
A. Potassium and phosphorus
B. * Calcium and phosphorus

C. S. Iron and calcium


D. Iron and magnesium
E. Magnesium and manganese
788.

The amino acid which detoxicated benzoic acid to form hippuric acid is
A. * Glycine
B. Alanine
C. Serine
D. Glutamic acid
E. Tyrosine

789.
The amount of urea in patient's urine is normal. Which of the listed
indexes are true in this case?
A. 10-15 g
B. 20-40 g
C. 10-35 g
D. * 25-30 g
E. 50-60 g
790.
The changes in kidneys at the glomerulonephritis as a result of damage
basic membrane
A. * Antibodies and immune complexes
B. Bacterium
C. Bacterial toxins
D. Biological active substance
E. Products of nitrogen metabolism
791.
The concentration of what substance does increase in the blood at
disorders of excretory function of kidneys?
A. Uric acid and creatinine
B. * Creatinine and urea
C. Creatinine and creatin
D. Creatinine and indican
E. Uric acid and indican
792.

The concentration of which substance is increased at an alkaptonuria?


A. Leukotrienes
B. Phenyl-pyruvic acid

C. Melatonin
D. * Gomogentisic acid
E. Erythropoietin
793.

The constant increase of uric acid concentration in blood is named:


A. Uremia
B. Hyperazotemiya
C. Hyperacidaminemiya
D. Hyperlaktatemiya
E. *Hyperuricemia

794.
The excretion of chlorides with urine decreases at all of the followings
states, except:
A. Vomiting
B. Diabetes insipidus
C. Diarrhea
D. * Addison disease
E. Icenko Cushing disease
795.
The hyperthermia causes the considerable increase of breathing rate.
What type of water-electrolyte balance disorder arose in that case?
A. * Dehydration hyperosmolar
B. Dehydration hypoosmolar
C. Dehydration isoosmolar
D. Hyperhydration isoosmolar
E. Hyperhydration hyperosmolar
796.
The indexes of acid-basic balance in plasma of a patient blood are: pH
7,33, concentration 3 15 mmol/l (norm 21-25 mmol/l); 2 49
mm Hg (norm 35-45 mm Hg). He has
A. * Mixed acidosis
B. Decompensated metabolic acidosis
C. Decompensated respiratory acidosis
D. Compensated metabolic acidosis
E. Metabolic and respiratory y alkalosis
797.
The insufficient production of mineralocorticoids (Addison diseases,
bronze diseases) is accompanied with muscle weakness. This is explained by
the excretion with urine increased amount of ions of:

A. * Na+
B. K+
C. H+
D. Ca2+
E. Mg2+
798.

The kidney threshold of glucose is:


A. * 8 10 mmol/l
B. 5 10 mmol/l
C. 1 5 mmol/l
D. 1 2 mmol/l
E. 10 15 mmol/l

799.

The kidney threshold of glucose is:


A. * 8 9 mmol/l
B. 5 10 mmol/l
C. 1 5 mmol/l
D. 1 2 mmol/l
E. 10 15 mmol/l

800.

The level of phenyl-pyruvic acid in urine increases at:


A. Osteomalacia
B. * Phenylketonuria
C. Addison disease
D. Icenko Cushing disease
E. Aldosteronism

801.
The lixiviated erythrocytes are found in day's urine of patient. For what
pathology of kidneys the most characterized this symptom?
A. * Diffuse glomerulonephritis
B. Urethritis
C. Nephrolithiasis
D. Acute pyelonephritis
E. Chronic pyelonephritis
802.

The main cause of chronic glomerulonephritis is


A. * Damage basic membrane of nephron

B. Hypoxic damage of tubule


C. Degenerative damage membrane of nephrons
D. Toxic damage of tubule
E. Microbial damage of tubule
803.

The main pigment of urine is:


A. Bilirubin
B. Uroeritrin
C. Uroporfirin
D. Urobilinoid
E. * Urochrome

804.

The man's daily secretion of creatinine with urine is:


A. 3-5 g
B. 4-6 g
C. 8-10 g
D. 7-9 g
E. * 1-2 g

805.
The massive crush of soft tissues caused appearance in the patient of
oliguria, hyposthenuria, proteinuria, myoglobinuria, hyperkalemia,
hyponatremia. What main mechanism of kidney function disorder in this
case?
A. * Development of toxemia
B. Painful stimulation
C. Stimulation of sympathetic nervous system
D. Catecholamine release
E. Protein loss
806.

The most dangerous for life such effect of acute kidney insufficiency:
A. Overhydratation
B. Hypokaliemia
C. Hypermagniyemia
D. * Hyperkaliemia
E. Hyperkal'ciemia

807.
The nephrotoxic serum of guinea-pig was entered to the rabbit. What
disease of man was designed in this experience?
A. * Diffuse glomerulonephritis

B. Nephrotoxic syndrome
C. Acute pyelonephritis
D. Chronic kidney insufficiency
E. Chronic pyelonephritis
808.
The normal organic components of urine are all of the followings,
except:
A. Urea
B. Hyaluronic acid
C. * Glucose
D. Uric acid
E. Creatinine
809.

The pathological components of urine are all of the followings, except:


A. Ketone bodies
B. Blood
C. * Amino acid
D. Bile pigments
E. Glucose

810.
The polysaccharide used in assessing the glomerular fittration rate
(GFR) is
A. Glycogen
B. Agar
C. * Inulin
D. Hyaluronic acid
E. All of these
811.

The possible reason of uric acid excretion decrease:


A. Action of ionizing an radiation
B. * Kidney insufficiency
C. Leucosis
D. Burns
E. Malignant tumor

812.
The preceding patient is given intravenous infusion of 0.9% normal
saline at 200 mLIh. Two days later, his flank pain worsens dramatically, but
nausea and vomiting have resolved. Blood pressure and pulse rate are
unchanged. Laboratory studies: Blood urea nitrogen 8 mg/dL Serum

creatinine 0.9 mg/dL Serum potassium 4.0 meq/L Serum chloride 105 meq/L
Serum bicarbonate 22 meq/L Arterial blood gases pH 7.48, PCO2 30 mm Hg
What is the best therapy to resolve the alkalemia?
A. Increase infusion of 0.9% normal saline to 300 mL/h
B. * Control the flank pain
C. Start treatment with acetazolamide, 125mg twice daily
D. Administer lactated Ringers solution at 200 mL/h
813.
The protracted starvation of experimental animals results in
development of edema. What possible mechanism of this process?
A. * Decrease of oncotic pressures plasma of blood
B. Decrease of sodium resorption in kidneys
C. Increase of filtration pressure in the capillaries of tissues
D. Increase of renin secretion
E. Decrease of albumen resorption in tubule
814.

The specific gravity of urine normally ranges from


A. 0.9000.999
B. * 1.0031.030
C. 1.0001.001
D. 1.1011.120
E. 1.999 2.111

815.
There is disorder of excretory kidneys function - oligo-anuria in the
stage of acute kidney insufficiency. Specify the main characterized index of
this phenomenon?
A. * Azotemia
B. Decrease of hematocrit
C. Decrease of arterial pressure
D. Hypokalemia
E. Hyponatremia
816.
There is proteinuria as a result of kidney illness. What the most likely
mechanism of this phenomenon?
A. * Damage of glomerular membranes
B. Increase of renin secretion
C. Increase of muscles tone
D. Increase of protein synthesis in a liver

E. Decrease of vasopressin secretion


817.
Two weeks after therapy is initiated in the preceding patient, her blood
pressure decreases from 150/90 mm Hg to 128/80mm Hg, and she feels well.
Repeated laboratory testing reveals an increase in serum creatinine
concentration from 1.9 mg/dL to 2.1 mg/dL. The potassium concentration is
4.2 mgld L. Which of the following is the most appropriate course of action?
A. Discontinue antihypertensive therapy
B. Perform noninvasive screening for possible renal artery stenosis
C. Perform renal angiography
D. * Continue antihypertensive therapy and monitor kidney function
818.
Two years ago in a patient was diagnosed chronic glomerulonephritis.
The edema appeared for the last 5 months. What changes underlie of it
development?
A. * Proteinuria
B. Disorder of hepatic function
C. Hyperaldosteronism
D. Hyperosmolarity plasma
E. Increase of vasopressin production
819.
Urinary potassium 20 meq/L Arterial blood gases pH 7.61, PCO2 36 mm
Hg What is this patients acid-base disorder?
A. Metabolic alkalosis
B. Respiratory alkalosis
C. * Metabolic and respiratory alkalosis
D. Metabolic and respiratory alkalosis, with hidden metabolic acidosis
E. Recognize a mixed acid-base disorder.
820.
Urinary potassium 20 meq/L Arterial blood gases pH 7.61, PCO2 36 mm
Hg What is this patients acid-base disorder?
A. Metabolic alkalosis
B. Respiratory alkalosis
C. * Metabolic and respiratory alkalosis
D. Metabolic and respiratory alkalosis, with hidden metabolic acidosis
E. Recognize a mixed acid-base disorder.
821.
Venous stagnation, increase of venous pressure and as a result
strengthening filtration plasma of blood in capillaries is characterized for:
A. * Cardiac edema
B. Toxic edema

C. Allergic edema
D. Cachectic edema
E. Kidney edema
822.
Weight of the patient - 56 kg, hematocrit - 0,55 , concentration of
sodium in the blood - 152 mmol/l (norm 135-145 mmol/l), potassium - 5,9
mmol/l, hemoglobin - 100 g/l. What from the resulted indexes testifies of
hyperosmolar dehydration?
A. * Concentration of sodium
B. Hematocrit
C. Concentration of potassium
D. Weight of body
E. Level of hemoglobin
823.
Weight of the patient - 69 kg, hematocrit - 0,59 , concentration of
sodium in the blood -142 mmol/l (norm 135-145 mmol/l), to potassium - 5,0
mmol/l. What type of water-salt balance disorder in the man?
A. * Isoosmolar dehydration
B. Hypoosmolar dehydration
C. Hypoosmolar hyperhydration
D. Hyperosmolar hyperhydration
E. Hyperosmolar dehydration
824.
Weight of the patient -56 kg, hematocrit - 0,55 , concentration of
sodium in the blood -142 mmol/l, potassium - 3,9 mmol/l, hemoglobin - 100
g/l. What from the resulted indexes suggests an idea about dehydration?
A. * Hematocrit
B. Concentration of sodium
C. Concentration of potassium
D. Weight of body
E. Hemoglobin
825.
Weight of the patient -65 kg, hematocrit - 0,59 , concentration of
sodium in blood -150 mmol/l (norm 135-145 mmol/l), potassium - 6,0 mmol/l.
What type of water-salt balance disorder in a man?
A. * Hyperosmolar dehydration
B. Hypoosmolar dehydration
C. Hypoosmolar hyperhydration
D. Hyperosmolar hyperhydration
E. Isoosmolar dehydration

826.
Weight of the patient -75 kg, hematocrit - 0,58 , concentration of
sodium in the blood -130 mmol/l (norm 135-145 mmol/l), potassium - 3,0
mmol/l. What type of water-salt balance disorder in a man?
A. * Hypoosmolar dehydration
B. Hyperosmolar dehydration
C. Hypoosmolar hyperhydration
D. Hyperosmolar hyperhydration
E. Isoosmolar dehydration
827.
What disorder of water-electrolyte metabolism the most probably
arises at burn disease?
A. * Isoosmolar dehydration
B. Hypoosmolar dehydration
C. Hyperosmolar dehydration
D. Hyperosmolar hyperhydration
E. Hypoosmolar hypohydration
828.
What form disorder of acid-basic balance develops in a patient with
kidney insufficiency?
A. * Renal azotemic acidosis
B. Respiratory acidosis
C. Metabolic alkalosis
D. Metabolic acidosis
E. Respiratory alkalosis
829.
What from disorders of water-salt balance in a human will develop in
use of salt water?
A. * Hyperosmolar hyperhydration
B. Isoosmolar hyperhydration
C. Hyposmolar hyperhydration
D. Hyperosmolar dehydration
E. Hypoosmolar dehydration
830.
What from pathogenetic factors is main in the mechanism of edema
development at a nephrotic syndrome?
A. * Decrease of oncotic pressures blood plasma
B. Increase of capillar permeability
C. Increase of hydrostatic pressure in capillaries

D. Increase of osmotic pressure in tissues


E. Increase of oncotic pressures in tissues
831.
What is the basic factor in pathogenesis of edema at an acute
nephritis?
A. * Decline of glomerular filtration
B. Increase of permeability of capillaries
C. Decline of secretion of ADH
D. Decline of sodium excretion
E. Secondary hyperaldosteronism
832.

What is the cause of primary enzymo-pathologies?


A. Liver diseases.
B. * Genetic disorders.
C. Trauma.
D. Ischemia.
E. All of these

833.

What is the main reason of proteinuria?


A. * Increase of permeability of filtration barrier of kidney
B. Decline of permeability of filtration barrier of kidney
C. Increase of osmolality of primary urine
D. Increase of osmolality of the secondary urine
E. Decrease of osmolality of primary urine

834.

What is the main reason of proteinuria?


A. * Increase of permeability of filtration barrier of kidney
B. Decline of permeability of filtration barrier of kidney
C. Increase of osmolality of primary urine
D. Increase of osmolality of the secondary urine
E. Decrease of osmolality of primary urine

835.
What is the most appropriate initial treatment for a patient with a
nonobstructing radiolucent stone in the right renal pelvis?
A. Hydrochlorothiazide
B. * Low-sodium diet
C. Allopurinol
D. Oral sodium bicarbonate or potassium citrate

E. Extracorporeal shock-wave lithotripsy


836.
What is the most appropriate initial treatment for a patient with a
nonobstructing radiolucent stone in the right renal pelvis?
A. Hydrochlorothiazide
B. * Low-sodium diet
C. Allopurinol
D. Oral sodium bicarbonate or potassium citrate
E. Extracorporeal shock-wave lithotripsy
837.
What is the most appropriate initial treatment for a patient with a
nonobstructing radiolucent stone in the right renal pelvis?
A. Hydrochlorothiazide
B. * Low-sodium diet
C. Allopurinol
D. Oral sodium bicarbonate or potassium citrate
E. Extracorporeal shock-wave lithotripsy
838.
What is the most appropriate initial treatment for a patient with a
nonobstructing radiolucent stone in the right renal pelvis?
A. Hydrochlorothiazide
B. * Low-sodium diet
C. Allopurinol
D. Oral sodium bicarbonate or potassium citrate
E. Extracorporeal shock-wave lithotripsy
839.

What is the most appropriate management plan?


A. Topical acyclovir ointment
B. Oral acyclovir, 200 mg five times daily
C. Oral acyclovir, 800 mg five times daily
D. Oral famciclovir, 500 mg three times daily
E. * Intravenous acyclovir

840.

What is the most appropriate management plan?


A. Topical acyclovir ointment
B. Oral acyclovir, 200 mg five times daily
C. Oral acyclovir, 800 mg five times daily
D. Oral famciclovir, 500 mg three times daily

E. * Intravenous acyclovir
841.
What is the most important next step in confirming the diagnosis of the
decreased renal function in the preceding patient?
A. * Serum and urine protein immunoelectrophoresis
B. Measurement of circulating 25-hydroxycholecalciferol level
C. Measurement of angiotensin-converting enzyme level
D. Measurement of N-terminal parathyroid hormone level
E. Urine toxicology screen
842.
What is the most important next step in confirming the diagnosis of the
decreased renal function in the preceding patient?
A. * Serum and urine protein immunoelectrophoresis
B. Measurement of circulating 25-hydroxycholecalciferol level
C. Measurement of angiotensin-converting enzyme level
D. Measurement of N-terminal parathyroid hormone level
E. Urine toxicology screen
843.

What is the normal ratio between a day and night diuresis?


A. * 3 : 1
B. 1 : 1
C. 9 : 1
D. 6 : 1
E. 7 : 1

844.

What is the simplest method for determination of urine pH:


A. * With the litmus paper
B. With biuret reaction
C. The Folls reaction
D. By the method of Altgauzen
E. With the ureometer

845.

What is used for determination of urines density?


A. PhEC
B. Densitometer
C. * Urometer
D. Spectrophotometer
E. No any correct answer

846.

What level of urines density at oliguriya?


A. Close to low
B. Low
C. Normal
D. * High
E. Close to normal

847.

What level of urines density at oliguriya?


A. Close to low
B. Low
C. Normal
D. * High
E. Close to normal

848.

What level of urines density at poliuriya?


A. Normal
B. High
C. * Low
D. Close to low
E. Close to normal

849.

What level of urines density at polyuria?


A. Normal
B. High
C. * Low
D. Close to low
E. Close to normal

850.
What method of research is characterized concentration property of
kidneys?
A. Creatinine of blood
B. * Zimnitskiys test
C. Glomerular filtration
D. Electrolytes of blood
E. All of the above
851.
What origin albumen,the most likely at selective proteinuria of intensity
11 g/day?

A. * Tubular
B. Suprarenal
C. Glomerular
D. Urethral
E. From an urinary bladder
852.
?What pathology is developed in the absence of phenylalanine 4monooxygenase?
A. * Phenylketonuria.
B. Alkaptonuria
C. Galactosemia.
D. Hyperglycemia.
E. Achilia
853.

What quantitative changes of diuresis result in uremia:


A. * Anuria
B. Dysuria
C. Nycturia
D. Isosthenuria
E. Polyuria

854.

What substances are precursors of creatin?


A. Creatinine, glycine, arginine
B. Urinary acid, urea
C. Indican, organic acids
D. Hyaluronic acid, methionine
E. * Glycine, arginine, methionine

855.
What type disorder of general blood volume will develop in a patient at
the decrease of kidney excretory function?
A. * Oligocythemic hypervolemia
B. Polycythemic hypovolemia
C. Oligocythemic hypovolemia
D. Polycythemic hypervolemia
E. Simple hypervolemia
856.
What type of edema can develop at the starvation in the stage of
disintegration and utilization of own albumens?

A. * Cachectic
B. Inflammatory
C. Allergic
D. Toxic
E. Lymphogenous
857.
What type of water-mineral metabolism disorder will develop in a
patient with the hypophysial form of diabetes mellitus?
A. * Dehydration hyperosmolar
B. Dehydration hypoosmolar
C. Dehydration isoosmolar
D. Hyperhydration hypoosmolar
E. Hyperhydration hyperosmolar
858.
Which hormone increases sodium and water reabsorption by renal
tubule cells:
A. A.Oxytocine
B. Aldosterone
C. Vasopressin
D. Prolactin
E. Cortisol
859.

Which hormone intensify reabsorption of sodium in renal tubules?


A. glucagon
B. insulin
C. foliculin
D. * aldosterone
E. vasopressin

860.

Which hormone intensify reabsorption of sodium in renal tubules?


A. glucagon
B. insulin
C. foliculin
D. * aldosterone
E. vasopressin

861.

Which hormone intensify reabsorption of sodium in renal tubules?


A. glucagon

B. insulin
C. foliculin
D. * aldosterone
E. vasopressin
862.

Which hormone intensify reabsorption of sodium in renal tubules?


A. glucagon
B. insulin
C. foliculin
D. * aldosterone
E. vasopressin

863.

Which hormone regulates the excretion of chlorides with urine?


A. * Aldosterone
B. Insulin
C. Testosteron
D. Corticotropin
E. Cortisol

864.

Which of the following statements about microalbuminuria is true?


A. Microalbuminuria is a predictor of cardiovascular risk only in patients
with diabetes
B. Microalbuminuria is present when the spot urine albumin-tocreatinine ratio is greater than 500 mg/g
C. * Microalbuminuria is a cardiovascular risk factor independent of
traditional Framingham risk factors
D. To be of clinical value, microalbuminuria must be measured in a timed
12- to 24-hour sample

865.

Which of the following statements about microalbuminuria is true?


A. Microalbuminuria is a predictor of cardiovascular risk only in patients
with diabetes
B. Microalbuminuria is present when the spot urine albumin-tocreatinine ratio is greater than 500 mg/g
C. * Microalbuminuria is a cardiovascular risk factor independent of
traditional Framingham risk factors
D. To be of clinical value, microalbuminuria must be measured in a timed
12- to 24-hour sample

866.

Why is hypomagnesemia associated with hypocalcemia?

A. Hypomagnesemia causes a shift of calcium into bone


B. * Hypomagnesemia inhibits the secretion and action of parathyroid
hormone
C. Hypomagnesemia causes renal calcium wasting
D. Hypomagnesemia impairs the peripheral actions of vitamin D
867.

Why is hypomagnesemia associated with hypocalcemia?


A. Hypomagnesemia causes a shift of calcium into bone
B. * Hypomagnesemia inhibits the secretion and action of parathyroid
hormone
C. Hypomagnesemia causes renal calcium wasting
D. Hypomagnesemia impairs the peripheral actions of vitamin D

868.
You see the preceding patient again in your office 4 months later. He
read in the newspaper about a medicine that prevents diabetic kidney
disease and stroke. Urinalysis by dipstick is negative for protein. What is the
best test to evaluate the patients risk for diabetic nephropathy and
cardiovascular disease?
A. * Microalbumin-to-creatinine ratio
B. Serum protein electrophoresis
C. Urine amino acid levels
D. 24-hour urine total protein
E. Annual urine dipstick analysis

Situation task
1.
54-year-old man status prostaortic value replacement complains of
fatigue, palpitations, tachypnea on exertion, insomnia. On examination,
jaundice is present. Which type of anemia is in this patient?
A. Iron deficiency anemia
B. Megaloblastic anemia
C. Hemolytic anemia
D. Folic acid deficiency anemia
E. * B12 vitamin deficiency anemia
2.
A 46 years old woman complains of weakness, malaise, anorexia,
fever, dental bleeding. Spleen, liver, and lymphatic nodes are enlarged,
petechiae on a skin. Laboratory findings: the platelet count is 90,000/?L, the
white count 100,000/?L
A. * Acute leukemia

B. Megaloblastic anemia
C. Hemolytic anemia
D. Thrombocytopenia
E. Chronic leukemia
3.
A man of 55 years old has a complaints for abdominal discomfort, gum
bleeding, large ecchymoses after trauma, weakness, sternal tenderness,
fever, skin nodules.Laboratory Findings: the white cell count 540,000/?L,
basophils, eosinophils and platelets are increased; and a few normoblasts are
seen; Er 3,1 1012/l, blast 40 %. What is the most probable diagnosis?
A. * Acute leukemia
B. Megaloblastic anemia
C. Hemolytic anemia
D. Thrombocytopenia
E. Chronic leukemia
4.
A 15-year-old girl complains of fatigue, palpitations, tachypnea on
exertion, insomnia, she likes eat coal. Examinations reveals pallor of skin and
mucous membranes, nail cracking ,tachycardia, soft systolic murmur on the
apex. Peripheral blood: Erythrocytes-3,0 1012/l, - 80 g/l, CI -0,8,
erythrocytes sedimantation rate (ESR) - 9 mm/hour, leucocytes - 4,5 109/l,
anisocytosis. Which investigations will you do?
A. * Serum Iron and Total Iron-Binding Capacity
B. Ultrasound examination
C. Urine analysis
D. Stool test
E. Bilirubin test
5.
A 17-year-old girl complains of fatigue, palpitations, tachypnea on
exertion, insomnia, she likes eat coal. Examinations reveals pallor of skin and
mucous membranes, nail cracking, tachycardia, soft systolic murmur on the
apex. Peripheral blood: Erythrocytes-3,0 1012/l, - 80 g/l, CI -0,8,
erythrocytes sedimantation rate (ESR) - 9 mm/hour, leucocytes - 4,5 109/l,
anisocytosis. What is your initial diagnosis?
A. * Iron deficiency anemia
B. Megaloblastic anemia
C. Anemia of myxedema
D. Folic acid deficiency anemia,
E. B12 vitamin deficiency anemia
6.
A 18-yr-old Asian girl presents with anaemia. She remembers her
brother died at the age of 5 after an illness since birth, which required
repeated transfusions. What is the nesessary test?

A. * Hb electrophoresis
B. Coombs' test
C. Serum B12
D. Urine for Bence-Jones proteins
E. Thrombin time, fibrin degradation products
7.
A 20 -yrs adult presents with severe hypoplastic anemia. What is most
effective treatment:
A. -interferon
B. IL-2
C. ATG therapy
D. * Bone marrow transplantation
E. Everything is correct
8.
A 21-year-old man complains of fatigue, palpitations, tachypnea on
exertion, insomnia, weakness, abdominal pain. He was treated previously in
gastroenterology 2 months ago (it was peptic ulcer). Examination reveals
pallor of skin and mucous membranes, nail cracking ,tachycardia, soft
systolic murmur on the apex. Peripheral blood: Erythrocytes-3,0 1012/l, 80 g/l, CI -0,8, erythrocytes sedimantation rate (ESR) - 9 mm/hour, leucocytes
- 4,5 109/l, anisocytosis. What is your initial diagnosis?
A. *Iron deficiency anemia
B. Megaloblastic anemia
C. Anemia of myxedema
D. Folic acid deficiency anemia
E. B12 vitamin deficiency anemia
9.
A 25-year-old woman complains of bone pain, fever, fatique, weakness,
weight loss. Physical and laboratory examinations find out tender bones,
normocytic and normochromic anemia, granulocytopenia and
thrombocytopenia. Bone x-ray examination shows lytic bone lesions. What is
the possible diagnosis?
A. *Multiple myeloma.
B. Polycytemia rubra vera
C. AML
D. CML
E. CLL
10.
A 29 year old woman was found to have hemoglobin of 7.8 g/dl with a
reticulocyte count 0.8%. The peripherial blood smear showed microcytic
hypochromic anemia. The serum iron and the total iron binding capacity were

15 microgram/dl, and 420 microgram/dl, respectively. The most likely cause


of anemia is:
A. * Iron deficiency anemia
B. Beta-thalassemia minor
C. Sideroblastic anemia
D. Anemia due to chronic infection
E. Megaloblastic anemia
11.
A 32 -year-old welder complains of weakness and fever. His illness
started as tonsillitis a month before. On exam, BT of 38.9C, RR of 24/min, HR
of 100/min, BP of 100/70 mm Hg, hemorrhages on the legs, enlargement of
the lymph nodes. CBC shows Hb of 70 g/L, RBC of 2.21012/L, WBC of
3.0109/L with 32% of blasts, 1% of eosinophiles, 3% of bands, 36% of
segments, 20% of lymphocytes, and 8% of monocytes, ESR of 47 mm/h.
What is the cause of anemia?
A. Chronic lympholeukemia
B. *Acute leukemia
C. Aplastic anemia
D. Vitamin B12 deficiency anemia
E. Chronic hemolytic anemia
12.
A 32-year-old patient status postterminal ileum resection for Crohns
disease complains of fatigue, palpitations, tachypnea on exertion, insomnia,
numbness, paresthesias in the extremities. Examinations reveals pallor of
skin and mucous membranes, nail cracking ,tachycardia, soft systolic murmur
on the apex. Peripheral blood: Erythrocytes-3,0 1012/l, - 80 g/l, CI -1,09,
erythrocytes sedimantation rate (ESR) - 9 mm/hour, leucocytes - 4,5 109/l,
anisocytosis. What is your initial diagnosis?
A. Iron deficiency anemia,
B. Megaloblastic anemia
C. Anemia of myxedema
D. Folic acid deficiency anemia,
E. * B12 vitamin deficiency anemia
13.
A 52-year-old woman suffers from thyrotoxicosis. The results of blood
test: RBC - 5,91012/l, hemoglobin level - 171 g/l, colour index - 0,9, WBC 4,9109/l. Name these changes of blood.
A. Hypoplastic anemia
B. Absolute leucocytosis
C. Leukemoid reaction
D. Relative leucopenia

E. * Absolute erythrocytosis
14.
A 54-year-old woman complains of increasing fatigue and easy bruising
of 3 weeks duration. Physical findings included pale, scattered ecchymoses
and petechiae and mild hepatosplenomegaly. CBC: RBC 2.550.000/mcL; Hb
73 g/L; HCT 20 %; PLT 23.000 mcL; and WBC 162.000/mcL with 82\%
blasts, that contained Auric rods; peroxidase stain was positive; What is the
most probable diagnosis?
a. * Acute leukemia
b. Megaloblastic anemia
c. Hemolytic anemia
d. Thrombocytopenia
e. Chronic leukemia
15.
A 65-yr-old woman presents with anaemia. She is noted to have
koilonychias and atrophic glossitis. Her blood smear reveals microcytic,
hypochromic blood cells. What is the diagnosis?
A. Megaloblastic anaemia
B. * Iron deficiency anaemia
C. Aplastic anemia
D. Hemolytic anemia
E. Sickle cell anaemia
16.
A 70-yr-old man presents with bone pain, anaemia and renal failure. His
bone marrow reveals abundance of malignant plasma cells. What is the
diagnosis?
A. * Multiple myeloma
B. Myeloid metaplasia
C. AML
D. CLL
E. Megaloblastic anaemia
17.
A decrease in hemoglobin level, increase in colour index were revealed
during examination of patient. There are megalocytes and megaloblasts in
the peripheral blood smear. What kind of anemia has the patient?
A. Hypoplastic
B. Posthemorrhagic
C. Irondeficiency
D. Hemolytic
E. * B12-deficiency

18.
A group of polar explorers 8 month worked in Antarctic Continent at
height of 3000 metes above level of seA) In the process of adaptation in the
appeared changes in blood, namely:
A. * Activation of erythropoiesis
B. Activation of leukopoiesis
C. Activation of immune system
D. Activation of phagocytes
E. Decrease of thrombocytopoiesis
19.
A man 25-year-old lost 1L of blood. At the expense of what substance
will be stimulate renewal of erythrocytes maintenance?
A. * Erythropoietin
B. Renin
C. Interferone
D. Interleukin -1
E. Serotonin
20.
A man was admitted in a clinic with complaints of dispnea, heart
acceleration, pain and burning in area of tongue, feeling of numbness of
extremities. In the past carried the resection of stomach stipulated ulcerous
illness. In blood test: Hb 80 g/l, erythrocytes 2,01012/l, leucocytes
3,5109/l, colour index 1,3. What type of anemia in the patient?
A. * B12-(folate)deficiency anemia
B. Hemolytic anemia
C. Posthemorrhagic anemia
D. Aplastic anemia
E. Proteindeficiency anemia
21.
A man, 35 years old, complains of weakness, palpitation, flickering
before eyes, dizziness. Data of anamnesis: peptic gastric ulcer, repeated
bleeding. Data of objective examination\: skin is pale, in the lungs vesicular
breathing is heard. Systolic murmur is heard at the apex, pulse rate is 100
per min, blood pressure - 100/70 mm of Hg. Mild pain is present at palpation
of epigastric region. Data of complete blood count: Red blood cells 3,2*1012/l, haemoglobin content - 75 g/l, colour index is 0,7. What kind of
anaemia is present in this case?
A. * Iron deficiency anaemia
B. Postgaemorrhagic anaemia
C. Vitamin B12-deficiency anaemia
D. Haemolitic anaemia
E. Hypoplastic anaemia

22.
A patient 2 year ago carried the operation of stomach resection for
cause tumor. At the moment of examination complains of ageneral weakness,
appearance of dark circles before eyes, dyspneA) In a blood test\: Hb 60g/l,
red corpuscles 2,81012/l, colour index 1,4. What forms of red corpuscles
are characterized for this state?
A. * Megalocytes
B. Macrocytes
C. Ovalocytes
D. Microcytes
E. Schistocytes
23.
A patient 25 year-old, palestinian, complains of weakness, dispneA. In
anamnesis there is anemia which is periodically intensified. In blood: Hb 60
g/l, erythrocytes 2,51012/l, reticulocytes 35 %. There are anisocytes and
poikilocytes, polychromatophils, much target erythrocytes. Name the type of
anemia in the patient.
A. * Thalassemia
B. Sickle cell
C. Minkovsky-Shoffars illness
D. Toxico-hemolytic anemia
E. Glucose-6-phosphat dehydrogenase anemia
24.
A patient applied to the doctor with complaints of hypodermic
hemorrhage at insignificant mechanical traumas. What from enumerated
below can be by reason of such phenomenon?
A. * Thrombocytopenia
B. Leukopenia
C. Decrease of hemoglobin content
D. Erythropenia
E. Lymphocytosis
25.
A patient complains of dyspnea for the rapid walking. A skin is pale,
cold. Results of analysis\: erythrocytes 3,21012/l, hemoglobin 90 g/l,
colour index 0,6, contents of proteins 72 g/l. In the smear of blood much
anulocytes and microcytes, there are reticulocytes, single oxiphilic
normocytes:
A. * Deficit of iron
B. Blood loss
C. Hemolysis of erythrocytes
D. Deficit of cyanocobolamin
E. Lack of protein

26.
A patient complains of general weakness, dispnae. Shortly before it she
accepted levomycetin for the prophylaxis of intestinal infection. In blood:
erythrocytes - 1,91012/l, - 58 g/l, colour index - 0,9, leucocytes - 2,2
109/l. Which anemia has the patient?
A. Metaplastic
B. Irondeficiency
C. Hemolytic
D. Aplastic
E. * Hypoplastic
27.
A patient presents with subleukemic leukemia, what changes in
peripheral blood smear will be seen?
A. Increased WBC due to blasts
B. * Blasts without increased WBC
C. None of the above
D. All of the above
E. Decreased WBC with no blasts
28.
A patient suffers from periodic attacks of fever, which caused by
malaria agent. Blood test revealed: amount of erythrocytes - 3,21012/l,
hemoglobin level - 115 g/l, colour index - 0,85. Which anemia has the
patient?
A. Pernicious
B. Posthemorrhagic
C. Proteindeficiecy
D. Iron-deficiency
E. * Hemolytic
29.
A patient suffers with periodic attacks of fever, was caused malaria
agent. On examination of blood was revealed: amount of erythrocytes
3,21012/l, content of hemoglobin 115 g/l, colour index 0,5. What anemia
does the patient suffers by?
A. * Hemolytic
B. Posthemorrhagic
C. Proteindeficiecy
D. Irondeficiency
E. Pernicious
30.
A patient was admitted at the hospital with continuous bleeding after
tooth extraction, what changes will be found in complete blood count?
A. Leukocytosis

B. Erythrocytopenia
C. * Thrombocytopenia
D. Lymphocytosis
E. None of the above
31.
A woman 24 years old, complains of general weakness, shortness of
breath, brittleness of hair and nails. Here menstruations started when she
was 10 years old. Duration of menses is for 7 days, first 4-5 days she loses a
lot of blood with menses. She had not deliveries and abortions. Data of
examination: sklera are of blue tint, pallor of skin is present. What changes
should you expect in here blood count?
A. * Decreased level of serum iron
B. Increased level of serum iron
C. High colour index
D. Increased level of free bilirubin
E. Decreased amount of thrombocites
32.
A woman 52-year-old suffers with thyrotoxicosis. Examination of blood
was revealed\: amount of erythrocytes 5,91012 /l, content of hemoglobin
171 g/l, colour index 0,9, amount of leucocytes 4,9109/l. How are named
these changes of blood ?
A. * Absolute erythrocytosis
B. Hypoplastic anemia
C. Absolute leucocytosis
D. Leukemoid reaction
E. Relative leucopenia
33.
An 8-yr-old boy presents with painful swelling of hands and feet,
jaundice and anaemia. He is noted to have splenomegaly. His blood film has
target cells. What is your diagnosis?
A. Iron-deficiency anemia.
B. * Sickle cell anaemia
C. B12-deficiency anemia
D. Hemolytic anemia.
E. Aplastic anemia.
34.
B12-deficiency anemia was appeared in a patient after resection of
stomach. Which colour index characterize this illness?
A. 1,0
B. 1,15
C. 0,85

D. 0,70
E. * 1,30
35.
Blood examination of the patient with anacidic gastritis reveals the
following results: RBC - 2,51012/l, Hb- 50 g/l, colour index - 0,6, microcytes.
Which anemia is characterized by such indexes?
A. B12-deficiency
B. Proteindeficiency
C. Aplastic
D. Hypoplastic
E. * Irondeficiency
36.
In a 52-year-old man with a history of surgery to remove the stomach
erythrocytes amout in blood is 2,01012/l, Hb- 85 g/l, colour index - 1,27.
Lack of what vitamin causes such changes?
A. A
B. C
C. B6
D. D
E. * B12
37.
In a patient was determined such changes in general blood acount:
RBC - 81012/l, hemoglobin - 179 g/l, hematocrit - 0,55 l/l. It is characterized
for:
A. Dehydration
B. Infusion of erythrocytic mass
C. Infusion of blood
D. B12-deficiency anemia
E. * Policytemia
38.
Jaundice, splenomegaly, pain in the left hypochondrium, pigment
gallstones were found in the patient during his examination. MinkowskiChauffard's disease was diagnosed. The peripheral blood smear in this case is
characterized by
A. Reticulocytosis
B. Agranulocytosis
C. Macrocytosis
D. Ovalocytosis
E. * Microspherocytosis

39.
Patient 30 years present at the hospital with fatigue, weight loss, bone
pain with tenderness. The diagnosis is chronic myelocytic leukemia, which of
these confirms the diagnosis?
A. Lymphocytic infiltrates
B. * Myeloid hyperplasia of bone marrow
C. Smudge cells
D. None of the above
E. A and E
40.
Patient 55 yrs was admitted at the hematological department with the
following laboratory result: GBC reveals lymphocytosis, smudge cells,
anemia, thrombocytopenia and bone marrow biopsy shows lymphocytic
infiltrates: What is clinical diagnosis?
A. * Chronic lymphocytic leukemia
B. Chronic myelocytic leukemia
C. Multiple myeloma
D. B and C
E. None of the above
41.
Patient D, was diagnosed with acute leukemia, peripheral blood reveals
blasts 35 % without increased WBC, what type of acute leukemia is present in
this patient?
A. * Subleukemic
B. Aleukemic
C. Leukemic
D. None of the above
E. All of the above
42.
Patient E was admitted at the hospital in the hematological
department, CBC revealed RBC- 4.0?1012/l , WBC- 6?109/l, Hb -130g/l,
platelets- 60 ?109/l, lymphocytes- 45 %, reticulocytes- 1 %, define abnormal
parameters.
A. Erythrocytosis and lymphocytosis
B. Lymphocytosis and leucocytopenia
C. * Thrombocytopenia and lymphocytosis
D. Lymphocytopenia and thrombocytosis
E. Erythrocytosis and reticulocytopenia
43.
Patient G, was admitted to the hospital with symptoms of leukemia,
how can chronic myelocytic be differentiated from chronic lymphocytic from
bone marrow biopsy?

A. Myeloid hyperplasia
B. High myeloid: Erythroid ratio
C. Hypercellular bone marrow
D. Increased basophils and eosinophils
E. * All of the above
44.
Patient I., 40 years old, was undergone resection of stomach because
of peptic ulcer 5 years ago. The patient develop general weakness and
dyspnea last time. Data of his complete blood count are the following:
erythrocytes count is 3,1*1012/l, haemoglobin content is 60 gr/l, colour index
is 0,6, leucocytes count is 4,5*109/l, stub neutrophyls - 2%, eosynophils - 3%,
segmented neutrophyls - 5%, lymphocytes - 32%, monocytes - 8%, ESR - 5
mm/hour, serum iron content is 6 mkmol/l. What pathological condition
developed in the patient?
A. * Chronic iron deficiency anaemia
B. Haemolytic anaemia
C. Aplastic anaemia
D. Acute posthemorrhagic anaemia
E. Folic acid deficiency anaemia
45.
Patient is diagnosed with Hodgkins lymphoma, during examination
painless lymphadenopathy, hepatosplenomegaly is observed. What other
name is Hodgkins lymphoma called?
A. CML
B. AML
C. ALL
D. Multiple myeloma
E. * Lymphogranulomatosis
46.
Patient is diagnosed with Kaylers disease, presence of Bence Jones
protein in urine was discovered. What other name can this disease be called?
A. CML
B. ALL
C. * Multiple myleloma
D. Hodgkins lymphoma
E. None of the above
47.
Patient K, arrived at the hospital with painless lymphanedopathy,
lymph node biopsy reveals Reed - Sternberg cells. Give clinical diagnosis.
A. Chronic lymphocytic leukemia
B. Multiple myeloma

C. * Hodgkins disease
D. None of the above
E. All of the above
48.
Patient L, was admitted to the hospital with symptoms of acute
leukemia, peripheral blood smear reveals decreased WBC with no blasts.
What form is diagnosed?
A. C and D
B. Leukemic
C. Subleukemic
D. * Aleukemic
E. None of the above
49.
Patient M, was admitted to the hospital with symptoms of kaylers
disease, presence of bence jones protein in urine was discovered. What is
Kaylers disease?
A. Malignant tumor of lymphoid tissue
B. Cancer of WBC
C. * Cancer of plasma cells
D. None of the above
E. All of the above
50.
Patient P, arrived at the hospital with recurrent infections, GBC
revealed plasma cells, what other means of examination will be done to
confirm diagnosis?
A. Biochemical blood profile
B. General urine analysis
C. Bone marrow biopsy
D. * Serum protein electrophoresis
E. A and B
51.
Patient present to the hospital with itchy skin, night sweats and
unexplained weight loss. Diagnosis is hodgkins lymphoma. What laboratory
examination will you to do confirm diagnosis?
A. Complete blood count
B. * Lymph node biopsy
C. Urine analysis
D. B and C
E. All of the above

52.
Patient S, was admitted and diagnosed with acute leukemia. What
changes will you find in complete blood count?
A. * Anemia, thrombocytopenia and leukocytosis
B. Anemia, thrombocytosis and leukocytopenia
C. Thrombocytosis and presence of blasts
D. Anemia and presence of blasts
E. All of the above
53.
Patient was admitted to the hospital and diagnosed with multiple
myeloma, what other diseases can it be differentiated with?
A. Peptic ulcer disease
B. COPD
C. None of the above
D. * Hodgkins lymphoma
E. All of the above
54.
Patient X, presents at the hospital with fatigue, sweat and weight loss.
During examination, hepatosplenomegaly, lymphadenopathy was revealed.
Bone marrow biopsy reveals presence of blast 35 %, promyelocytes and
promonocytes. Give clinical diagnosis.
A. Chronic lymphocytic leukemia
B. Chronic myelocytic leukemia
C. All of the above
D. * Acute leukemia
E. None of the above
55.
Patient Z, was admitted to the hospital with suspicion of leukemia, how
can you differentiate acute leukemia from chronic leukemia.
A. Clinical presentation
B. Onset of disease
C. All of the above
D. None of the above
E. * Morphology of cells
56.
Patient, 60 yrs, is diagnosed with Hodgkins disease at the hospital.
What specific cells can confirm diagnosis?
A. Sickle cells
B. Cabot ring
C. Target cells

D. Burr cells
E. * Reed Sternberg cells
57.
Posthemorrhagic anemia was developed in a patient who suffers from
the periodic bleeding due to fibromyoma of uterus. What is its type?
A. , hyporegeneratory
58.
?Thalasemia with hemolysis of erythrocytes was determined in a
patient which arrived from Tunis. Illness was diagnosed on the basis of
presence in blood:
B. Grainy erythrocytes
C. Polychromatic erythrocytes
D. Sickle-cell erythrocytes
E. Reticulocytes
F. * Taget erythrocytes
59.
The amount of reticulocytes was increased in the peripheral blood of a
patient with an stomach ulcer at 5 days after acute bleeding. What may this
symptom testify about?
A. Presence of allergy
B. Presence of inflammation in stomach
C. Increase of proteins synthesis
D. Presence of hypoxia
E. * Activation of hemopoiesis
60.
The atrophy-inflammatory processes in the cavities of mouth, disorder
of deep sensitiveness were revealed in a woman. Addison-Birmers anemia
was diagnosed. This anemia is
A. Normochromic
B. Erythroblastic
C. Hemolytic
D. Irondeficiency
E. * Megaloblastic
61.
The excessive entering into blood of estrogenes because of persistence
of follicle often indicate by reason of the uterine bleeding. What anemia can
develop in this case?
A. Metaplastic
B. Sideroblastic
C. Sickle sel
D. Hypoplastic

E. * Iron-defficiency
62.
The resection of stomach was made in a man 5 years ago.
Megaloblastic anemia was revealed. What is direct cause of origin and
development of megaloblastic aaemia in the patient?
A. Alimentary lack of vitamin B12
B. Alimentary lack of folic acid
C. Deficit of iron
D. Deficit of vitamin A
E. * Intrinsic factor deficiency
63.
There are hypochromic erythrocytes, micro- and anisocytes,
poikilocytes in the smear of patient blood. These signs are common for irondeficiency and iron-refractory anemias. What investigations will allow
differentiate these anemias?
A. Determination of erythrocytes amount
B. Analysis of bone marrow smear
C. Determination of haemoglobin content
D. Determination of colour index
E. * Determination of serum iron
64.

What is the clinical diagnosis?


A. Iron deficiency anemia
B. * Acute leukemia
C. None of the above
D. Post hemorrhagic anemia
E. All of the above

65.
4-month old child suffers from severe rickets. Disorders of digestion
were not found. A child is exposed to sunlight foe enough time every day. For
two months a child obtained the vitamin D3, however the symptoms of
rickets were not diminished. Disorders of the synthesis of which compound
can explain the development of rickets in this child?
A. Calcitonin
B. * Calcitriol (1,25- dihydroxycholecalciferol)
C. Thyroxine
D. Parathormone
E. Insulin
66.
A patient complains about a general weakness and bleeding of gums.
Insufficiency of which vitamin can be reason of such state?

A. * Vitamin C
B. Vitamin E
C. Vitamin A
D. Vitamin H
E. Vitamin D
67.
A patient complains about loss of weight, general weakness, pain in the
area of heart, petehial hemorrhages, bleeding of gums, loss of teeth. The lack
of which vitamin caused such symptoms?
A. Vitamin K
B. Vitamin PP
C. Vitamin B1
D. * Vitamin C
E. Vitamin B2
68.
For the patient with decreased immunity and frequent cold diseases
was recommend to use ascorutin as more effective preparation than ascorbic
acid. Which component in this preparation strengthen the action of vitamin of
C?
A. * Vitamin P
B. Vitamin A
C. Glucose
D. Lactose
E. Vitamin D
69.
Patient complains for dry lips, cracks and crusts in the mouth
corners, bright red tongue, dermatitis of nasolabial folds, photophobia and
conjunctivitis. To the lack of which vitamin is it related?
A. * Riboflavin
B. Cholecalciferol
C. Pyridoxin
D. Coballamin
E. Ascorbic acid
70.
Patient complains for pain in joints. By sight can be seen edemas and
reddening in the joints area. What enzyme activity have be investigated to
draw out a diagnosis?
A. hyaluronidase
B. creatine kinase
C. alkaline phosphatase

D. acidic phosphatase
E. urease
71.
Patient of 62 years has diabetes mellitus. Glomerulosclerosis has
evolved. The most possible reason for this complication is the affection in
renal glomerulus of:
A. calcium excretion
B. formation of primary urine
C. gluconeogenesis process
D. glucose reabsorption
E. * collagen structure
72.
Patient of 63 years suffers from rheumatitis. The concentration of
oxyprolin in blood and in urine is increased. What is the main reason of
hyperoxyprolinemia?
A. hyaluroprotein degradation
B. * collagen degradation
C. kidney malfunction
D. cathepsin activation
E. prolyl hydroxylase activation
73.
A 30 year old patient was admitted to the hospital, she excretes small
amount of mucous, viscous sputum, but later the amount of sputum she
excretes increased. It became mucous purulent during microscopic studies a
lot of cylindrical, epithelium, leucocytes and at times red blood cells. Which
diagnosis is typical for such symptoms?
A. Bronchial Asthma
B. * Acute Bronchitis
C. Lobar Pneumonia
D. Bronchiectasis
E. Lung Abcess
74.
A cloudy pleural fluid has a glucose level of 30 mg/dL (serum glucose
level is 100 mg/dL) and a pH of 6.8.
A. * Pneumonia
B. Bronchitis
C. Pleuritis
D. Myocardial infarction
E. Non is correct

75.
A patient expectorates up to 600 ml of sputum a day, she went to the
hospital and microscopic examination was conducted microscopically, there
was presence of white blood cells, elastic fibers, scraps of lung tissue,
crystals of fatty acids and cholesterol and different flora. What is the
diagnosis of these patient?
A. Lobar Pneumonia
B. Lung Cancer
C. Tuberculosis of lungs
D. * Lung Abcess
E. Bronchial Asthma
76.
Doctor consults a patient with complaints of frequent cough, high
temperature, coupious amount of sputum (>200 ml/day), which has foul odor.
What disease can be suspected?
A. Acute Pneumonia
B. Acute bronchitis
C. Bronchial asthma
D. Tuberculosis
E. * Lung abscess, gangrene, bronchiectasis
77.
Doctor consults a patient with complaints of sleeplessness, night
sweats, frequent cough. There is small amount of sputum with traces of blood
in it. What disease can be suspected?
A. Acute Pneumonia
B. Acute bronchitis
C. Bronchial asthma
D. * Tuberculosis
E. Lung abscess
78.
Fluid from a patient with congestive heart failure is collected by
thoracentesis and sent to the laboratory for testing. It appears clear and pale
yellow and has a WBC count of 450/mL, fluid:serum protein ratio of 0.35, and
fluid:serum LD ratio of 0.46. What type of fluid was collected?
A. * Pleural fluid
B. Peritoneal fluid
C. Serum
D. Plasma
E. Non is correct
79.
Fluid from a patient with congestive heart failure is collected by
thoracentesis and sent to the laboratory for testing. It appears clear and pale
yellow and has a WBC count of 450/mL, fluid:serum protein ratio of 0.35, and

fluid:serum LD ratio of 0.46. Based on the laboratory results, would this fluid
be considered a transudate or an exudate?
A. * Transudate
B. Exudate
C. Serum
D. Plasma
E. Non is correct
80.
Patient K. was diagnosed of lung cancer, what type of sputum does he
expectorate:
A. Mucous purulent
B. Viscous rusty
C. * Mucous purulent with RBC
D. Gelatinous
E. Purulent
81.
Patient K. was diagnosed of lung cancer. Which type of cell can be
found in microscopic examination of sputum in this case?
A. * Atypical cells
B. Chief cells
C. Parietal cell
D. Typical cell
E. Satellite cell
82.
The following results were obtained on a peritoneal fluid: serum
albumin, 2.8 g/dL; fluid albumin, 1.2 g/dL. Calculate the SAAG.
A. 1,2
B. * 1,6
C. 2,5
D. 5,5
E. Non is correct
83.
What type of sputum is excreted by patient, which was admitted to the
hospital with lobar pneumonia?
A. Serous purulent
B. Glassy
C. * Viscous rusty sputum
D. Serous

E. Bloody sputum
84.
A man who ascented at height 4,5 km lost consciousness suddenly. The
cause is a hypocapnia due to
A. * Hyperventilation
B. Decrease of metabolism rate
C. Binding of carbonic acids by proteins
D. Neutralization of carbonic acid by bicarbonates
E. Absorption of carbonic acids by red blood cells
85.
A man with the barbiturate poisoning was hospitalized to emergency
department. He has hypopnea due to oppression of respiratory center. What
type insufficiency of breathing does he have?
A. * Disregulative
B. Obstructive
C. Restrictive
D. Perfusive
E. Diffusion
86.
A nurse has been asked to create a cancer risk reduction pamphlet for
the clients who come to a clinic that serves a large African American
population. Therefore, prevention and early detection tips for which cancer
types would be most important to include in this pamphlet?
A. * Lung and prostate
B. Bone and leukemia
C. Skin and lymphoma
D. Stomach and esophageal
87.
A patient 28 years old has a pneumonia The most typical symptom of
this disease is:
A. * Rapid and shallow breathing
B. Rapid and deep breathing
C. Slow and deep breathing
D. Chein-Stocks breathing
E. Kussmaul`s breathing
88.
A patient has an acute decrease of surfactant activity in the lungs.
What changes can be expected in this patient?
A. * Inclination alveoles to spasm and impossibility of their rapid
spread
B. Change elastic properties of lungs

C. Decrease lungs secretion


D. Decrease circulation of blood in lungs
E. Growth up of connective tissue in lungs
89.
A patient was hospitalized with a cranial-cerebral trauma in the grave
condition. The breathing is characterized with the convulsive attempts to
breath, which is not stoped, that is sometimes broken by exhalation. What
type of breathing has this patient?
A. * Apneutic
B. Gasping
C. Kussmaul
D. Chein-Stocks
E. Biot
90.
A patient with a cerebral hemorrhage is in a coma. He has growth of
deepand frequency of breathing, and then its slowing to apnea, then the
cycle of respiratory recurs. What type of breathing has this patient?
A. * Chein-Stocks
B. Gasping
C. Kussmaul
D. Apneutic
E. Biot
91.
A patient with bronchial asthma has the attack of dyspnea after a walk.
The disorders of breathing due to primary disturbance of:
A. * Ventilation capability of alveoli
B. Integrality of pleura cavity
C. Movement of thorax
D. Neuro-muscular function
E. Function of respiratory center
92.
A patient with the edema of brain has the disorders of breathing. They
are characterized by permanent amplitude, but respiratory movements
suddenly stopped, and then suddenly renew. What pathological type of
breathing in this patient?
A. * Biot
B. Apneutic
C. Hysterical
D. Kussmaul
E. Chein-Stocks

93.
A patient, 64 years old, was hospitalized with complaints of a cough
with sputum, expressed dyspnea . During examiation the next signs were
revealed: position is forced, breath rate 32/min, the intercostales muscles
take part in the breathing. During X-ray examination the increased
transparency of lungs were determined. What is the most important in the
pathogenesis of respiratory failure in this patient?
A. * Decrease of elastic properties of lungs
B. Accumulation of sputum in the bronchial tubes
C. Thinning of mucus shell of bronchial tubes
D. Insufficiency of the surfactant system of lungs
E. Fibrosis of lungs
94.
for:

Alternation of periods apnea with periods of respiratory motions typical


A. * Periodical breathing
B. Bradypnea
C. Hypoxia
D. Apnea
E. Hyperpnea

95.

Alveolar ventilation can be violated at:


A. * Pleurisy
B. Ischemic heart disease
C. Cyanides poisoning
D. Poisoning by nitrates
E. Poisoning by carbon oxide

96.
An young man with the signs of morphin poisoning was hospitalized to
the emergency department. His breathing is shallow and slow as a result of
oppression of respiratory center. What type of breathing disorders is present
in this patient?
A. * Disregulative disorders of alveolar ventilation
B. Perfusive
C. Ventilation restrictive
D. Diffusive
E. Ventilative obstructive
97.
At the height 7 km alpinist feeled dizziness and severe weakness. He
lost consciousness, his breathing stoppeD) These disorders was a result of
A. * Surplus discharge of 2 from his organism

B. Insufficient formation of 2 in tissues


C. Insufficient supply of organism by 2
D. Insufficient utilization of 2 by tissues
E. Insufficient release of 2 from oxyhemoglobin
98.
In which of the enumerated pathological processes you can determine
the restrictive form of external breathing disorders?
A. * Lung edema
B. Bronchial asthma
C. Poliomyelitis
D. Syringomyelitis
E. Bronchitis
99.
Inability of breathing organs to provide normal gas composition of
blood is named:
A. * Respiratory failure
B. Hypercapnia
C. Hypoxia
D. Apnea
E. Asphyxia
100. Nitric oxide mediates this effect on vascular smooth muscle:
A. * Smooth muscle relaxation
B. Smooth muscle contraction
C. No effect
D. Provide production of energy
E. Smooth muscle protection
101. The obstructive type of respiratory insufficiency develops in patient G.
as a result:
A. * Stricture passage of respiratory ways
B. Collapse alveoli
C. Edema lungs
D. Atelectasis
E. Pneumonia
102. The patient is diagnosed with lobar pneumonia What type of
respiratory insufficiency will be observed in this patient?
A. * Restrictive

B. Obstructive
C. Pectoral
D. Abdominal
E. Mixed
103. The patient was asked to breath deep during auscultation. After 10
respiratory movements she felt dizziness. What is the cause of this disorder?
A. * Respiratory alcalosis
B. Decrease concentrations of hemoglobin
C. Disorder of diffusions of gases in lungs
D. Respiratory acidosis
E. Decrease quantity of erythrocytes
104. What type of breathing in patient A. is typical for a child with
diphtheria of larynges?
A. * Dispnea (shortness of breath)
B. Gasping breathing
C. Apneustic breathing
D. Kussmaul`s breathing
E. Biot's breathing
105. What type of hypoxia in patient G. is conditioned by violation of
exchange gases in lungs:
A. * Respiratory
B. Exogenous
C. Hemic
D. Tissue
E. Circulatory
106. A 23-year-old woman with type 1 diabetes mellitus presents to the
emergency department because of a 2-day history of dysuria and urinary
frequency. She has no gross hematuria, fever, or chills. She states that 3
years ago, she had cystitis twice in 6 months; in both occasions, she was
treated with antibiotics. She uses insulin to control diabetes and takes 1 or 2
ibuprofen tablets daily for headaches. On physical examination, the patient is
alert and in no distress. Blood pressure is 115/80 mm Hg, pulse rate 80/min,
and temperature 37.4 C (99.3 F). Optic funduscopy reveals
microaneurysms. The neck is supple, the carotids are normal, and the lungs
are clear. Cardiac examination reveals regular sinus rhythm and no murmur
or rub. Abdominal examination is normal. No lower extremity edema or ulcers
are present. Neurologic examination demonstrates diminished sensitivity to
pinprick and light touch in the lower extremities. Laboratory studies:
Leukocyte count 8400/?L Polymorphonuclear cells 70% Lymphocytes 20%

Hematocrit 40% Hemoglobin 13.8 g/dL Serum creatinine 1.8 mg/dL (was 1.6
mg/dL 1 month ago) Serum sodium 140 meq/L Serum chloride 106 meq/L
Serum potassium 6.2 meq/L Serum bicarbonate 23 meq/L Urinalysis Specific
gravity 1.020; 2+ glucosuria, 1 + hematuria, 3+ proteinuria, no ketonuria ,
3+ leukocyturia; 25 to 50 leukocytes/hpf, 10 to 20 erythrocytes/hpf, broad
casts On renal ultrasonography, the right kidney is 11.0 cm and the left
kidney is 10.9 cm. No hydronephrosis or stones are present. What is the most
likely cause of this patients hyperkalemia?
A. Diabetic ketoacidosis
B. * Hyporeninemic hypoaldosteronism
C. Acute renal failure
D. High potassium diet
E. No correct answer
107. A 26-year-old woman with type 1 diabetes mellitus presents to the
emergency department because of abdominal pain for the past 24 hours. Her
temperature is 38C (101 F). Laboratory studies: Blood urea nitrogen 20
mg/dL Serum creatinine 1.2 mg/dL Serum sodium 133 meq/L Serum
potassium 3.9 meq/L Serum chloride 97 meq/L Serum bicarbonate 10 meq/L
Serum glucose 450 mg/dL Arterial blood gases pH 7.2, PCO2 23 mm Hg Blood
cultures Negative Whole-blood lactate 0.6 mmol/L What condition best
explains the patients acid-base status?
A. Diabetic ketoacidosis alone
B. * Diabetic ketoacidosis complicated by a proximal renal tubular
acidosis
C. Diabetic ketoacidosis complicated by sepsis
D. Diabetic ketoacidosis complicated by respiratory acidosis
E. No correct answer
108. A 28-year-old woman presents for evaluation of recurrent kidney stones
that she says contain calcium. She estimates that she has passed four
stones during the past 4 years. She currently has no symptoms of renal colic.
For several years, she has had dry eyes and dry mouth. She also describes
symptoms of Raynauds phenomenon. Crohns disease was diagnosed 10
years ago; the patient is currently asymptomatic and passes one formed stool
daily. She takes no medications. There is no family history of renal stone
disease. On examination, the patient is alert and healthy. Blood pressure is
115/74 mm Hg, pulse rate is 72/min, and temperature is 37 C (98.6 F). The
skin is clear, and the joints are normal. The lungs are clear. Cardiac
examination shows regular sinus rhythm and no murmur. The liver and spleen
are not palpable, and the abdomen is not tender. Plain abdominal
radiography shows multiple calcifications overlying both renal
shadows.Laboratory studies: Hemoglobin 13.2 g/dL Hematocrit 39%
Leukocyte count 7400/?L Blood urea nitrogen 18 mg/dL Serum creatinine 0.9
mg/dL Serum sodium 138 meq/L Serum potassium 2.8 meq/L Serum chloride
109 meq/L Serum bicarbonate 19 meq/L Serum calcium 9.1 mg/dL Serum
phosphorus 3.2 mg/dL Urinalysis pH 6.0; specific gravity 1.020; trace
hematuria, no proteinuria Arterial blood pH 7.29 What is the most likely
etiology of this patients renal stone disease?

A. Idiopathic hypercalciuria
B. Primary hyperthyroidism
C. * Distal renal tubular acidosis
D. Enteric hyperoxaluria
E. No correct answer
109. A 32-year-old white woman with slowly progressive chronic kidney
disease secondary to post-streptococcal glomerulonephritis is seen for
routine follow-up. Her medical regimen includes dietary phosphorus
restriction; oral calcium acetate, 667 mg three times daily with meals as a
phosphorus binder; and ramipril, 10 mg/d. The estimated glomerular filtration
rate is stable, at 22 mL/min. Laboratory studies: Serum creatinine 3.2 mg/dL
Serum calcium 8.4 mg/dL Serum phosphorus 4.9 mgldL Serum albumin 4.0
g/L Serum parathyroid hormone 256 pg/mL What is the most appropriate
management plan?
A. Add 1,25-dihydroxyvitamin D (calcitriol), to increase the serum
calcium level to 9.5 to 10.5 mg/dL
B. Add calcium carbonate, 2 tablets with each meal
C. Add 1 ,25-dihydroxyvitamin D (calcitriol), to suppress parathyroid
hormone to normal levels or below
D. Increase calcium acetate binder from two to three tablets three
times daily with meals
E. * Add 1 ,25-dihydroxyvitamin D (calcitriol), to suppress parathyroid
hormone to two to three times the upper limit of normal
110. A 34-year-old pregnant woman with a 5-year history of biopsydiagnosed hypertensive nephropathy has been followed in obstetric clinic for
3 months after her last menstrual period. One year ago, her serum creatinine
concentration was 1 .6 mg/dL. Her pregnancy has been uneventful. Her blood
pressure has been well controlled on a combination of methyldopa and
hydralazine and is currently 130/85 mm Hg. She has trace edema. Laboratory
studies: Hematocrit 37% Leukocyte count Normal Platelet count Normal
Peripheral smear No schistocytes Blood urea nitrogen 14 mg/dL Serum
creatinine 1.8 mg/dL Serum uric acid 4.9 mg/dL Urinalysis Specific gravity,
1.010; urinary protein 4+ by dipstick; no glucosuria, hematuria, or ketonuria
Microscopic urine examination shows rare broad casts. Liver function tests
are normal. Which one of the following statements about the patients course
is true?
A. She has developed preeclampsia.
B. * The course is most consistent with progression of her chronic
renal disease.
C. She has developed microangiopathic hemolytic anemia.
D. She has developed prerenal azotemia.
E. Her blood pressure is likely to improve during the course of her
pregnancy.

111. A 34-year-old woman presents to the emergency department because


of recurrent episodes of palpitations, numbness of the hands, a generalized
feeling of warmth, and muscle weakness. She has no chest pain or dyspnea.
There is no history of weight loss, diarrhea, or vomiting. She does not smoke
and drinks less than 1 ounce of alcohol per month. Graves disease was
diagnosed 2 years ago and was treated with radioiodine; she now takes
levothyroxine, 100 ?g/d. She also takes paroxetine and norgestimate/ethinyl
estradiol. She has no family history of renal disease or diabetes. Cervical disc
surgery was performed 1 year ago. On examination, the patient is alert and
oriented but is in mild distress from her symptoms. Blood pressure is 110/70
mm Hg, pulse rate 95/min, respiratory rate 1 5/min, temperature 36.7 C (98
F). No neck vein distention is present. The lungs are clear, and cardiac
examination reveals regular sinus rhythm and no murmur. The abdomen is
soft, without organomegaly or mass. In the lower extremities, pulses are
normal and no edema is present. Cranial nerves are intact. She has 2+
bilateral reflexes and mild generalized weakness but no Babinski reflex.
Laboratory studies: Complete blood count Normal Blood urea nitrogen 12
mg/dL Serum creatinine 0.9 mg/dL Serum sodium 138 meq/L Serum
potassium 3.5 meq/L Serum chloride 103 meq/L Serum bicarbonate 24 meq/L
Serum thyroid-stimulating hormone 3.2 mIU/L Serum calcium 9.2 mg/dL
Serum magnesium 1.8 mg/dL Serum phosphorus 1.1 mg/dL (repeat, 0.9
mg/dL) Serum albumin 4.0 g/dL Serum glucose 98 mg/dL Arterial blood
gasespH 7.4; PCO2 40 mm Hg What is the most likely cause of this patients
hypophosphatemia?
A. Hyperparathyroidism
B. Hyperventilation syndrome related to panic attacks
C. * Renal phosphate wasting
D. Gastrointestinal malabsorption
E. No correct answer
112. A 35-year-old man had HIV infection diagnosed 2 months ago. His
serum creatinine concentration was 0.6 mg/dL. Treatment with highly active
antiretroviral therapy with zidovudine, lamivudine, and abacavir was
recommended, but he wished to wait before starting treatment. He is brought
to clinic by a friend who states that the patient has had fever, confusion, and
disorientation for 1 day. Physical examination reveals blood pressure 110/70
mm Hg and pulse rate 1 00/min that is regular supine and standing. The
chest is clear, without cardiac murmur or gallop, and the abdomen is normal.
Moderate bilateral lower extremity edema is present. Laboratory studies:
Hemoglobin 7.8 g/dL Leukocyte count 10,2000/?L Platelet count 19,000/?L
Blood urea nitrogen 37 mg/dL Serum creatinine 2.7 mg/dL Serum sodium 136
meq/L Serum potassium 5.2 meq/L Serum chloride 99 meq/L Serum
bicarbonate 22 meq/L Urinalysis Specific gravity 1.030; 3+ hematuria,
traceproteinuria, trace ketonuria, no glucosuria Urinary microscopic
examination shows a few erythrocytes, but no erythrocyte casts. The lactate
dehydrogenase level is elevated. Peripheral blood smear shows many
schistocytes. What is the most likely cause of this patients renal failure?
A. * Thrombotic thrombocytopenic purpura
B. HIV-associated nephropathy
C. Surreptitious ingestion of antiretroviral drugs

D. Outpatient acute tubular necrosis


E. HIV-associated immune-mediated glomerulonephritis
113. A 38-year-old black man presents for hypertension discovered during a
pre-employment examination. He is healthy but has a family history of
hypertension in both parents and two siblings. He has no history of
cardiovascular disease and does not use tobacco, alcohol, or recreational
drugs. He is taking no medications. The patient appears well. Height is 173
cm (68), body weight is 78 kg (172 Ib), and blood pressure is 158/1 02 mm
Hg seated and standing. The physical examination is otherwise normal. A
complete blood count and electrolyte panel are normal. The serum creatinine
concentration is 1 .8 mg/dL, and urinalysis reveals 2+ proteinuria. Which is
the most appropriate antihypertensive therapy for this patient?
A. Intensive lifestyle modification
B. Diuretic
C. Nondihydropyridine calcium channel blocker
D. * Angiotensin-converting enzyme inhibitor
E. No correct answer
114. A 38-year-old man with a history of idiopathic focal and segmental
glomerulosclerosis developed end-stage renal disease and subsequently
underwent a cadaveric renal transplant 28 months ago. He presents to your
office for a routine follow-up visit. His transplantation was uncomplicated,
without delayed graft function or clinically apparent acute rejection episodes.
His immunosuppression regimen consisted of prednisone, cyclosporine, and
azathioprine. His serum creatinine concentration on discharge was 1.4 mg/dL.
He was given colchicine for a gouty attack 4 months ago. At a follow-up clinic
appointment 3 months ago, his blood pressure was elevated, and his serum
creatinine concentration was 1 .7 mg/dL. The urinary protein-to-creatinine
ratio was less than 0.3. He was given diltiazem for better control of blood
pressure. His immunosuppressive regimen remained unchanged. At the
current visit, physical examination reveals a mild tremor and blood pressure
of 150/90 mm Hg. Cardiac, pulmonary, and abdominal examinations are
unremarkable. There is no tenderness at the transplant site, and the patient
has trace bilateral edema. Laboratory studies: Serum creatinine 2.2 mg/dL
Serum uric acid 12 mg/dL Urinalysis Specific gravity 1.010; trace proteinuria;
no glucosuria, hematuria, or ketonuria Urine microscopy Few broad casts,
scattered renal epithelial cells Urine protein-to-creatinine 0.4 Urine uric acidto-creatinine 0.6 What is the most likely cause of this patients current renal
dysfunction?
A. Transplant renal artery stenosis
B. Recurrent focal and segmental glomerulosclerosis
C. * Cyclosporine toxicity
D. Uric acid nephropathy
E. Polyoma virus nephropathy
115. A 39-year-old male carpenter presents to the emergency department
with a 4-hour history of gradually worsening right flank and right upper

quadrant pain radiating to the right lower quadrant and into the right testicle.
He vomits once shortly after arrival. He does not have fever or chills but has
mild dysuria. On examination, the patient is restless because of pain. Blood
pressure is 145/89 mm Hg, pulse rate is 92/min, and temperature is 37 C
(98.6 F). Abdominal examination reveals mild right costovertebral angle
tenderness, but no abdominal guarding. Genitalia are normal. The serum
creatinine concentration is 0.9 mg/dL. Urinalysis shows a specific gravity of
1.025, 3+ hematuria, no proteinuria. Urine microscopy reveals more than 50
erythrocytes/hpf, 3 to 5 leukocytes/hpf, and occasional calcium oxalate
crystals. You suspect that a renal stone is causing the colicky pain and
hematuria. What radiologic procedure is best to confirm the diagnosis?
A. Plain radiography of the abdomen
B. Intravenous pyelography
C. Renal ultrasonography
D. * Noncontrast spiral computed tomography
E. No correct answer
116. A 39-year-old male carpenter presents to the emergency department
with a 4-hour history of gradually worsening right flank and right upper
quadrant pain radiating to the right lower quadrant and into the right testicle.
He vomits once shortly after arrival. He does not have fever or chills but has
mild dysuria. On examination, the patient is restless because of pain. Blood
pressure is 145/89 mm Hg, pulse rate is 92/min, and temperature is 37 C
(98.6 F). Abdominal examination reveals mild right costovertebral angle
tenderness, but no abdominal guarding. Genitalia are normal. The serum
creatinine concentration is 0.9 mg/dL. Urinalysis shows a specific gravity of
1.025, 3+ hematuria, no proteinuria. Urine microscopy reveals more than 50
erythrocytes/hpf, 3 to 5 leukocytes/hpf, and occasional calcium oxalate
crystals. You suspect that a renal stone is causing the colicky pain and
hematuria. What radiologic procedure is best to confirm the diagnosis?
A. Plain radiography of the abdomen
B. Intravenous pyelography
C. Renal ultrasonography
D. * Noncontrast spiral computed tomography
E. No correct answer
117. A 39-year-old nurse has recurrent calcium nephrolithiasis due to
idiopathic hypercalciuria (24-hour urinary calcium excretion of 350 mg and
sodium excretion of 250 meq). You prescribe a low-sodium (100 meq/d), lowoxalate, normal-calcium diet and start therapy with hydrochlorothiazide, 50
mg/d. Two months later, you obtain the following laboratory studies: Serum
sodium 138 meq/L Serum potassium 2.9 meq/L Serum chloride 110 meq/L
Serum bicarbonate 33 meq/L Arterial blood pH 7.43 24-Hour urine studies:
Creatinine 900 mg Calcium 290 mg Oxalate 45 mg Uric acid 540 mg Citrate
356 mg Potassium 45 meq Sodium 225 meq pH 4 What is the most likely
cause of the persistent hypercalciuria?
A. Distal renal tubular acidosis

B. Hyperoxaluria
C. Noncompliance with the normal-calcium diet
D. Surreptitious laxative use
E. * Noncompliance with the low-sodium diet
118. A 39-year-old nurse has recurrent calcium nephrolithiasis due to
idiopathic hypercalciuria (24-hour urinary calcium excretion of 350 mg and
sodium excretion of 250 meq). You prescribe a low-sodium (100 meq/d), lowoxalate, normal-calcium diet and start therapy with hydrochlorothiazide, 50
mg/d. Two months later, you obtain the following laboratory studies: Serum
sodium 138 meq/L Serum potassium 2.9 meq/L Serum chloride 110 meq/L
Serum bicarbonate 33 meq/L Arterial blood pH 7.43 24-Hour urine studies:
Creatinine 900 mg Calcium 290 mg Oxalate 45 mg Uric acid 540 mg Citrate
356 mg Potassium 45 meq Sodium 225 meq pH 4 What is the most likely
cause of the persistent hypercalciuria?
A. Distal renal tubular acidosis
B. Hyperoxaluria
C. Noncompliance with the normal-calcium diet
D. Surreptitious laxative use
E. * Noncompliance with the low-sodium diet
119. A 39-year-old salesman is admitted for elective right inguinal hernia
repair. He previously underwent left inguinal hernia repair. He has bipolar
disorder, for which he takes lithium carbonate. He also takes a multivitamin
daily. In preparation for surgery, he has received nothing by mouth for the
previous 12 hours. He feels well but is thirsty. On examination, the patient is
alert and in no distress. Blood pressure is 135/85 mm Hg seated and
standing, pulse rate 70/min, respiratory rate 12/min, temperature 36.9 C
(98.4 F). No neck vein distention is present. The lungs are clear. Cardiac
examination shows regular sinus rhythm and no murmur. Abdominal
examination is normal. Right inguinal hernia is present. There is no lower
extremity edema and no evidence of volume depletion. Laboratory studies:
Leukocyte count 7800/?L Hemoglobin 16.5 g/dL Hematocrit 45% Blood urea
nitrogen 18 mg/dL Serum creatinine 1.1 mg/dL Serum sodium 150 meq/L
Serum potassium 4.5 meq/L Serum chloride 112 meq/L Serum bicarbonate 26
meq/L Serum glucose 85 mg/dL Urinalysis Specific gravity 1.006; no
proteinuria, hematuria, or cyturia What is the cause of the elevated serum
sodium level?
A. Syndrome of inappropriate antidiuretic hormone secretion
B. * Renal concentrating defect
C. High dietary sodium intake
D. Fluid restriction
E. No correct answer
120. A 40-year-old man has recurrent nephrolithiasis due to idiopathic
hypercalciuria. He has had more than 40 calcium oxalate stones in the past 5
years. He is started on hydrochlorothiazide therapy and a low-sodium diet.

During treatment, his 24-hour urinary calcium concentration decreases from


385 mg/d to 180 mg/d. No new stones have formed in the past 6
months;however, hypokalemia has developed (serum potassium level, 2.9
meq/L). Taking the hypokalemia into account, what therapy should the
patient receive for hypercalciuric stone disease?
A. High-potassium diet plus hydrochlorothiazide
B. Acetazolamide plus hydrochlorothiazide
C. Magnesium oxide plus hydrochlorothiazide
D. * Amiloride plus hydrochlorothiazide
E. No correct answer
121. A 40-year-old man has recurrent nephrolithiasis due to idiopathic
hypercalciuria. He has had more than 40 calcium oxalate stones in the past 5
years. He is started on hydrochlorothiazide therapy and a low-sodium diet.
During treatment, his 24-hour urinary calcium concentration decreases from
385 mg/d to 180 mg/d. No new stones have formed in the past 6
months;however, hypokalemia has developed (serum potassium level, 2.9
meq/L). Taking the hypokalemia into account, what therapy should the
patient receive for hypercalciuric stone disease?
A. High-potassium diet plus hydrochlorothiazide
B. Acetazolamide plus hydrochlorothiazide
C. Magnesium oxide plus hydrochlorothiazide
D. * Amiloride plus hydrochlorothiazide
E. No correct answer
122. A 40-year-old man has recurrent nephrolithiasis due to idiopathic
hypercalciuria. He has had more than 40 calcium oxalate stones in the past 5
years. He is started on hydrochlorothiazide therapy and a low-sodium diet.
During treatment, his 24-hour urinary calcium concentration decreases from
385 mg/d to 180 mg/d. No new stones have formed in the past 6 months;
however, hypokalemia has developed (serum potassium level, 2.9 meq/L).
Taking the hypokalemia into account, what therapy should the patient receive
for hypercalciuric stone disease?
A. High-potassium diet plus hydrochlorothiazide
B. Acetazolamide plus hydrochlorothiazide
C. Magnesium oxide plus hydrochlorothiazide
D. * Amiloride plus hydrochlorothiazide
E. No correct answer
123. A 40-year-old man has recurrent nephrolithiasis due to idiopathic
hypercalciuria. He has had more than 40 calcium oxalate stones in the past 5
years. He is started on hydrochlorothiazide therapy and a low-sodium diet.
During treatment, his 24-hour urinary calcium concentration decreases from
385 mg/d to 180 mg/d. No new stones have formed in the past 6
months;however, hypokalemia has developed (serum potassium level, 2.9

meq/L). Taking the hypokalemia into account, what therapy should the
patient receive for hypercalciuric stone disease?
A. High-potassium diet plus hydrochlorothiazide
B. Acetazolamide plus hydrochlorothiazide
C. Magnesium oxide plus hydrochlorothiazide
D. * Amiloride plus hydrochlorothiazide
E. No correct answer

124. A 43-year-old woman presents with back pain and is evaluated for renal
insufficiency. Infection with HIV was diagnosed 2 years ago, and the patient
began taking highly active antiretroviral therapy with zidovudine, lamivudine,
and indinavir 1 year later because of a decreasing CD4 count and
development of oral candidiasis. Six months ago, she developed fasting
hyperglycemia and hypercholesterolemia and was treated with rosiglitazone
and atorvastatin. Physical examination reveals a blood pressure of 130/85
mm Hg and a pulse rate of 88/rn in that is regular, with no orthostatic
changes. The respiratory rate is 18/min, and ternperature is 37.8 C (100 F).
There is no neck vein distention or hepatojugular reflux. The cardiac,
pulmonary, and abdominal exarninations are normal, but 2+ lower extremity
ederna is present. Laboratory studies: Blood urea nitrogen 22 mg/dL Serum
sodium 141 rneq/L Serum potassium 6.0 meq/L Serum chloride 101 meq/L
Serum bicarbonate 19 meq/L Serum creatinine 3.2 mg/dL Serum calcium 7.2
mg/dL Serum phosphate 8.3 mg/dL Serum uric acid 9.0 mg/dL Serum total
cholesterol 177 mg/dL Fasting blood glucose and glycosylated hemoglobin
concentrations are elevated. Hematocrit is 31%, with an elevated mean
corpuscular volume. Leukocyte count is 3300/?L, but platelet count is normal.
Urinalysis reveals specific gravity 1.010, trace proteinuria, 2+ hematuria, and
no ketonuria or glycosuria. Microscopic examination shows muddy brown
casts and tubular epithelial cells, but no erythrocytes or crystalluria. What is
the most probable diagnosis?
A. * Rhabdomyolysis caused by atorvastatin therapy
B. Indinavir nephrolithiasis
C. Indinavir tubulointerstitial renal disease and atrophy
D. HIV-associated nephropathy
E. Diabetic nephropathy
125. A 46-year-old man with chronic kidney disease secondary to biopsyproven focal and segmental glomerulosclerosis returns for routine follow-up.
The hematocrit is 28%, and potentially correctable causes of anemia have
been excluded. Therapy with recombinant human erythropoietin is
recommended. In patients with chronic kidney disease and pre-end-stage
renal disease, what is a benefit of therapy with erythropoietin to effectively
treat anemia?
A. Reduced mortality
B. Decreased cardiovascular event rates

C. Normalization of hypertension
D. * Regression of left ventricular hypertrophy
E. No correct answer
126. A 47-year-old man calls Monday morning seeking help with the worst
headache ever Friday night and Saturday. The headache was associated with
severe lethargy and intermittent confusion. He recovered and has felt well for
the past 24 hours. He states that he does not have fever or neurologic or
cardiovascular symptoms. His medical history is significant for hypertension
and recurrent urinary tract infections related to his known autosomal
dominant polycystic kidney disease. He is concerned because his father died
of a stroke during dialysis. The serum creatinine concentration is 2.6 mg/dL.
What do you recommend for this patient?
A. Make an office appointment for him to see you this week
B. Arrange a consultation with the neurology/headache clinic
C. Order computed tomography of the head without contrast
D. * Arrange urgent magnetic resonance angiography of the head
E. No correct answer
127. A 47-year-old man calls Monday morning seeking help with the worst
headache ever Friday night and Saturday. The headache was associated with
severe lethargy and intermittent confusion. He recovered and has felt well for
the past 24 hours. He states that he does not have fever or neurologic or
cardiovascular symptoms. His medical history is significant for hypertension
and recurrent urinary tract infections related to his known autosomal
dominant polycystic kidney disease. He is concerned because his father died
of a stroke during dialysis. The serum creatinine concentration is 2.6 mg/dL.
What do you recommend for this patient?
A. Make an office appointment for him to see you this week
B. Arrange a consultation with the neurology/headache clinic
C. Order computed tomography of the head without contrast
D. * Arrange urgent magnetic resonance angiography of the head
E. No correct answer
128. A 47-year-old man with autosomal dominant polycystic kidney disease
presents with a recurrent urinary tract infection despite therapy with
ampicillin forEscherichia coli infection 3 weeks earlier, which was sensitive to
all antibiotics tested. During that urinary tract infection, there was concern
about an infected cyst, since the patient had right flank discomfort. The
serum creatinine concentration was 1.0 mg/dL, and renal ultrasonography did
not identify obstruction, stones, or abscess. Currently, the patient describes a
3-to 4-day history of dysuria without fever or pain. He is taking no
medications and is allergic to ciprofloxacin. On examination, the patient
appears well and is afebrile. Physical examination is normal, without
tenderness over either polycystic kidney. Urine culture grew E. ccli sensitive
to ampicillin, trimethoprim-sulfamethoxazole, ciprofloxacin, gentamicin, and
ceftriaxone. What would you recommend for this patients urinary tract
infection?

A. * Oral trimethoprim-sulfamethoxazole for several weeks


B. Oral ampicillin at an increased dosage and duration
C. Intravenous therapy with ceftriaxone and gentamicin
D. Indium-labeled leukocyte scanning to detect abscess in polycystic
kidney disease
E. No correct answer
129. A 47-year-old man with autosomal dominant polycystic kidney disease
presents with a recurrent urinary tract infection despite therapy with
ampicillin forEscherichia coli infection 3 weeks earlier, which was sensitive to
all antibiotics tested. During that urinary tract infection, there was concern
about an infected cyst, since the patient had right flank discomfort. The
serum creatinine concentration was 1.0 mg/dL, and renal ultrasonography did
not identify obstruction, stones, or abscess. Currently, the patient describes a
3-to 4-day history of dysuria without fever or pain. He is taking no
medications and is allergic to ciprofloxacin. On examination, the patient
appears well and is afebrile. Physical examination is normal, without
tenderness over either polycystic kidney. Urine culture grew E. ccli sensitive
to ampicillin, trimethoprim-sulfamethoxazole, ciprofloxacin, gentamicin, and
ceftriaxone. What would you recommend for this patients urinary tract
infection?
A. * Oral trimethoprim-sulfamethoxazole for several weeks
B. Oral ampicillin at an increased dosage and duration
C. Intravenous therapy with ceftriaxone and gentamicin
D. Indium-labeled leukocyte scanning to detect abscess in polycystic
kidney disease
E. No correct answer
130. A 48-year-old white male plumber transfers to your practice after a
change of insurance status. His medical history is positive for primary
hypertension without target organ damage. He has no history of renal or
prostatic disease. Laboratory values obtained from his former primary care
physician show normal results for blood urea nitrogen, serum creatinine,
electrolytes, urinalysis, prostate-specific antigen, and electrocardiography. He
takes the ?-blocker doxazosin, 2 mg at bedtime. On examination, blood
pressure is 146/92 mm Hg seated and standing. Body weight is 84 kg (185
Ib). The remainder of the examination is normal. What is the appropriate
course of action regarding the patients antihypertensive therapy?
A. Increase doxazosin to 4 mg
B. Advise high dietary intake of calcium and potassium
C. * Discontinue doxazosin therapy and consider an alternative agent
D. Advise a low-sodium diet
E. No correct answer
131. A 49-year-old man is brought to the emergency department after being
found unresponsive on a city street. His medical history is unknown. The

patient is comatose, with a Glasgow Coma Score of 3. Initial rectal


temperature is 32 C (89.6 F), systolic blood pressure 70mm Hg, respiratory
rate 6/min, and pulse rate 120/min. Funduscopy shows no hemorrhage or
papilledema. The patient has numerous superficial lacerations and
ecchymoses on his extremities. There is no odor of alcoholic beverages. The
remainder of the physical examination is unremarkable. The patient is
emergently intubated and supported aggressively with intravenous fluids.
Shortly thereafter, the patients blood pressure increases to 207/131 mm Hg,
requiring intravenous antihypertensive medication. Urine output is 100 to
200 mL/h. Results of noncontrast computed tomography of the head and
portable chest radiography are normal. Laboratory studies: Hematocrit 41 %
Leukocyte count 32,600/?L Platelet count 422,000/?L Serum sodium 151
meq/L Serum potassium 5.3 meq/L Serum chloride 112 meq/L Serum
bicarbonate 5 meq/L Blood urea nitrogen 11 mg/dL Serum creatinine 1.8
mg/dL Serum glucose 152 mg/dL Serum lactate 4.3 mmol/L Serum osmolality
375 mosmol/kg H2O Arterial blood gaspH 6.8, PCO2 16 mm Hg, Po2 159 mm
Hg, SaO2 99% Urinalysis pH 5.0, specific gravity 1.012, trace glucosuria,
moderate hematuria, trace ketonuria, proteinuria 100 mg/dL Urine
microscopy Numerous erythrocytes No salicylate, acetaminophen, or ethanol
is detected on toxicology screening. Intravenous infusion of bicarbonate is
begun, and the patient is transferred to the medical intensive care unit.
Repeated arterial blood gas analysis shows a pH of 6.8. What is the next most
appropriate step in the management of this patient?
A. Continue bicarbonate supplementation and add insulin to control
blood glucose
B. * Initiate ethanol drip and hemodialysis
C. Institute plasma exchange to treat acidemia
D. Initiate ethanol drip and continue bicarbonate and insulin
supplementation
E. Perform emergency contrast computed tomography of the
abdomen and pelvis
132. A 49-year-old woman is hospitalized because of weakness and
diarrhea. The diarrhea began 2 days ago, in association with coryza,
myalgias, and fever. She has a 4-year history of hypertension that is treated
with valsartan. She had taken ibuprofen for tendinitis until the morning of
admission. Her renal function was previously normal. On physical
examination, the supine blood pressure is 122/72 mm Hg, pulse rate 98/min,
respiratory rate 22/min, and temperature 39.6 C (103.2 F). While standing,
the blood pressure is 90/60 mm Hg and pulse rate is 116/min. There is no
neck vein distention or hepatojugular reflux. Cardiac and chest examinations
are normal. The abdomen is diffusely tender, but there is no rigidity or
rebound and no organomegaly. Gynecologic and rectal examinations show no
mass or tenderness; stool is negative for occult blood. The rest of the
examination is normal. Laboratory studies: Hematocrit 32% Leukocyte count
13,400/?L Platelet count 200,000/?L Blood urea nitrogen 50 mg/dL Serum
sodium 143 meq/L Serum potassium 5.9 meq/L Serum chloride 99 meq/L
Serum bicarbonate 21 meq/L Urine creatinine 185 mgldL Urine sodium 6
meq/L Urinalysis pH 6.0; specific gravity 1.023; no hematuria, proteinuria, or
ketonuria Urine microscopy No formed elements, casts, or debris Which
action is NOT appropriate in the treatment of this patient with acute renal
failure?

A. Discontinue valsartan
B. Discontinue ibuprofen
C. Obtain renal ultrasonography
D. Administer normal saline
E. * Administer acetylcysteine
133. A 52-year-old man is referred by his primary care physician for
hypertension and hypokalemia over the past 6 months. Blood pressure and
routine chemistries were normal last year at the time of an executive
physical. He has no history of cardiovascular disease, stroke, or renal disease.
Family history is negative for hypertension. He uses alcohol socially and does
not smoke but chews tobacco. He takes no medications regularly. On
examination, the patientweighs 77kg (168 Ib). Blood pressure is 164/102mm
Hg seated and standing. Except for trace pedal edema, the remainder of
examination is normal. The primary care physician provides the following
laboratory values: Blood urea nitrogen 21 mg/dL Serum creatinine 0.9 mg/dL
Serum sodium 141 meq/L Serum potassium 3.1 meq/L Serum chloride 100
meq/L Serum bicarbonate 28 meq/L A 24-hour urine test during salt loading
reveals the following values: Creatinine 1.1 g Sodium 252 meq Potassium 128
meq The daily aldosterone excretion rate is 6 mg (normal, 5 to 15 mg),
plasma renin activity is 1 ?g/L/h, and plasma aldosterone level is 9 ng/dL.
Which diagnostic test would you order next?
A. Computed tomography of the adrenal glands
B. * Serum cortisol and urinary free cortisol measurement
C. Magnetic resonance angiography with gadolinium
D. Adrenocorticotropin hormone stimulation test
E. No correct answer
134. A 54-year-old man is admitted with jaundice and edema. He has been
healthy but has a 13-year history of habitual heavy alcohol use and
intermittent binge drinking. He felt well until yesterday, when he experienced
difficulty urinating and dysuria. On physical examination, the blood pressure
is 122/72 mm Hg, without orthostatic changes; heart rate 98/min; respiratory
rate 22/min; and temperature 38.8 C (101 .8 F). Scleral icterus is present,
but there is no neck vein distention or hepatojugular reflux. The cardiac and
chest examinations are normal. The abdomen is distended, but there is no
rigidity or rebound. Bilateral lower extremity edema is present. There is no
asterixis. Laboratory studies: Hematocrit 32% Leukocyte count Normal
Platelet count Normal Blood urea nitrogen 24 mg/dL Serum creatinine 1.9
mg/dL Serum potassium 4.2 meq/L Serum chloride 99 meq/L Serum
bicarbonate 25 meq/L Direct bilirubin 6.5 mg/dL Serum albumin 2.1 g/dL
Urine creatinine 105 mg/dL Urine sodium 12 meq/L Urinalysis pH 6.0; specific
gravity, 1 .023; trace proteinuria; no hematuria or ketonuria Microscopic urine
examination reveals 30 to 50 leukocytes/hpf but no other formed elements,
casts, or debris. Which of the following is true regarding the diagnosis of the
decreased renal function in this patient?
A. The ratio of blood urea nitrogen to creatinine indicates chronic renal
insufficiency

B. Renal ultrasonography is not necessary because the anemia is


consistent with chronic renal disease
C. The urine electrolyte levels are diagnostic of the hepatorenal
syndrome
D. Urine culture and sensitivity testing are critical to establishing the
diagnosis of renal insufficiency
E. The response over the next several days to decreasing total body
sodium overload and maximizing cardiac output will differentiate
prerenal azotemia from the hepatorenal syndrome
135. A 56-year-old black man with diabetic nephropathy is seen in clinic for
routine follow-up. Laboratory studies: Serum calcium 9.6 mg/dL Serum
phosphorus 6.0 mg/dL Serum parathyroid hormone 387 pg/mL Serum
albumin 3.9 dg/L Serum creatinine 2.6 mg/dL Estimated glomeru mar
filtration rate 38 mL/min Because the patient has adhered to a phosphaterestricted diet, phosphate binder therapy with calcium acetate, 667 mg, two
tablets three times daily with meals is begun. Three weeks later, repeated
calcium and phosphorus measurements are 11.9 mg/dL and 5.4 mg/dL,
respectively. What would be the most appropriate action?
A. Refer for parathyroidectomy as definitive therapy for secondary
hyperparathyroidism
B. Discontinue calcium acetate therapy and avoid use of phosphate
binders in the future
C. * Discontinue calcium acetate therapy and, once calcium
normalizes, start sevelamer therapy as a non-calcium-based
phosphate binder
D. Discontinue calcium acetate and, once calcium normalizes, restart
phosphate binder therapy with aluminum hydroxide
E. No correct answer
136. A 56-year-old man was admitted with fever, cough, chest pain, and
leukocytosis. Evaluation revealed sputum with gram-positive diplococci, and
radiography showed a right lower lobe infiltrate. Serum creatinine
concentration was 0.9 mg/dL. He received intravenous penicillin,
defervesced, and was sent home on the second hospital day with a
prescription for a 10-day course of oral penicillin. The patient presents to your
office 1 week later because he feels well but has anorexia and a rash on both
legs. On physical examination, the blood pressure is 130/90 mm Hg, with no
orthostatic changes; pulse rate, 80/min; respiratory rate, 12/min; and
temperature 39.0 C (102.2 F). There is no change in pulse rate on change
in position. There is no neck vein distention or hepatojugular reflux.
Cardiopulmonary examination is normal. No lower extremity edema is
present. A diffuse erythematous macular rash is found on the volar aspects of
both lower extremities from the ankles to the thighs. Laboratory studies:
Hematocrit 39% Leukocyte count 16,300/?L Platelet count Normal Blood urea
nitrogen 46 mg/dL Serum creatinine 3.4 mg/dL Serum potassium 4.3 meq/L
Arterial blood gases pH 7.34, PCO2 32 mm Hg Urinalysis pH 6.0, specific
gravity 1.014, 1+ proteinuria, trace hematuria, no ketonuria Urine
microscopy 30 to 40 leukocytes/hpf What is the most important next step in
the evaluation of the decreased renal function in this patient?

A. * Stop antibiotic therapy and obtain urine culture and sensitivities


B. Perform renal scanning
C. Obtain antineutrophil cytoplasmic antibody serology
D. Request nephrologic consultation for renal biopsy
E. Request nephrologic consultation for dialysis
137. A 56-year-old man was admitted with fever, cough, chest pain, and
leukocytosis. Evaluation revealed sputum with gram-positive diplococci, and
radiography showed a right lower lobe infiltrate. Serum creatinine
concentration was 0.9 mg/dL. He received intravenous penicillin,
defervesced, and was sent home on the second hospital day with a
prescription for a 10-day course of oral penicillin. The patient presents to your
office 1 week later because he feels well but has anorexia and a rash on both
legs. On physical examination, the blood pressure is 130/90 mm Hg, with no
orthostatic changes; pulse rate, 80/min; respiratory rate, 12/min; and
temperature 39.0 C (102.2 F). There is no change in pulse rate on change
in position. There is no neck vein distention or hepatojugular reflux.
Cardiopulmonary examination is normal. No lower extremity edema is
present. A diffuse erythematous macular rash is found on the volar aspects of
both lower extremities from the ankles to the thighs. Laboratory studies:
Hematocrit 39% Leukocyte count 16,300/?L Platelet count Normal Blood urea
nitrogen 46 mg/dL Serum creatinine 3.4 mg/dL Serum potassium 4.3 meq/L
Arterial blood gases pH 7.34, PCO2 32 mm Hg Urinalysis pH 6.0, specific
gravity 1.014, 1+ proteinuria, trace hematuria, no ketonuria Urine
microscopy 30 to 40 leukocytes/hpf What is the most important next step in
the evaluation of the decreased renal function in this patient?
A. * Stop antibiotic therapy and obtain urine culture and sensitivities
B. Perform renal scanning
C. Obtain antineutrophil cytoplasmic antibody serology
D. Request nephrologic consultation for renal biopsy
E. Request nephrologic consultation for dialysis
138. A 56-year-old man with a 25-pack-year smoking history, distant
cerebrovascular accident, and a 10-year history of hypertension treated with
hydrochlorothiazide presents with generalized fatigue. Blood pressure is
110/70mm Hg. Laboratory studies: Serum sodium 128 meq/L Serum
potassium 3.3 meq/L Serum chloride 79 meq/L Serum bicarbonate 38 meq/L
Arterial blood gases on room air pH 7.50, PCO2 250 mm Hg, PO2 74mm Hg
What condition best explains the acid-base disturbance?
A. * Metabolic alkalosis induced by diuretic use
B. Respiratory acidosis induced by chronic obstructive pulmonary
disease
C. Neurogenic-induced respiratory alkalosis
D. Primary hyperaldosteronism
E. No correct answer

139. A 58-year-old black woman presents for routine follow-up of diabetes


mellitus and hypertension. She feels well but states that she stopped taking
verapamil because of constipation. Current medications include glipizide,
pravastatin, and aspirin; evidence of drug intolerance includes angiotensinconverting enzyme inhibitor cough. On examination, blood pressure is 156/92
mm Hg seated and standing. Except for the patients findings for background
diabetic retinopathy, the remainder of the examination is normal. Recent
laboratory values are a serum creatinine concentration of 1.6 mg/dL, 24-hour
urinary protein excretion of 1.5 g/d, and creatinine clearance of 45 mL/min.
On the basis of recent evidence, what is the most efficacious therapy to slow
the progression of the patients type 2 diabetic nephropathy?
A. Angiotensin-converting enzyme inhibitor
B. * Angiotensin receptor blocker
C. Dihydropyridine calcium antagonist
D. -Blocker
E. No correct answer
140. A 58-year-old nun comes to your office because of lethargy, mild
nausea, and weakness for the past 2 weeks. Three years ago, pulmonary
sarcoidosis was diagnosed by biopsy. Three months ago, the patient began
taking oral calcium (1500 mg/d) and 25-hydroxyvitamin D as treatment for
osteoporosis that was diagnosed by screening bone density testing. She has
chronichypertension that is well controlled with metoprolol, 50 mg/d. On
examination, the patient appears thin but well nourished and is in no distress.
She is oriented to time, date, and place. Blood pressure is 140/80 mm Hg,
pulse rate 80/min, temperature 37 C (98.6F). The thyroid is normal, and the
neck veins are not distended. The lungs are clear. Cardiac examination shows
regular sinus rhythm, no murmur, and normal first and second heart sounds.
The abdomen is not tender, the liver and spleen are not palpable, and no
mass is present. There is no edema in the lower extremities, and reflexes are
1 + and symmetrical. Laboratory studies: Hemoglobin 13.8 g/dL Hematocrit
38% Leukocyte count 5600/?L Blood urea nitrogen 24 mg/dL Serum creatinine
2.2 mg/dL (was 1.0 mg/dL 3 months ago) Serum sodium 141 meq/L Serum
potassium 4.4 meq/L Serum chloride 105 meq/L Serum bicarbonate 24 meq/L
Serum calcium 12.8 mg/dL Serum phosphorus 3.5 mg/dL Serum parathyroid
hormone 18 pg/mL Urinalysis pH 5.5; specific gravity 1.010; no proteinuria,
hematuria, or glucosuria; no cells on microscopy Serum and urine
immunoglobulins showed no monoclonal protein. A polyclonal increase in IgG
is present. Renal ultrasonography demonstrates no hydronephrosis and no
calculi. What is the most likely cause of this patients acute renal failure?
A. Myeloma kidney
B. * Acute interstitial nephritis
C. Hypercalcemia
D. Acute glomerulonephritis
E. Bilateral renal artery stenosis
141. A 58-year-old nun comes to your office because of lethargy, mild
nausea, and weakness for the past 2 weeks. Three years ago, pulmonary
sarcoidosis was diagnosed by biopsy. Three months ago, the patient began

taking oral calcium (1500 mg/d) and 25-hydroxyvitamin D as treatment for


osteoporosis that was diagnosed by screening bone density testing. She has
chronichypertension that is well controlled with metoprolol, 50 mg/d. On
examination, the patient appears thin but well nourished and is in no distress.
She is oriented to time, date, and place. Blood pressure is 140/80 mm Hg,
pulse rate 80/min, temperature 37 C (98.6F). The thyroid is normal, and the
neck veins are not distended. The lungs are clear. Cardiac examination shows
regular sinus rhythm, no murmur, and normal first and second heart sounds.
The abdomen is not tender, the liver and spleen are not palpable, and no
mass is present. There is no edema in the lower extremities, and reflexes are
1 + and symmetrical. Laboratory studies: Hemoglobin 13.8 g/dL Hematocrit
38% Leukocyte count 5600/?L Blood urea nitrogen 24 mg/dL Serum creatinine
2.2 mg/dL (was 1.0 mg/dL 3 months ago) Serum sodium 141 meq/L Serum
potassium 4.4 meq/L Serum chloride 105 meq/L Serum bicarbonate 24 meq/L
Serum calcium 12.8 mg/dL Serum phosphorus 3.5 mg/dL Serum parathyroid
hormone 18 pg/mL Urinalysis pH 5.5; specific gravity 1.010; no proteinuria,
hematuria, or glucosuria; no cells on microscopy Serum and urine
immunoglobulins showed no monoclonal protein. A polyclonal increase in IgG
is present. Renal ultrasonography demonstrates no hydronephrosis and no
calculi. What is the most likely cause of this patients acute renal failure?
A. Myeloma kidney
B. * Acute interstitial nephritis
C. Hypercalcemia
D. Acute glomerulonephritis
E. Bilateral renal artery stenosis
142. A 58-year-old nun comes to your office because of lethargy, mild
nausea, and weakness for the past 2 weeks. Three years ago, pulmonary
sarcoidosis was diagnosed by biopsy. Three months ago, the patient began
taking oral calcium (1500 mg/d) and 25-hydroxyvitamin D as treatment for
osteoporosis that was diagnosed by screening bone density testing. She has
chronic hypertension that is well controlled with metoprolol, 50 mg/d. On
examination, the patient appears thin but well nourished and is in no distress.
She is oriented to time, date, and place. Blood pressure is 140/80 mm Hg,
pulse rate 80/min, temperature 37 C (98.6F). The thyroid is normal, and the
neck veins are not distended. The lungs are clear. Cardiac examination shows
regular sinus rhythm, no murmur, and normal first and second heart sounds.
The abdomen is not tender, the liver and spleen are not palpable, and no
mass is present. There is no edema in the lower extremities, and reflexes are
1 + and symmetrical. Laboratory studies: Hemoglobin 13.8 g/dL Hematocrit
38% Leukocyte count 5600/?L Blood urea nitrogen 24 mg/dL Serum creatinine
2.2 mg/dL (was 1.0 mg/dL 3 months ago) Serum sodium 141 meq/L Serum
potassium 4.4 meq/L Serum chloride 105 meq/L Serum bicarbonate 24 meq/L
Serum calcium 12.8 mg/dL Serum phosphorus 3.5 mg/dL Serum parathyroid
hormone 18 pg/mL Urinalysis pH 5.5; specific gravity 1.010; no proteinuria,
hematuria, or glucosuria; no cells on microscopy Serum and urine
immunoglobulins showed no monoclonal protein. A polyclonal increase in IgG
is present. Renal ultrasonography demonstrates no hydronephrosis and no
calculi. What is the most likely cause of this patients acute renal failure?
A. Myeloma kidney
B. Acute interstitial nephritis

C. * Hypercalcemia
D. Acute glomerulonephritis
E. Bilateral renal artery stenosis
143. A 58-year-old woman with a 4-year history of type 2 diabetes mellitus
is evaluated in the emergency department for weakness. Six months ago, her
serum creatinine concentration was 1.0 mg/dL. She now has polydipsia and
polyuria. On physical examination, blood pressure is 120/60 mm Hg and heart
rate is 98/min while supine; blood pressure is 108/50 mm Hg and heart rate
was 112/min standing. The chest is clear, and cardiac examination is normal.
The remainder of the examination is unremarkable. Laboratory studies: Blood
urea nitrogen 32 mg/dL Serum creatinine 1.6 mg/dL Serum sodium 148
meq/L Serum potassium 3.2 meq/L Serum chloride 99 meq/L Serum
bicarbonate 19 meq/L Serum glucose 405 mg/dL Urine creatinine 35 mg/dL
Urine sodium 76 meq/L Urinalysis Specific gravity, 1.009; no hematuria; trace
proteinuria; 1 + glucosuria; 1 + ketonuria Microscopic urine examination was
unremarkable. Which statement is true regarding the evaluation of this
patient with renal insufficiency?
A. The fractional excretion of sodium (FE Na) is incompatible with a
diagnosis of prerenal azotemia
B. * The FE Na is due to glycosuria
C. The history, physical examination, and laboratory evaluation are
consistent with chronic renal insufficiency secondary to diabetic
nephropathy
D. The FE Na is incompatible with a diagnosis of urinary tract
obstruction
E. The FE Na is most reliable in evaluation of acute renal failure if
oliguria is not present
144. A 59-year-old man presents to the emergency department with a 3-day
history of worsening weakness, decreased mental acuity and responsiveness,
and slurred speech. The patient had been experiencing worsening weakness
over the past 6 months. The patients medical history includes bipolar
disorder (diagnosed 10 years ago) and hypothyroidism (diagnosed 5 years
ago). His medications are lithium, 300 mg/d, and levothyrroxine, 150 ?g/d.
The patient is disoriented and lethargic, with slurred speech and periods of
agitation. A fine tremor and hyperreflexia are present. On physical
examination, supine blood pressure is 148/79 mm Hg, pulse rate 101/min,
respiratory rate 16/min, temperature 37.7 C (99.9 F). While he is standing,
his blood pressure is 142/80 mm Hg and heart rate is 108/min. The mucous
membranes are dry, and the neck veins are flat. Cardiac, pulmonary, and
abdominal examinations are normal. No lower extremity edema is noted.
Laboratory studies: Blood urea nitrogen 82 mg/dL Serum creatinine 9.2 mg/dL
Serum sodium 162 meq/L Serum potassium 6.7 meq/L Serum chloride 131
meq/L Serum bicarbonate 17.2 meq/L Serum calcium 10.7 mg/dL Serum
albumin 4.5 g/dL Serum lithium 4.5 meq/L Serum thyroid-stimulating
hormone
A. Begin intravenous infusion of normal saline for volume repletion
B. Administer 1 ampule of dextrose and 10 U of insulin intravenously
for hyperkalemia

C. Transfer to the intensive care unit and perform emergent peritoneal


dialysis
D. Begin intravenous infusion of half-normal saline followed by 80 mg
of furosemide intravenously for hypercalcemia
E. * Transfer to the intensive care unit and perform emergent
hemodialysis
145. A 60-year-old woman with a history of essential hypertension is
admitted to hospital after 7 days of severe vomiting. On presentation, she
appears ill. The systolic blood pressure is 110 mm Hg seated and 70 mm Hg
standing. The pulse rate while seated is 120/min. Abdominal examination
reveals rebound tenderness and no bowel sounds. Laboratory studies: Blood
urea nitrogen 90 mg/dL Serum creatinine 3 mg/dL Serum sodium 140 meq/L
Serum potassium 3.2 meq/L Serum chloride 80 meq/L Serum bicarbonate 11
meq/L Serum glucose 90 mg/dL Arterial blood gasespH 7.29, PCO2 24 mm Hg
What state does the patients acid-base status indicate?
A. Non-anion gap metabolic acidosis
B. Anion gap metabolic acidosis
C. * Anion gap metabolic acidosis and metabolic alkalosis
D. Anion gap metabolic acidosis and respiratory alkalosis
E. No correct answer
146. A 60-year-old woman with adult polycystic kidney disease is seen
urgently in the office for high fever. The illness started abruptly and involves
chills and dysuria. She has had hypertension for the past 5 years, treated
with quinapril and hydrochlorothiazide. She has lost approximately 8 kg (17
Ib) of weight over the last 3 months. On physical examination, the patient
appears thin and frail. Body weight is 48 kg (106 Ib). Blood pressure is 90/70
mm Hg, pulse rate is 110/min, respiratory rate is 24/min, and body
temperature is 39 C (102 F). The kidneys are palpable bilaterally, and she
has right costovertebral angle tenderness. Serum creatinine concentration is
1.1 mg/dL, and urinalysis shows pyuria and bacteriuria. The patient is
admitted and prescribed intravenous ampicillin and gentamicin to treat
pyelonephritis. Why does the dosage of antibiotic need to be adjusted in this
patient?
A. The infection is in a cyst
B. * The glomerular filtration rate is reduced
C. The patient is septic
D. The patient has hypertension
E. No correct answer
147. A 61-year-old woman with a previously normal serum creatinine
concentration is admitted with abdominal pain. Abdominal aortic aneurysm
was diagnosed after an intravenous contrast study, and the patient
underwent emergency aneurysmectomy. On the third hospital day, in the
intensive care unit, the patient was oliguric. She was given an intravenous
furosemide infusion but was still oliguric several hours later. On physical
examination, the blood pressure is 90/62 mm Hg, with no orthostatic

changes; pulse rate 11 5/min; respiratory rate 22/min; and temperature 36.8
C (98.2 F). Three-fingerbreadth neck-vein distention at 45 degrees and
hepatojugular reflux are present. Cardiac examination shows an S3 gallop.
There are scattered bibasilar rales. The abdomen has a fresh surgical scar.
Bowel sounds are not heard, and there is diffuse tenderness. The patient has
2+ lower extremity edema. She is arousable but somnolent and moves all
extremities in response to commands. She complains of dyspnea. Laboratory
studies: Hematocrit 37% Leukocyte count Leukocytosis Platelet count Low
Blood urea nitrogen 75 mg/dL Serum creatinine 4.4 mg/dL Serum sodium 130
meq/L Serum potassium 6.3 meq/L Serum chloride 90 meq/L Serum
bicarbonate 16 meq/L Arterial blood gaspH 7.26, Pco2 25 mm Hg, Po2 65 mm
Hg Urinalysis pH 6.0, specific gravity 1.009, 2+ proteinuria, no hematuria or
ketonuria There are muddy brown casts on microscopic examination. The
electrocardiogram shows prominent, peaked T waves. What is the next step
in treatment of this patient with acute renal failure?
A. Administer fenoldopam
B. Start ultrafiltration
C. * Start continuous venovenous hemofiltration
D. Start peritoneal dialysis
E. Initiate plasma exchange
148. A 61-year-old woman with a previously normal serum creatinine
concentration is admitted with abdominal pain. Abdominal aortic aneurysm
was diagnosed after an intravenous contrast study, and the patient
underwent emergency aneurysmectomy. On the third hospital day, in the
intensive care unit, the patient was oliguric. She was given an intravenous
furosemide infusion but was still oliguric several hours later. On physical
examination, the blood pressure is 90/62 mm Hg, with no orthostatic
changes; pulse rate 11 5/min; respiratory rate 22/min; and temperature 36.8
C (98.2 F). Three-fingerbreadth neck-vein distention at 45 degrees and
hepatojugular reflux are present. Cardiac examination shows an S3 gallop.
There are scattered bibasilar rales. The abdomen has a fresh surgical scar.
Bowel sounds are not heard, and there is diffuse tenderness. The patient has
2+ lower extremity edema. She is arousable but somnolent and moves all
extremities in response to commands. She complains of dyspnea. Laboratory
studies: Hematocrit 37% Leukocyte count Leukocytosis Platelet count Low
Blood urea nitrogen 75 mg/dL Serum creatinine 4.4 mg/dL Serum sodium 130
meq/L Serum potassium 6.3 meq/L Serum chloride 90 meq/L Serum
bicarbonate 16 meq/L Arterial blood gaspH 7.26, Pco2 25 mm Hg, Po2 65 mm
Hg Urinalysis pH 6.0, specific gravity 1.009, 2+ proteinuria, no hematuria or
ketonuria There are muddy brown casts on microscopic examination. The
electrocardiogram shows prominent, peaked T waves. What is the next step
in treatment of this patient with acute renal failure?
A. Administer fenoldopam
B. Start ultrafiltration
C. * Start continuous venovenous hemofiltration
D. Start peritoneal dialysis
E. Initiate plasma exchange

149. A 61-year-old woman with diabetes mellitus for 6 years is admitted


with headache and disorientation. Her glucose level has been controlled by
diet and exercise. Neurologic examination is nonfocal, volume status is
normal, and serum creatinine concentration is 1.2 mg/dL. Mucormycosis
sinusitis is diagnosed after noncontrast computed tomography and lumbar
puncture studies. She is treated with amphotericin B for 9 days. On physical
examination, the patient is somnolent but arousable and in pain. The blood
pressure is 124/72 mm Hg, with no orthostatic changes; pulse rate 75/min;
respiratory rate 18/min; and temperature 37.8 C (100 F). There is no
evidence of retinopathy. No neck vein distention or hepatojugular reflux is
present. The cardiac examination is normal. The left lower lung field shows
scattered basilar crackles. The abdominal examination is normal. No lower
extremity edema is present. Cranial nerves are normal. Laboratory studies:
Blood urea nitrogen 32 mg/dL Serum creatinine 2.4 mg/dL Serum sodium 147
meq/L Serum potassium 3.2 meq/L Serum chloride 109 meq/L Serum
bicarbonate 15 meq/L Serum glucose 255 mg/dL Hematocrit is 34%, and
leukocytosis is present with a normal platelet count. Urinalysis shows a pH of
7.0 and 1 + proteinuria, but no hematuria or ketonuria. Scattered epithelial
cells and cellular casts appear in most high-power fields on microscopic
examination. What is the appropriate option at this point?
A. Obtain amphotericin level
B. Reduce dose of amphotericin
C. * Administer liposomal amphotericin
D. Discontinue amphotericin therapy
E. No correct answer
150. A 62-year-old man with a nonhealing diabetic ulcer is seen in the
hospital for preoperative clearance. He has a 10-year history of diabetes,
hypertension, and severe peripheral vascular disease. He receives Humulin N
insulin twice daily (30 U every morning and evening); amlodipine, 10 mg/d;
and aspirin, 81 mg/d. On physical examination, body weight is 70 kg (154 Ib),
height is 160 cm (62). Blood pressure is 140/90 mm Hg. No cardiopulmonary
abnormality and no volume overload are detected. Two weeks earlier, his
serum creatinine concentration was 1.4 mg/dL. Urinalysis reveals a specific
gravity of 1.015, 1+ hematuria, no proteinuria, and no glucosuria. What is the
most appropriate method to evaluate this patients renal function?
A. Measure the serum creatinine
B. Perform 24-hour urine collection to assess creatinine and volume
C. Perform technetium-99m-diethylenetriam me pentaacetic acid renal
flow scanning
D. Measure the fractional excretion of sodium
E. * Estimate by using a creatinine-based formula
151. A 62-year-old woman with coronary artery disease and atherosclerotic
peripheral vascular disease is hospitalized because of pain in her left leg. She
has had hypertension for 17 years and hypercholesterolemia for 13 years,
both of which are well controlled by various medications. On physical
examination, the pulse rate is 90/min and regular, and blood pressure is
148/94 mm Hg. The chest is clear. No murmurs or gallops are heard. The

abdomen is not tender. There is trace bilateral lower extremity edema. The
left leg is cooler than the right, and no popliteal or dorsalis pedis pulse is
detected. Blood urea nitrogen is 29 mg/dL, and serum creatinine is 1.4
mg/dL. Urinalysis shows a specific gravity of 1.018, trace protein, and no
glucose or ketones. Microscopic examination of the urine is normal. The
patient undergoes arteriography with a limited amount of iopamidole and
receives acetylcysteine and hydration. Laboratory tests are ordered for the
next day. On physical examination, her pulse rate is 98/min and regular, and
blood pressure is 142/90 mm Hg. Chest, abdominal, and cardiac
examinations are normal. The lower extremities are unchanged. Laboratory
studies: Blood urea nitrogen 43 mg/dL Serum creatinine 1.9 mg/dL Serum
sodium 141 meq/L Serum potassium 3.7 meq/L Serum chloride 100 meq/L
Serum bicarbonate 21 meq/L Urinalysis Specific gravity 1 .009; trace
proteinuria; no glucosuria, ketonuria, or hematuria Urine microscopy Tubular
epithelial cells, rare granular casts Which of the following should be done?
A. Administer dopamine
B. Administer bicarbonate
C. Administer half-normal saline and readminister acetylcysteine
D. Start dialysis to clear contrast medium
E. * Observe for complications of acute renal failure
152. A 62-year-old woman with coronary artery disease and atherosclerotic
peripheral vascular disease is hospitalized because of pain in her left leg. She
has had hypertension for 17 years and hypercholesterolemia for 13 years,
both of which are well controlled by various medications. On physical
examination, the pulse rate is 90/min and regular, and blood pressure is
148/94 mm Hg. The chest is clear. No murmurs or gallops are heard. The
abdomen is not tender. There is trace bilateral lower extremity edema. The
left leg is cooler than the right, and no popliteal or dorsalis pedis pulse is
detected. Blood urea nitrogen is 29 mg/dL, and serum creatinine is 1.4
mg/dL. Urinalysis shows a specific gravity of 1.018, trace protein, and no
glucose or ketones. Microscopic examination of the urine is normal. The
patient undergoes arteriography with a limited amount of iopamidole and
receives acetylcysteine and hydration. Laboratory tests are ordered for the
next day. On physical examination, her pulse rate is 98/min and regular, and
blood pressure is 142/90 mm Hg. Chest, abdominal, and cardiac
examinations are normal. The lower extremities are unchanged. Laboratory
studies: Blood urea nitrogen 43 mg/dL Serum creatinine 1.9 mg/dL Serum
sodium 141 meq/L Serum potassium 3.7 meq/L Serum chloride 100 meq/L
Serum bicarbonate 21 meq/L Urinalysis Specific gravity 1 .009; trace
proteinuria; no glucosuria, ketonuria, or hematuria Urine microscopy Tubular
epithelial cells, rare granular casts Which of the following should be done?
A. Administer dopamine
B. Administer bicarbonate
C. Administer half-normal saline and readminister acetylcysteine
D. Start dialysis to clear contrast medium
E. * Observe for complications of acute renal failure

153. A 63-year-old male executive with hypertension requests a conference


with you to discuss the results of laboratory studies required by his
companys health insurers. The report states that he has 1 + proteinuria, a
serum creatinine concentration of 1 .6 mg/dL, and a calculated glomerular
filtration rate of 48 mL/min, which are consistent with stage III chronic kidney
disease. In doing his own research, the patient has come across the National
Kidney Foundations efforts to publicize the increasing incidence and
prevalence of chronic kidney disease. He requests more information from you
on the prevalence of chronic kidney disease. Which of the following
statements about chronic kidney disease in the United States is true?
A. * Nearly 0.5 million patients have end-stage renal disease (i.e.,
requiring dialysis or transplantation)
B. Approximately 5% of the adult population has chronic kidney
disease, as manifested by any combination of microalbuminuria,
clinical proteinuria, or glomerular filtration rate less than 60 mL/min
C. Approximately 1 .5 million adults have an elevated serum creatinine
concentration of 1 .5 mg/dL or greater
D. Approximately 3% of the adult population has abnormal urinary
protein excretion, defined as microalbuminuria or clinical
proteinuria
154. A 63-year-old man is admitted with acute somnolence, disorientation,
and right upper and lower extremity weakness. He has a 3-year history of
renal insufficiency and congestive heart failure attributed to long-standing
hypertension, which has been poorly controlled in part because of poor
adherence. On physical examination, the blood pressure is 160/96 mm Hg,
pulse rate 11 0/min, respiratory rate 14/min, and temperature 38 C (100.4
F). There is three-fingerbreadth neck vein distention while sitting and
hepatojugular reflux. Cardiac examination shows an S3 gallop; pulmonary
examination reveals bilateral crackles. The abdomen is benign, and 2+ lower
extremity edema is present. Right biceps and patellar reflexes are increased,
and a right Babinski response is noted. Laboratory studies: Hematocrit 33%
Leukocyte count 10,700/?L Platelet count Normal Blood urea nitrogen 35
mg/dL Serum creatinine 2.3 mg/dL Serum sodium 128 meq/L Serum
potassium 4.3 meq/L Serum chloride 93 meq/L Serum bicarbonate 16 meq/L
Arterial blood gas pH 7.30, Pco2 33 mm Hg Urinalysis pH 6.0, specific gravity
1.014, 2+ proteinuria, no hematuria or ketonuria; no formed elements on
microscopy Noncontrast computed tomography of the head shows only an illdefined mass effect. Neurology and neurosurgical consultants request
contrast computed tomography to more precisely demarcate the suspected
tumor and thus pinpoint a site for biopsy or resection. What is the most
important next step to prepare this patient for a contrast study?
A. * Administer acetylcysteine
B. Administer half-normal saline
C. Administer dopamine
D. Administer bicarbonate
E. Administer calcium channel blocker
155. A 63-year-old man is admitted with acute somnolence, disorientation,
and right upper and lower extremity weakness. He has a 3-year history of

renal insufficiency and congestive heart failure attributed to long-standing


hypertension, which has been poorly controlled in part because of poor
adherence. On physical examination, the blood pressure is 160/96 mm Hg,
pulse rate 11 0/min, respiratory rate 14/min, and temperature 38 C (100.4
F). There is three-fingerbreadth neck vein distention while sitting and
hepatojugular reflux. Cardiac examination shows an S3 gallop; pulmonary
examination reveals bilateral crackles. The abdomen is benign, and 2+ lower
extremity edema is present. Right biceps and patellar reflexes are increased,
and a right Babinski response is noted. Laboratory studies: Hematocrit 33%
Leukocyte count 10,700/?L Platelet count Normal Blood urea nitrogen 35
mg/dL Serum creatinine 2.3 mg/dL Serum sodium 128 meq/L Serum
potassium 4.3 meq/L Serum chloride 93 meq/L Serum bicarbonate 16 meq/L
Arterial blood gas pH 7.30, Pco2 33 mm Hg Urinalysis pH 6.0, specific gravity
1.014, 2+ proteinuria, no hematuria or ketonuria; no formed elements on
microscopy Noncontrast computed tomography of the head shows only an illdefined mass effect. Neurology and neurosurgical consultants request
contrast computed tomography to more precisely demarcate the suspected
tumor and thus pinpoint a site for biopsy or resection. What is the most
important next step to prepare this patient for a contrast study?
A. * Administer acetylcysteine
B. Administer half-normal saline
C. Administer dopamine
D. Administer bicarbonate
E. Administer calcium channel blocker
156. A 63-year-old man is hospitalized with chest pain. The patient has had
hypercholesterolemia for 10 years and hypertension for 8 years. He has been
treated most recently with atorvastatin, furosemide, and losartan. His last
serum creatinine concentration as an outpatient 2 months ago was 0.9
mgldL. Evaluation in the cardiac care unit with coronary angiography
revealed right and left coronary artery disease, and he underwent emergent
percutaneous angioplasty and stenting of the involved coronary arteries.
During the procedure, he developed chest pain, and a dissection of the right
coronary artery was noted, along with acute increased ST-segments in the
inferior leads. He underwent immediate coronary artery bypass. On the day
after the procedure, he is alert and oriented. His pulse rate is 1 06/min supine
and 1 08/min seated. Blood pressure is 96/70 mm Hg supine and 100/75 mm
Hg seated. Neck vein distention is not noted when the patient is lying flat,
and there is no hepatojugular reflux. The chest is clear. No murmur or gallop
is present. The abdomen lacks rebound and rigidity. There is no abdominal
bruit and no sacral or lower extremity edema. Distal pulses and the skin of
the lower extremity digits are normal. Laboratory studies: Blood urea nitrogen
30 mg/dL Serum creatinine 1.9 mg/dL Serum sodium 145 meq/L Serum
potassium 3.4 meq/L Serum chloride 109 meq/L Serum bicarbonate 21 meq/L
Urinalysis Specific gravity 1.013, trace proteinuria, trace ketonuria, no
glucosuria Urine microscopy Rare tubular cells, muddy brown casts, cellular
debris Urinary sodium 36 meq/L Urinary creatininel3 mg/dL Urine osmolality
110 mosmol/kg H2O What is the correct therapy?
A. Administer endothelin antagonist
B. Administer insulin-like growth factor

C. Administer low-dose dopamine


D. Administer thyroxine
E. * Observe
157. A 64-year-old black woman has had hypertension for 25 years. On
therapy, her blood pressure has been in the range of 140 to 160/95 to 100
mm Hg. She presents for blood pressure management. Review of past
laboratory data shows that the serum creatinine concentration was 1 .2
mg/dL 10 years ago, 1.7 mg/dL 5 years ago, 2.0 mg/dL 1 year ago, and 1.9
mg/dL 2 months ago. Recent urinalysis shows 2+ proteinuria, no hematuria,
and occasional granular and hyaline casts. Urine protein:creatinine ratio is
0.5. Renal ultrasonography shows no hydronephrosis with kidney sizes at 9.5
cm bilaterally. On examination, body weight is 84 kg (185.2 Ib) and
temperature is 36.9 C (98.4F). Blood pressure is 148/96 mm Hg in both
arms. Optic funduscopy shows moderate arteriolar sclerosis and constriction.
No jugulovenous distention is present. The lungs are clear. Cardiac
examination shows regular sinus rhythm, S4 but no S3, and no murmur.
There is no edema of the extremities. What is the most likely cause of this
patients renal disease?
A. Membranous glomerulopathy
B. Obstructive uropathy
C. IgA nephritis
D. * Nephrosclerosis
E. Obesity-related glomerular disease
158. A 64-year-old man is admitted with a 5-day history of lethargy and mild
confusion. He is known to have alcoholic cirrhosis, nonbleeding esophageal
varices, and ascites. There is no history of recent alcohol consumption,
melena, or hematemesis. He has no abdominal pain and had not fallen. He
takes a 2-g sodium diet and multivitamins daily. On examination, the patient
is lethargic and confused to time and place but not date. Blood pressure is
110/70 mm Hg, pulse rate 87/min, temperature 36 C (96.8 F). Icteric
sclerae and spider angiomata are present. The neck veins are not distended.
The lungs are clear, with decreased breath sounds at both bases. Cardiac
examination reveals regular sinus rhythm and no gallop or rub. The abdomen
is protuberant but nontender, with a shifting dullness; the liver is not
palpable. The lower extremities have 1 + ankle edema. Asterixis is present,
but the patient has no focal neurologic signs. Laboratory studies: Hemoglobin
11.5 g/dL Hematocrit 32% Leukocyte count 5400/?L Platelet count 84,000/?L
Blood urea nitrogen 20 mg/dL Serum creatinine 1.2 mg/dL Serum sodium 114
meq/L Serum potassium 4.1 meq/L Serum chloride 80 meq/L Serum
bicarbonate 28 meq/L Serum total protein 6.9 g/dL Serum albumin 2.5 g/dL
Cholesterol 186 mg/dL Serum osmolality 241 mosmol/kg H2O Urine
osmolality 200 mosmol/kg H2O Spot urine sodium 10 meq/L What is the
cause of this patients hyponatremia?
A. Nonosmotic stimulation of antidiuretic hormone
B. Hepatorenal syndrome
C. * Low-sodium diet

D. Reset osmostat
E. Pseudohyponatremia
159. A 65-year-old man who is known to have alcoholism is transferred from
a local jail to the hospital because of generalized weakness and a witnessed
seizure 1 hour ago, shortly after he was arrested for vagrancy. In the
emergency department, he is lethargic but conversant and oriented. He
reports a several-day history of diarrhea and has muscle cramps. He has no
history of trauma or previous seizures. He is taking no medications but has
smoked 1 pack of cigarettes daily for the past 30 years. On physical
examination, blood pressure is 110/75 mm Hg, pulse rate is 100/min, and
respiratory rate is 18/min. The neck is supple, and the carotids are normal.
The lungs are clear, and cardiac examination shows regular sinus rhythm and
a G1-2/6 systolic murmur at the base. The abdomen is soft, with bowel
sounds; the edge of the liver is palpable, but the spleen is not. Cranial nerves
are normal, and the patient has 3+ reflexes. Carpal pedal spasm is noted
intermittently during examination, and Chvosteks sign is present. Laboratory
studies: Leukocyte count 9500/?L Hemoglobin 12 g/dL Hematocrit 37% Blood
urea nitrogen 35 mg/dL Serum creatinine 1.4 mg/dL Serum sodium 136
meq/L Serum potassium 2.7 meq/L Serum chloride 98 meq/L Serum
bicarbonate 23 meq/L Serum calcium 7.6 mg/dL Serum magnesium 0.5
mg/dL Serum phosphorus 3.0 mg/dL Serum total protein 7.2 g/dL Serum
albumin 3.5 g/dL Urinalysis Specific gravity 1.025; no proteinuria or
hematuria; 1-3 erythrocytes/hpf and 5-7 leukocytes/hpf What is the most
appropriate therapy for this patients electrolyte disorder?
A. Intravenous magnesium sulfate
B. * Intravenous magnesium sulfate and potassium chloride
C. Oral magnesium and potassium chloride
D. Intravenous potassium chloride
E. Intravenous calcium
160. A 66-year-old woman is hospitalized after a right hip fracture requiring
open reduction and internal fixation. She received intravenous hydration
through postoperative day 3. On presentation, her blood pressure was 160/90
mm Hg. Three years earlier, her blood pressure at a routine office visit was
128/82mm Hg. The patient is discharged on postoperative day 4 to an
inpatient extended care center. On postoperative day 10, a consultation is
obtained to help manage her persistent hypertension. Her only medication is
celecoxib. Laboratory studies: Value At presentation On postoperative day 2
On postoperative day 10 Blood pressure 160/90 mmHg 180/104 mmHg
160/95 mmHg Serum sodium 141 meq/L 142 meq/L 141 meq/L Serum
potassium 3.0 meq/L 2.4 meq/L 2.9 meq/L Serum chloride 100 meq/L 98
meq/L 99 meq/L Serum bicarbonate 31 meq/L 32 meq/L 32 meq/L Arterial
blood gas-pH 7.46, Pco2 46 mm Hg What condition best explains the patients
status?
A. Essential hypertension
B. Pheochromocytoma
C. Hypertension induced by use of nonsteroidal anti-inflammatory
drugs

D. * Primary hyperaldosteronism
E. No correct answer
161. A 67-year-old retired nurse presents because of a 6-month history of
gradual-onset dementia. Aside from loss of recent memory and confusion
about time and place, she has no symptoms or complaints. She has
hypothyroidism that is treated with levothyroxine, 100 ?/d, and hypertension
treated with amlodipine, 5 mg/d. She also takes estrogen therapy. She has
smoked 1 to 2 packs of cigarettes daily for the past 35 years, and she drinks
less than 1 ounce of alcohol weekly. On examination, the patients demeanor
is pleasant. She is in no distress, but she is disoriented to time and place.
Blood pressure is 142/88 mm Hg seated and 135/85 mm Hg, pulse rate
68/min, respiratory rate 12/min, temperature 37 C (98.6 F). There is no
neck vein distention, and the carotids are normal. The lungs are clear. Cardiac
examination reveals regular sinus rhythm, a grade 1/6 systolic murmur at the
base, and no gallop. Abdominal examination is normal. There is no edema,
and lower extremity pulses are present and normal. Neurologic examination
is normal. Laboratory studies: Complete blood count Normal Plasma glucose
84 mg/dL Blood urea nitrogen 6 mg/dL Serum creatinine 0.5 mg/dL Serum
sodium 124 meq/L Serum potassium 4.2 meq/L Serum chloride 89 meq/L
Serum bicarbonate 24 meq/L Serum thyroid-stimulating hormone 3.2 mIU/L
Serum uric acid 2.3 mg/dL Serum cholesterol 182 mg/dL Serum triglyceride
60 mg/dL Serum total protein 7.5 g/dL Serum albumin 3.8 g/dL Serum
osmolality 255 mosmol/kg H2O Urinalysis Specific gravity 1.030; no
hematuria or proteinuria What is the most likely cause of this patients
hyponatremia?
A. * Syndrome of inappropriate antidiuretic hormone secretion
B. Pseudohyponatremia
C. Surreptitious diuretic use
D. Cryptogenic cirrhosis
E. Psychogenic polydipsia
162. A 69-year-old white man is referred for worsening hypertension over
the past 2 to 3 months. He has had hypertension for the past 18 months. It
had been controlled by ?-blocker therapy, which was begun after he had an
inferior myocardial infarction. A recent blood pressure measurement was
200/120mm Hg, requiring additional therapy with amlodipine. His medical
history is significant for the myocardial infarction and a right femoral popliteal
bypass. He smokes two packs of cigarettes daily and drinks alcohol socially.
On examination, blood pressure is 178/104 mm Hg seated and standing, and
body weight is 72 kg (159 Ib). Optic funduscopy reveals background
hypertensive retinopathy. A left carotid bruit is heard. Cardiopulmonary and
neuromuscular examinations are normal. Abdominal examination showed no
organomegaly, but an epigastric bruit is present. No peripheral edema is
noted. Serum creatinine concentration is 2.3 meq/dL, and serum potassium
concentration is 3.9 mg/dL. Urinalysis shows 1 + proteinuria without
hematuria. Electrocardiography is positive for left ventricular hype rtrophy.
What is the most appropriate noninvasive screening test for possible renal
artery stenosis in this patient?
A. * Magnetic resonance angiography with gadolinium
B. Computed tomographic angiography with contrast

C. Captopril renography
D. Captopril plasma renin activity test
E. No correct answer
163. A 70-year-old woman is admitted because she has had malaise and
anorexia for 1 week. She has been previously healthy, except for
hypertension and hypercholesterolemia, treated with hydrochlorothiazide and
atorvastatin. On physical examination, the supine blood pressure is 150/95
mm Hg, pulse rate 80/min, respiratory rate 20/min, and temperature 37.4 C
(99.3 F). The blood pressure is 125/80 mm Hg and the pulse rate 96/min
while standing. There is no neck vein distention or hepatojugular reflux.
Cardiac, breast, abdominal, and pulmonary examinations are normal. No
lower extremity edema is present. Laboratory studies: Hematocrit 29%
Leukocyte count 3,200/?L Platelet count 90,000/?L Blood urea nitrogen 62
mg/dL Serum creatinine 4.6 mg/dL Serum sodium 134 meq/L Serum
potassium 5.0 meq/L Serum chloride 114 meq/L Serum bicarbonate 15 meq/L
Serum glucose 105 mg/dL Serum calcium 12.5 mg/dL Serum inorganic
phosphate 8.5 mg/dL Urine creatinine 25 mg/dL Urine sodium 50 meq/L
Urinalysis Specific gravity 1 .007; trace proteinuria; no glucosuria or ketonuria
Arterial blood gaspH 7.30, PCO2 28 mm Hg Microscopic analysis shows
scattered tubular epithelial cells. Posteroanterior and lateral films of the chest
are normal. What is the most likely diagnosis?
A. Milk-alkali syndrome
B. Sarcoidosis
C. * Multiple myeloma
D. Primary hyperparathyroidism
E. Clinical consequence of hydrochlorothiazide therapy
164. A 72-year-old white man returns for a follow-up visit subsequent to
admission to another hospital for hypertension. He was seen in the local
emergency department for severe musculoskeletal back pain, where the
treating physician noted elevated blood pressure (200/92 mm Hg) and a
serum creatinine concentration of 1.6 mg/dL. Results of other laboratory tests
were normal. The patient was admitted for evaluation and management of
back pain, and the attending physician obtained additional studies relating to
the patients hypertension. Renal ultrasonography was negative for calculus,
mass, or obstruction; kidney size was 11 cm on the right and 12 cm on the
left. Renal artery duplex ultrasonography was suggestive of right renal artery
stenosis. Renal angiography revealed a normal left renal artery and 50%
stenosis in the right renal artery. Analysis of renal vein renin activity showed
a low inferior vena cava value of 1.5 ?g/L/h, right renal vein value of 2.0 ?
g/L/h, left renal vein value of 2.0 ?g/L/h, and a high inferior vena cava value
of 2.0 ?g/L/h. The plasma renin activity is 1 .0 mg/LIh, and the plasma
aldosterone level is 8.0 ng/dL.The thyroid-stimulating hormone level is 1.0 ?
U/mL. Review of your office records confirms that the patient has a 22-year
history of hypertension controlled with a ?-blocker and diuretic therapy. The
serum creatinine concentration has been stable at 1.6 mgldL for more than 3
years, and urinalysis shows 1 + proteinuria. What is the cause of this mans
hypertension?
A. * Primary hypertension

B. Hypothyroidism
C. Primary hyperaldosteronism
D. Renovascular hypertension
E. Pheochromocytoma
165. A 73-year-old frail white woman is seen for preoperative assessment of
kidney function before aortic valve replacement. Body weight is 46 kg (101
Ib). The serum creatinine concentration is 1.6 mg/dL, and results of urinalysis
are normal. In evaluating and classifying patients with chronic kidney
disease, the National Kidney Foundation recommends estimating the
patients glomerular filtration rate. Which of the following statements is true?
A. Measurement of serum creatinine is the best predictor of
glomerular filtration rate, independent of the patients age, body
weight, and sex
B. Calculation of the timed 24-hour creatinine clearance is the clinical
gold standard for estimating glomerular filtration rate, as it is
simple and reproducible
C. Measurement of the clearance of 125I-iothalamate or inulin is the
most accurate measurement of glomerular filtration rate and should
be applied to all patients
D. * The glomerular filtration rate should be estimated by using
prediction equations (Cockcroft-Gault or Modification of Diet in
Renal Disease) that take into account serum creatinine
concentration, age, body weight, and sex
E. No correct answer
166. A 74-year-old man is hospitalized with cough and chest pain. He was
previously healthy and has not been seen by a physician in more than 14
years. At a health fair 1 year ago, he had a blood pressure check and blood
and urine tests, but he did not return for a scheduled office examination. He
came to the office at the request of his wife. On physical examination, the
blood pressure is 148/92 mm Hg, with no orthostatic changes; heart rate,
75/min; respiratory rate, 18/min; and temperature 37.8 C (100 F). There is
no neck vein distention or hepatojugular reflux. The cardiac examination is
normal. The left lower lung field shows increased fremitus, dullness to
percussion, and scattered basilar crackles. No lower extremity edema is
present. The electrocardiogram is normal. Hematocrit is 34 %, and
leukocytosis is present with a normal platelet count. The serum creatinine
concentration is 2.3 mg/dL. Urinalysis shows a pH of 6.0, 1 + proteinuria, and
no hematuria or ketonuria. No formed elements appear on microscopic
examination. What is the most important next step in determining the
diagnosis of the decreased renal function in this patient?
A. Calculate the ratio of blood urea nitrogen to creatinine
B. Obtain renal ultrasonography
C. * Obtain creatinine clerence
D. Obtain previous serum creatinine concentration and urinary protein
excretion

E. Obtain previous hematocrit

167. You are asked to see the patient 2 days postpartum to assist in
managing persistently elevated blood pressure. The patients only symptom is
mild frontal headache. She is alert and oriented, and she appears well. Blood
pressure is 175/95 mm Hg, with no orthostatic changes; pulse rate is 84/min;
respiratory rate is 18/min; and temperature is 38 C (100.4 F). She has no
evidence of retinopathy. There is no neck vein distention or hepatojugular
reflux. Cardiopulmonary examination is normal. No lower extremity edema is
present. You recommend increasing the dose of -blocker and adding
hydralazine. Blood pressure on day 3 postpartum is 130/75mm Hg. You find
the following laboratory studies in the chart: Postpartum day 2 Postpartum
day 3 Hemoglobin 12.8 g/dL 11.0 g/dL Platelet count 180,000/?L 120,000/?L
Serum glucose 89 mg/dL 97 mg/dL Blood urea nitrogen 24 mg/dL 36 mg/dL
Serum creatinine 1.2 mg/dL 2.9 mg/dL Serum sodium 134 meq/L 132 meq/L
Serum potassium 3.8 meq/L 4.2 meq/L Serum chloride 99 meq/L 100 meq/L
Serum bicarbonate 24 meq/L 20 meq/L You request a urine sample for
microscopy, and the patient produces only 5 mL of blood-tinged urine. She
comments that this is the first time she has urinated since last night.
Urinalysis shows specific gravity of 1.009, positive dipstick hematuria, trace
proteinuria, and small urobilinogen with no cellular casts. What is the next
appropriate step in management of this patient?
A. Aggressive intravenous saline infusion for volume repletion
B. * Emergency renal ultrasonography to rule out obstructive uropathy
C. Repeat complete blood count and inspection of blood smear.
D. Two sets of blood cultures and ticarcillin-clavulanate therapy for
empiric coverage of early sepsis.
E. Transfer to intensive care unit and start dopamine infusion

Вам также может понравиться